rmm 6.pdf

76
CUPRINS - 0 - Cuvânt înainte din partea colectivului redacţional ____________ 1 Membrii Filialei Mehedinţi a S.S.M.R. ________________________ 2 Per aspera ad astra - Eduard Băzăvan __________________ 3 Andrei Ungureanu - la finalul carierei olimpice de elev în România _ 5 A doua ediţie a Concursului interjudeţean “Petre Sergescu” Programul Simpozionului National ______________________________ 7 Subiectele de concurs __________________________________________ 9 Rezultatele concursului __________________________________________ 12 O generalizare a problemei celor 12 bile ________________________ 15 Elemente de probabilităţi geometrice ________________________ 17 Asupra unor identităţi trigonometrice condiţionate ____________ 22 Generator de probleme __________________________________________ 23 Primul concurs judetean de geometrie Gheorghe Ţiţeica ______ 25 De la Severin la Rutgers ____________________________________ 27 Teme pentru grupele de performanţă Clasa a V-a Metode de rezolvare a problemelor de matematica ____________ 29 Clasa a VII-a Formule de calcul prescurtat ____________________________________ 32 Clasa a VIII-a Probleme de algebră cu soluţii geometrice ________________________ 33 Clasa a IX-a Algoritmul „Plicului” __________________________________________ 35 Clasa a IX-a Teorema lui Casey __________________________________________ 38 Clasa a X-a Comentarii metodica asupra unor probleme date la Olimpiadele SUA 40 Clasa a X-a Tetraedrul şi capcanele lui ____________________________________ 42 Clasa a XI-a Asupra teoremei lui Knaster ______________________________ 44 Clasa a XII-a Caracteristica unui inel ____________________________________ 45 Clasa a XII-a Teoremele lui Sylow __________________________________________ 49 Clasa a XII-a Asupra polinomului caracteristic ______________________________ 51 mATh Studio ________________________________________________ 55 Probleme propuse ____________________________________ 56 Premianţii concursurilor din 2006 ______________________________ 62 Rubrica rezolvitorilor ____________________________________ 67

Upload: silviu-boga

Post on 15-Jan-2016

199 views

Category:

Documents


6 download

TRANSCRIPT

Page 1: RMM 6.pdf

CUPRINS

- 0 -

Cuvânt înainte din partea colectivului redacţional ____________ 1 Membrii Filialei Mehedinţi a S.S.M.R. ________________________ 2 Per aspera ad astra - Eduard Băzăvan __________________ 3 Andrei Ungureanu - la finalul carierei olimpice de elev în România _ 5 A doua ediţie a Concursului interjudeţean “Petre Sergescu” Programul Simpozionului National ______________________________ 7 Subiectele de concurs __________________________________________ 9 Rezultatele concursului __________________________________________ 12 O generalizare a problemei celor 12 bile ________________________ 15 Elemente de probabilităţi geometrice ________________________ 17 Asupra unor identităţi trigonometrice condiţionate ____________ 22 Generator de probleme __________________________________________ 23 Primul concurs judetean de geometrie Gheorghe Ţiţeica ______ 25 De la Severin la Rutgers ____________________________________ 27 Teme pentru grupele de performanţă

Clasa a V-a Metode de rezolvare a problemelor de matematica ____________ 29

Clasa a VII-a Formule de calcul prescurtat ____________________________________ 32 Clasa a VIII-a Probleme de algebră cu soluţii geometrice ________________________ 33 Clasa a IX-a Algoritmul „Plicului” __________________________________________ 35 Clasa a IX-a Teorema lui Casey __________________________________________ 38 Clasa a X-a Comentarii metodica asupra unor probleme date la Olimpiadele SUA 40 Clasa a X-a Tetraedrul şi capcanele lui ____________________________________ 42 Clasa a XI-a Asupra teoremei lui Knaster ______________________________ 44 Clasa a XII-a Caracteristica unui inel ____________________________________ 45 Clasa a XII-a Teoremele lui Sylow __________________________________________ 49 Clasa a XII-a Asupra polinomului caracteristic ______________________________ 51 mATh Studio ________________________________________________ 55 Probleme propuse ____________________________________ 56 Premianţii concursurilor din 2006 ______________________________ 62 Rubrica rezolvitorilor ____________________________________ 67

Page 2: RMM 6.pdf

REDACTIA

- 1 -

H S S M

Page 3: RMM 6.pdf

EDITORIAL

H SSM

- 1 -

Cuvânt înainte din partea colectivului redacţional

Aparem astazi in fata cititorilor nostri cu al saselea numar al revistei RMM consecventi propunerilor initiale de a aduce in paginile revistei evenimentele matematice din judetul nostru atat cat ele sunt cunoscute prin intermediul colaboratorilor nostri membrii activi ai Filialei Mehedinti a Societatii de Stiinte Matematice din Romania. Pentru a stimula si alti colegi profesori de matematica si nu numai sa ni se alature in activitatea noastra avand ca principal obiectiv asa cum reiese din statutul Societatii sa ridicam nivelul cunostintelor de matematica ale tinerilor , publicam in acest numar membrii Filialei la 31 decembrie 2005. Reamintim cititorilor ca prima conditie de a deveni membru al Societatii noastre este de a achita la zi cotizatia de 10 lei pe an. Consiliul Filialei va avea de grija ca acesti bani sa fie folositi in conformitate cu statutul si hotararile luate in Adunarile Generale ale Filialei Mehedinti a SSMR. Anul 2006 a debutat din punctul nostru de vedere cu a doua editie a Concursului Interjudetean Petre Sergescu, ecourile acestui eveniment le veti gasi in paginile revistei noastre. De asemenea folosim paginile revistei pentru a anunta ca pe 19 Ianuarie 2007 elevii claselor IV-XII impreuna cu cadrele lor didactice sunt asteptate la cea de-a treia editie a concursului. Persoane de contact: prof. Gheorghe Căiniceanu - [email protected] , prof. Prajea Manuela – [email protected] sau prof. Dan Nănuţi – [email protected] . Pe 31 ianuarie s-a desfasurat pentru a cincea oara in Colegiul National Traian American Mathematic Competition, rezultate si comentarii veti gasi de asemenea aici. Concursuri interjudetene, Olimpiada judeteana , Olimpiada nationala, Balcaniada, Olimpiada internationala, toate sunt reprezentate in paginile revistei noastre prin prisma rezultatelor obtinute de elevii nostrii. Nu putem trece mai departe fara sa amintim aici ca, anul 2006, a reprezentat prin medaliile de aur la OIM ale elevilor nostrii de exceptie UNGUREANU ANDREI BOGDAN si BAZAVAN EDUARD, cel mai bun an din istoria concursurilor de matematica pentru judetul nostru. Ne bucuram ca in paginile Gazetei Matematice Metodice Nr.2/2006, revista noastra este recenzata alaturi de alte publicatii prestigioase pe taram matematic din tara. Inauguram in acest numar o rubrica noua, si anume prezenterea unor scurte biografii a unor elevi de-ai judetului nostru care cu ajutorul matematicii si-au croit un drum in viata in tara si pe alte meleaguri. Fac apel la colegii nostrii ai caror elevi de exceptie au ales o cariera legata de matematica sa ni se adreseze astfel ca publicand aceste biografii sa stimulam pe mai tinerii matematicieni in activitatea lor de studiu si cercetare. In speranta ca si acest numar al revistei noastre dupa ce va fi ajuns pe mesele de lucru ale matematicienilor mehedinteni si nu numai va reprezenta un prilej de satisfactii intelectuale va adresam rugamintea sa va alaturati noua in viitor atat prin materiale interesante cat si prin popularizarea revistei in randurile elevilor si cadrelor didactice.

Presedintele Filialei Mehedinti a SSMR,

Profesor doctor Gh.Cainiceanu

Page 4: RMM 6.pdf

EDITORIAL

- 2 -

Membrii Filialei Mehedinţi a S.S.M.R.

- 31.XII.2005 -

1 Albu Elena Sc.9 43 Lugoj Tanta Decebal 2 Antonie Rodica CNT 44 Lupu Adrian Decebal 3 Badaluta Anghel Sc.6 45 Mada Gheorghe Decebal 4 Bala Dumitru C.Univ. 46 Malineanu Gabriela Sc.4 5 Baloi Valeria Odobleja 47 Marin Felicia Sc.9 6 Balu Nicoleta Odobleja 48 Moclea Patrutescu Adr Sc.11 7 Barbulescu Marin Baia 49 Nanuti Dan CNT 8 Bejenaru Laviniu CNT 50 Nedeianu Dan Dl.Tudor. 9 Bizdoaca Claudia C.E. 51 Oprita Manuela Decebal

10 Bobic Nicolae Lalescu 52 Paponiu Dana CNT 11 Bogdan Dorel LM Orsova 53 Pasov Nicoleta L.Auto 12 Bondoc Gabriela L.Auto 54 Paulescu Petre Baia 13 Bondoc Lucian Tribunal 55 Petrache Elena LM Orsova 14 Bosneagu Dina Sc.6 56 Pit Marica Sc.5 15 Budanescu Felicia Sc.9 57 Pit Vasile CNT 16 Buzatu Carmen Dl.Tudor. 58 Ploscaru Cornel Baia 17 Cainiceanu Gheorghe CNT 59 Popescu Eleodor CNT 18 Calafeteanu Gheorghe Sc.9 60 Popescu Marcel Odobleja 19 Carbunaru Dumitru Sc.4 61 Popescu Rodica Ţiţeica 20 Chilea Ion Dl.Tudor. 62 Prajea Manuela CNT 21 Ciuca Ionel Dl.Tudor. 63 Presneanu Doru Odobleja. 22 Coada Carmen Sc.2 64 Pupaza Ecaterina Decebal 23 Constantin Magdalena Simian 65 Raducan Emilia Decebal 24 Cristel Elena Dl.Tudor. 66 Ramniceanu Elena Sc.11 25 Croitoru Ion C.E. 67 Sitaru Daniel C.E. 26 Dan Daniel Ţiţeica 68 Stoican Victor Baia 27 Doman Elena Baia 69 Stretcu Daniel Ţiţeica 28 Draga Tatucu Porfirel Simian 70 Stuparu Dragos C.Univ. 29 Dutu Constantin Sc.2 71 Tatucu Mariana Ţiţeica 30 Fallon Florica LM Orsova 72 Ticusi Ovidiu Odobleja 31 Farago Alexandru Lalescu 73 Tomita Vasile C.Univ. 32 Florescu Violeta Sc.4 74 Tudorescu Loredana Baia 33 Fritea Eugen Dl.Tudor. 75 Tudose Elena LM Orsova 34 Gimoiu Iuliana CNT 76 Ungureanu Octavian Ţiţeica 35 Giugiuc Constantin Dl.Tudor 77 Untaru Ilie Odobleja 36 Giugiuc Leonard CNT 78 Vaduva Ion Odobleja 37 Gorun Sanda Lalescu 79 Varzaru Mariana L.Auto 38 Grecu Vasile C.E. 80 Vasilcan Florentina Dl.Tudor. 39 Hinoveanu Sorin Sc.6 81 Vasilcanu Octavian Sc.6 40 Ianasi Ion Sc.4 82 Vasiluta Ion Dl.Tudor. 41 Ionescu Adela C.Univ 83 Vuc Ionela Lalescu 42 Lazar Alexandru Baia

Page 5: RMM 6.pdf

EDITORIAL

H SSM

- 3 -

Per aspera ad astra Eduard Băzăvan

elev Noël Negrea, Liceul Teoretic ,,Gh. Ţiţeica”

Mă întreb câte epitete am putea adăuga, chiar şi în joacă, numelui Eduard Băzăvan. Nu sunt

sigur. Ştiu doar că, oricât m-aş strădui, aş lăsa pe dinafară prea multe. E lucrul care mă determină să

nu pornesc în ,,temerara” odisee sortită ab initio eşecului. Chiar dacă, printr-o minune, aş reuşi să le

găsesc pe toate, rezultatul ar fi acelaşi: detaşarea discretă cu care „destinatarul” şi-ar declina tăcut

meritele, ca pe lucrul cel mai natural de pe lume... Am avut privilegiul nemăsurat de a mă găsi o vreme – e drept, nu atât de multă pe cât mi-aş fi

dorit! - în preajma unui om care a înţeles ca acceptarea adulaţiei celorlalţi l-ar supune la un efort extenuant şi nociv pentru drumul pe care şi l-a croit prin truda minţii către o lume în care nu şi-a găsit

locul decât percepţia obiectivă. Şi asta pentru că, de ani buni, Eduard învăţase să nu admită năruirea

carierei - pe care şi-o clădea cu minuţie – prin alterarea atitudinilor propriei minţi. Ce preţ va fi trebuind să plătească pentru răbdarea şi efortul lui – ingrediente indispensabile

reuşitei fără cusur - nu vom putea şti poate niciodată... In schimb, ştim cu certitudine altceva: Fără să se teama a fi, chiar şi pentru o singura clipă, excentric în opinii şi fapte, Eduard nu a

fost niciodată, ceea ce un matematician prolific - dar şi super excentric precum Pál Erdős – numea „un dispozitiv de turnat cafea în teoreme”... Dacă s-ar fi întâmplat asta, nu am fi aflat până azi că activitatea

intelectuală corect dirijată nu denota o deficienţa vitală sau un handicap, ci aşa cum, cu discreţie ne-a fost revelat, un mod superior de a fi, un privilegiu şi nicidecum o carenţă... Mai mult, a fost un fericit prilej ca să tălmăcim fără greş ceea ce Aristotel spunea în „Metafizica”: „Înainte de orice, ştiinţele

matematice exprimă ordinea, simetria şi limitarea, iar acestea sunt cele mai măreţe forme prin care se

manifestă frumosul”. Dacă ar fi să numim doar un singur merit pe care îl are Eduard Băzăvan, am putea afirma că îi

datorăm transformarea matematicii pure într-o măiestrită poema de idei logice. E idealul pe care,

citându-l pe William James, doar cei aleşi au reuşit să îl atingă: fervoarea împletită cu simţul măsurii

dublate de pasiunea pusa în slujba găsirii adevărului pe calea cea mai dreaptă cu putinţă şi care nu are

nevoie să se sprijine pe nici un subterfugiu ca să conducă invariabil spre râvnitul deziderat - excelenţa

supremă în dezarmanta ei simplitate. In numele ei stă astăzi mărturie o galerie impresionanta de

premii, medalii şi diplome care se vor fi adunat laolaltă de-a lungul anilor de şcoală. Pentru a cuantifica

totul, ai avea nevoie de timp îndelungat şi rezultatul ar fi, fără îndoială, acelaşi. Ai ajunge să vezi - pentru a câta oara? – cât de justă ar fi observaţia pe care Eduard o face de fiecare dată când, într-un fel sau altul, cineva ajunge să vorbească despre meritele lui incontestabile : „Pentru mine, răsplata cea

mai mare e o ipostază care mă obliga la mai mult!” Modestia sa înnăscută este dublată de floarea rară a recunoştinţei faţă de oamenii care i-au

marcat destinul. Consideră că profesorul de matematică din liceu, domnul Daniel Stretcu, este omul

providenţial care l-a susţinut, stimulat neîncetat pentru a atinge perfecţiunea pură a matematicii.

Dialogurile platoniciene au fost strămutate cu succes în solul ştiinţelor exacte. Atunci când a decis să urmeze cursurile Politehnicii din Bucureşti pentru a deveni inginer, a fost

întrebat, în glumă, dacă este de acord cu „teoria” că singurul membru care nu se va atrofia complet la

om va fi degetul cu care apasă butoanele maşinăriilor. A răspuns cu un citat din Bertold Brecht: „Scopul

ştiinţei nu este acela de a deschide uşa înţelepciunii infinite, ci acela de a fixa o limită pentru eroarea

infinită”. Cu un asemenea feedback, orice polemică cu el ar deveni deplasată. Ştim doar că e un adept

al ideii că prin adresarea necontenită de întrebări „impertinente” se menţine tot timpul pe un drum care speră să-l conducă, într-o zi, la Răspunsul Pertinent pe care îl caută cu înfrigurare sfidând constant şi

fără temeri dureroasa axioma: „Ştiinţa este topografia ignoranţei”... Încercând să dibuim cheia în care s-a clădit Succesul sau am crezut că acesta stă ascuns în

spatele unor abilităţi cu totul ieşite din comun şi care sunt menite numai celor care au puterea să

forţeze în fiecare clipă graniţele curiozităţii. Nu ştiu daca rezultatul căutărilor noastre a fost un succes

Page 6: RMM 6.pdf

EDITORIAL

- 4 -

sau doar o mare dezamăgire: ne-am plecat capetele după ce am învăţat de la Eduard că abilităţile

speciale se pierd iremediabil în eter atunci când nu sunt dublate de atitudini inteligente. Cât despre sofisticatele mecanisme pe care le explorează intens mintea lui nu am aflat decât ce reuşise el însuşi

să retină de la Alexander Chase: ,,Mintea este şi acel lucru cu care omul a fost dotat ca să uite”, nu

doar să construiască lucruri minunate... Dar, ca să nu uite şi ca să poată crea toate lucrurile acelea minunate pe care le-a subsumat

unui palmares greu de atins – dar şi mai greu de depăşit în condiţii normale – Eduard a ştiut să îşi

menţină vie curiozitatea, să o lase să-l ,,macine”, conştient că dacă plictiseala se vindecă prin

curiozitate, pentru cea de pe urma nu s-a aflat încă vreun leac… Şi deocamdată este bine că aceasta

descoperire nu a fost făcută pentru că, aşa cum ne-a spus-o chiar el, dacă nu ar mai exista curiozitate

fără vindec, oamenii – mai precis, categoria din care el face parte – nu ar mai avea prea multe şanse

să devină experţii care ştiu tot mai mult despre tot mai puţine lucruri… Cu astfel de gânduri a absolvit liceul ,,Gheorghe Ţiţeica” prietenul nostru Edi Băzăvan. Am

înţeles la despărţire că lecţia pe care am primit-o de la el este una fără seamăn, nepreţuită şi pe care, cu un aport din parte-ne, trebuie să o lăsăm ca pilda celor care vor păşi pe coridoarele liceului nostru

atunci când vom pleca la rândul nostru, vom alege sa păşim pe drumuri noi, în căutarea adevărurilor ce

nu se lasă uşor desferecate. Vom merge şi noi mai departe, atunci când va sosi clipa despărţirii de

lăcaşul de cultura care ne deschide încă şi ne lărgeşte în fiecare zi drumul spre Adevăr. Iţi mulţumim, Eduard Băzăvan, pentru că ne-ai ajutat să înţelegem că noi - cei care azi suntem

încă elevi ai liceului în care ai adăstat timp de patru ani – trebuie să fim un ideal al vremurilor ce vin şi

nu doar un modest rezultat al trecutului… ,,Ce minune că eşti, ce întâmplare că sunt!” (Nichita Stănescu)

N.B. Pe parcursul celor patru ani – până în vara lui 2006 – cât a fost elev al Liceului “Gh. Ţiţeica”, Eduard Băzăvan a obţinut următoarele distincţii :

Clasa a IX - a Premiul II la Olimpiada de Matematică , faza Judeţeană Menţiune la Olimpiada de Matematică , faza Naţională Mentiune la Concursul Interjudeţean de Matematică “Ion Ciolac” , Craiova

Mentiune la Concursul Interjudeţean de Matematica “Recreaţii Matematice” , Iaşi Premiul I la Concursul revistei de Matematică Alpha

Premiul I la Concursul Interjudeţean de Matematică “Gheorghe Ţiţeica” Premiul III la concursul de matematică “Cangurul”, faza Naţională

Premiul III la concursul de matematică “MCM” , Poiana Pinului Premiul III la Concursul Interjudeţean de Informatică “Info-Oltenia”

Clasa a X-a Premiul II la Olimpiada de Matematică , faza Judeţeană Medalie de Argint la Olimpiada de Matematică , faza Naţională

Medalie de Bronz la Olimpiada Internaţională Pluridisciplinară “Tuymaada” 2004 Premiul I la Olimpiada de Informatică , faza Judeţeană Premiul III la Concursul Interjudeţean de Matematică “Gheorghe Dumitrescu” Premiu Special la Concursul Naţional de Fizică-Matematică “Vrănceanu Procopiu” Menţiune la Concursul Interjudeţean de Matematică “Nicolae Păun” Premiul II la Concursul de Matematică “Cangurul”, faza Naţională

Clasa a XI-a Premiul II la Olimpiada de Matematică, faza Judeţeană Medalie de Argint la Olimpiada de Matematică, faza Naţională Premiul III la Olimpiada de Informatică, faza Judeţeană Premiul II la Concursul Interjudeţean de Matematică “Gheorghe Dumitrescu” Premiul II la Concursul Interjudeţean de Matematică “Petre Sergescu”

Clasa a XII - a Premiul I la Olimpiada de Matematică, faza Judeţeană Medalie de Aur la Olimpiada de Matematică, faza Naţională

Medalie de Aur la Olimpiada Balcanica de Matematică, Cipru 2006 Medalie de Aur la Olimpiada Internaţională de Matematică, Slovenia 2006 Premiul I la Concursul de Matematică “Cangurul”, faza Naţională Premiul II la Concursul Interjudeţean de Matematică “Gheorghe Dumitrescu”

Page 7: RMM 6.pdf

EDITORIAL

H SSM

- 5 -

Andrei Ungureanu

La finalul carierei olimpice de elev în România

Ne-am despartit anul acesta, noi, Colegiul National „Traian”, de unul dintre cei mai straluciti elevi ai scolii traianiste, de Andrei Bogdan Ungureanu, absolvent promotia 2006 si actualmente student al Universitatii „Princeton”-Statele Unite ale Americii. Medalia de Aur şi Premiul I la Olimpiada Internaţională de Matematică desfăşurată în perioada 10-20 iulie 2005 -Marena - Mexic obţinută de olimpicul internaţional

Andrei Bogdan Ungureanu îl situează pe tânărul matematician pe primele locuri pe

mapamond şi confirmă încă o dată valoarea incontestabilă a acestuia pe plan internaţional.

Performanţa unică în judeţul Mehedinţi până la ora actuală (medaliat cu aur la olimpiada

internaţională), rezultatul lui Andrei Bogdan Ungureanu se conturează după o carieră

olimpică de succes începută cu mulţi ani în urmă. Premiul I la toate olimpiadele judeţene şi naţionale (începând cu cls. a V-a şi până în

prezent – actualmente este elev în clasa a XI-a) ar fi suficiente pentru a-l situa pe Andrei pe treapta cea mai de sus a olimpiadelor naţionale. Dar rezultatele nu se opresc aici pentru că în spatele aparenţelor avem un talent nativ,

o inteligenţă remarcabilă susţinută cu succes de un caracter de învingător, de o performanţă

uimitoare, de putere şi dăruire. Şi pentru că gloria nu se câştigă fără sacrificii, avem multe ore

de muncă, multă ambiţie şi hotărâre în dorinţa de birui. Evoluţia lui Andrei pe scara internaţională a început gradat, treaptă cu treaptă, baraj

cu baraj (pentru selectarea lotului olimpic naţional se susţin în fiecare an alte 6 baraje),

medalie după medalie la fiecare participare internaţională. În anul 2002, elev pe atunci în clasa a VIII-a, Andrei Bogdan Ungureanu obţine

Medalia de Aur la Olimpiada Balcanică de Matematică pentru juniori, Târgu Mureş-România. Urmează apoi trecerea la liceu, alte începuturi, alte standarde, pe undeva o altă

matematică: arie extinsă ca volum şi subtilitate, probleme adânci, acuitate maximă,

ingeniozitate în creaţie şi construcţie. În ciuda terenului nebătătorit, reuşim să doborâm obstacolele, să ne ridicăm deasupra

greutăţilor şi Andrei se califică în lotul pentru seniori al României (concurează acum cu elevii

claselor IX-XII pe acelaşi tip de subiecte) obţinând Medalia de Bronz la Olimpiada de

Matematică Pluridisciplinară din Iacuţia-Rusia. Trecerea la seniori şi selecţia în lotul României în fiecare an au fost probe foarte dure

atât din punct de vedere ştiinţific cât şi psihologic: rezistenţă la efort, emoţii, presiuni,

concentrare. În clasa a X-a, anul 2004, a obţinut Medalia de Argint la Olimpiada Balcanică de

Matematică – Plevna - Bulgaria şi Medalia de Argint la Olimpiada Internaţională de

Matematică – Atena - Grecia – succes răsunător pentru prima participare la olimpiada internaţională şi pentru vârsta fragedă a lui Andrei. În clasa a XI-a, anul 2005, situaţia se complică întrucât curriculum-ul pentru olimpiada naţională este altul decât cel pentru internaţionale şi Andrei reuşeşte în paralel să le

parcurgă pe amândouă la cote maxime. Premiul I la Olimpiada Naţională de Matematică, calificarea în lotul României urmate

apoi de rezultatele internaţionale: Medalia de Bronz la Olimpiada Balcanică de Matematică – Iaşi- România şi în sfârşit împlinirea visului din copilărie, succes şi lacrimi de bucurie – Medalia de Aur la Olimpiada Internaţională de Matematică – escaladarea olimpică

Page 8: RMM 6.pdf

EDITORIAL

- 6 -

ajunsă la apogeu, Andrei a reuşit să transforme visul în realitate.Si cum istoria se

repeta,acelasi succes se reediteaza si in 2006- Medalia de Aur la Olimpiada Internaţională

de Matematică-Slovenia. Andrei a obţinut primul loc şi la concursurile americane American Mathematics

Competitions şi American Invitational Mathematics Examination (echivalentele olimpiadelor locale şi judeţene din România), fiind singurul calificat pentru faza finală U.S.A.M.O.

(United States American Mathematics Olympiad). Punctajul foarte bun obţinut l-a remarcat în S.U.A. drept pentru care a primit o bursă pentru olimpici cu durata de un an din partea unei instituţii americane de selecţie a performerilor: Exeter Academy – New Hampshire SUA. Andrei nu a onorat însă această bursă întrucât a considerat că pregătirea în România îl va

propulsa foarte bine în ierarhia valorilor mondiale. Elev eminent la toate disciplinele, Andrei a avut din clasa a V-a până în prezent,

media generală 10, a participat şi la olimpiadele judeţene de fizică şi informatică în clasa a

VIII-a şi a IX-a, obţinând numeroase premii I. Colegiul Naţional „Traian” a fost dintotdeauna o şcoală a elevilor, a performanţelor, a

olimpicilor iar succesele obţinute de Andrei Ungureanu ne determină să-l declarăm ca fiind

cel mai bun elev al acestuia de la începuturi şi până în prezent şi totodată cel mai valoros

olimpic al tuturor timpurilor pentru şcoala matematică mehedinţeană. Elevă fiind, am fost şi eu olimpică la rândul meu (premiul I, II, III la olimpiadele

judeţene, naţionale şi interjudeţene), dar nu am beneficiat de o atenţie aparte în pregătirea

mea. Când mi-am ales să fiu profesor, am ştiut că voi sprijini cu orice preţ şi sacrificii elevii

cu înclinaţii deosebite căci numai aşa succesele pot fi maxime. Am creat mai mulţi olimpici

naţionali dar din prima clipă când l-am întâlnit pe Andrei (în clasa a VIII-a) am ştiut că

aceasta este o şansă unică în viaţă. Îmi amintesc de prima impresie a primei întâlniri: rapiditatea, ingeniozitatea şi

naturaleţea cu care rezolva problemele (în demersul a două ore ajunsesem deja la olimpiadele

naţionale de clasa a X-a în mod firesc) mi-au umplut sufletul de bucurie şi efortul era cu atât

mai mare cu cât nu puteam să-i arăt acest lucru pentru că eram abia la început de drum. A urmat apoi o muncă de apostolat fără ezitări şi renunţări, lecţiile noastre durau 4-5

ore în fiecare duminică, le pregăteam cu eforturi (dificultate, concentrare, învăţare, timp)

fiind destul de greu să ne ridicam la standardele impuse de ambitiile noastre. S-a depus mult suflet în acest vis, satisfacţiile sunt pe măsură şi desi Andrei studiaza

acum in polul universitar al inteligentelor mondiale, satisfactiile noastre, ale celor care l-am creat si format, parinti si profesori, nu se opresc aici.Vom mai auzi de Andrei Ungureanu,ne vom mandri cu el peste ani si, cand inceputurile lui vor fi fost date deja uitarii, vom putea spune cu modestie: ”l-am cunoscut candva”.

Si nu pot sa nu imi amintesc spusele istoricului, scriind despre menirea omului potrivit la locul potrivit si despre opera unei vointe: „poate s-ar fi dezvoltat el si fara aceasta vointa, insa cine stie cum si cine stie cand” . Un exemplu, un mentor, o vointa, nu putem exprima in cuvinte ceea ce simtim cu totii atunci cand ne gandim la Andrei si la performantele lui.

Manuela Prajea

Page 9: RMM 6.pdf

Petre Sergescu

H

- 7 -

SSM A doua ediţie a Concursului interjudeţean

“Petre Sergescu” -Drobeta Tr. Severin-

20 ianuarie 2006 Asa cum anuntasem in numarul 5 al revistei noastre, pe 20.01.2006 a avut

loc la Colegiul National Traian a doua editie a Concursului Interjudetean de matematica Petre Sergescu.La concurs au participat peste 600 de elevi din judetele Mehedinti, Dolj, Gorj, Caras-Severin, Timis.La Simpozionul National dedicat acestui eveniment au participat un numar mare de cadre didactice, sustinandu-se un numar de 38 de Referate,dupa cum reiese din programul de mai jos.

Programul Simpozionului National “PETRE SERGESCU” 20.01.2006

1. Dezvoltarea creativitatii in cadrul orelor de matematica Inspector General Doinita Mariana Chircu, Lector universitar doctorand asociat

2. Combinatorica, o perspectiva moderna in sfera concursurilor de matematica Prof.doctorand Prajea Manuela CNT

3. Consideratii metodice asupra unor probleme date la olimpiadele din SUA Profesor dr.Cainiceanu Gheorghe CNT

4. Bertrand Russel matematician si filosof Prof.Stoica Rodica, Director adjunct al Colegiului National Traian

5. Viata si opera lui gheorghe titeica Prof.Falon Florica Liceul de Marina Orsova

6. Generator de probleme Prof. Bejenaru Laviniu CNT

7. Caracteristica unui inel Prof. Dan Nedeianu Lic. D-l Tudor Drobeta Tr.Severin

8. Asupra unor identitati trigonometrice Prof.Lucian Dragomir Liceul Otelul Rosu

9. Problematicul in probleme Prof. Gh. Calafeteanu Sc.Gen. 9 DrobetaTurnu Severin

10. Opera matematica a lui Grigore Moisil Prof.Florin Ghiocel CNT

11. Proprietati geometrice ala conicelor Prof.Gimoiu Iuliana CNT.

12. Spectrul unei matrici prof.Paponiu Dana CNT

13. Metode si tehnici de rezolvare a problemelor de aritmetica Institutor Teis Alina, Lic.Ped.Stefan Odobleja.

14. Conditii pentru echivalenta unor clase de functii elementare Prof.Adi Lupu, Grup Scolar Decebal

15. Problema lui Titeica Prof.Adriana Patrutescu Moclea, Sc.Gen.11

16. Viata si opera lui David Emmanuel Prof.Doru Presneanu, Liceul Pedagogic Stefan Odobleja

17. Algebra in ajutorul algebrei Prof. Farago Alexandru Liceul TL Orsova, Farago Gabriela si Raescu Gheorghita ,Sc.Gen.Orsova

18. Teorema lui rolle si aplicatii Prof.Dan Nanuti ISJ, Prof.Bizdoaca Claudia Colegiul Economic

19. Ştefan Odobleja si psihologia consonantista Prof.Emanuela Busoi Colegiul National Traian

20. Profesori de prestigiu ai invatamantului mehedintean

Page 10: RMM 6.pdf

Petre Sergescu

- 8 -

Prof.Stefan Marica, pensionar 21. Metode generale de rezolvare a problemelor de aritmetica

prof. Hinoveanu Sorin, Sc.Gen.6 22. Demonstratii ale teoremei lui Ţiţeica

Prof. Tache Oana ,Sc.Gen Craguesti 23. Transformari geometrice si aplicatii

Prof.Drula Ileana,Strehaia 24. Academician Gheorghe Calugareanu 30 de ani de la moarte

Prof. Roman Florentina,Lic.Matei Basarab, Strehaia 25. Probleme de geometrie...inselatoare

Prof. Leonard Giugiuc, CNT 26. Matrici speciale

Prof. Giugiuc Constantin, Lic.Domnul Tudor 27. Definirea functiilor elementare cu ajutorul sirurilor

Prof.Raducan Emilia, Grupul Scolar Industrial Decebal 28. Aplicatii ale seriilor in studiul functiilor elementare

Prof. Pupaza Ecaterina, Grupul Scolar Industrial Decebal 29. Probabilitati geometrice

Prof.Popescu Tudor, Colegiul F.Nitti, Timisoara 30. Algoritmii - puntea dintre matematica si informatica

Prof.Cristina Draghescu, Grupul Scolar Industrial Decebal 31. Marele matematician Gheorghe Ţiţeica

Prof. Draga Tatucu Mariana ,Liceul Gheorghe Titeica 32. Problema celor 12 bile

Prof.Stoican Victor, Grup Sc.Ind.Baia de Arama 33. Idei antice, lectii contemporane

Prof.Baloi Valeria, St.Odobleja. 34. Viata si opera marelui matematician Gheorghe Ţiţeica

Profesori Adriana Nanuti Liceul Gh.Titeica, Prof.Dana Mosor ISJ 35. Probleme de aritmetica, tehnici si metode de rezolvarte

Prof.Victor Saceanu Sc.Generala Nr.11 36. Şiruri definite prin relatii de recurenta de ordinul II

Prof. Buzatu Carmen Liceul Domnul Tudor. 37. Asupra primitivelor unor functii compuse

Prof. Bacila Alin Colegiul George Cosbuc Motru 38. Complemente de geometrie sintetica

Prof.Antonie Rodica CNT 39. Testul grila, avantaje si dezavantaje

Prof.Popescu Eleodor CNT Chairman

Lector univ. dr. Vasile Tomiţă Centrul Universitar Drobeta Turnu Severin

In cadrul concursului adresat elevilor, asa cum se stie deja ca o noutate au fost invitati si

elevii clasei a IV-a. Numarul mare de participanti la aceasta clasa (peste 100) ne arata ca initiativa introducerii in concurs a acestei grupe de varsta a fost buna. Concursul a fost organizat ca si la prima editie de catedra de matematica din Colegiul National Traian, in colaborare cu Filiala Mehedinti a Societatii de Stiinte Matematice. Presdintele comisiei de organizare a fost Prof.drd.Prajea Manuela, director al Colegiului National Traian, vicepresedinte a fost Prof.dr.Cainiceanu Gheorghe, presedintele Filialei Mehedinti a SSMR, secretar fiind Prof. Dan Daniel de la Liceul Gh.Titeica. Subiectele au fost propuse de colegi din 14 scoli ale judetului, comisiile pe clase fiind conduse de profesorii: Piţ–Rada Marica, Sc.gen.5 la clasa a IV-a, Antonie Rodica CNT, la clasa a V-a, Gimoiu Iuliana, CNT, la clasa a VI-a, Stretcu Daniel, Gh.Titeica, la clasa a VII-a, Popescu Eleodor CNT, la clasa a VIII-a, Paponiu Dana, CNT, clasa a IX-a, Nedeianu Dan, D-l Tudor, la clasa a X-a, Sitaru Daniel, Colegiul Economic, la clasa a XI-a, Giugiuc Leonard, CNT, la clasa a XII-a.

Page 11: RMM 6.pdf

Petre Sergescu

H

- 9 -

SSM Concursul Interjudeţean de Matematică

“PETRE SERGESCU” Ediţia a II-a, 20 ianuarie 2006, Drobeta Turnu-Severin

SUBIECTE Clasa a IV - a

1) Un elev citeşte o carte care are 385 pagini. a) Ce pagini citeşte elevul , dacă suma numerelor celor două pagini este 413? b) Câte cifre s-au folosit pentru numerotarea paginilor acestei cărţi?

*** 2) Un băiat are 11 ani.

a) Determinaţi vârsta tatălui acestui băiat ştiind că atunci când băiatul va avea vârsta tatălui, tatăl va avea 59 ani.

b) Peste câţi ani vârsta tatălui va fi egală cu dublul vârstei fiului? Prof. Marica Piţ-Rada

3) Un balaur s-a născut cu 11 capete. În primii 13 ani de viaţă i-a crescut în fiecare an câte un cap. În următorii 15 ani , i-au mai crescut în fiecare an câte 2 capete. Apoi balaurul s-a îmbolnăvit şi a început să piardă în fiecare an câte 3 capete. a) Care a fost numărul maxim de capete şi la ce vârstă le-a avut? b) Câţi ani a trăit balaurul? c) La ce vârstă a avut balaurul 30 capete?

Prof.Ionel-Vasile Piţ-Rada Clasa a V-a

1) a) Determinati numerele naturale x,y,z,t,u, x<y<z<t<u astfel incat 2x + 2y +2z + 2t + 2u = 248. b) Determinati ultima cifra a numarului 22002 + 22003 + 22004 +22005 + 22006.

Prof.Antonie Rodica 2) Sa se scrie numarul 52006 ca o suma de cinci numere naturale a,b,c,d,e astfel incat resturile

impartirii fiecaruia dintre numerele a,b,c,d,e la 5 sa fie distincte. Prof.Nedeianu Dan

3) Se considera multimea M={n∈N* | n≤ 2006}. a) Stabiliti daca se poate scrie multimea M ca reuniune de submultimi disjuncte Mk astfel incat in fiecare multime Mk sa existe un element egal cu suma celorlalte. b) Sa se demonstreze ca din oricare 1005 elemente din M se pot alege doua elemente diferite astfel incat suma lor sa fie 2006.

Prof. Paponiu Dana Clasa a VI-a

1) Se considera multimea M={ba(1+a+a2)| a, b ∈ N} a)Sa se arate ca M contine o infinitate de patrate perfecte; b)Pentru b numar prim, determinati elementele lui M ce pot fi scrise ca suma a cinci numere naturale consecutive.

Prof.Dana Paponiu

2) Daca n ∈ N*, notam cu Pn produsul divizorilor naturali ai numarului n. Sa se determine n cu proprietatea ca Pn=10450.

Prof.Manuela Prajea 3) Fie [OX, [OY doua semidrepte situate in interiorul unghiului propriu <AOB, [OX ≠ [OY,

m(<AOX)=a; m(<BOX)=b,m(<AOY)=c si m(<BOY)=d. Daca ac=bd, atunci sa se arate ca: a) ab+cd=2bd b) unghiurile <AOB si <XOY au aceeasi bisectoare.

Prof.Iuliana Gimoiu

Page 12: RMM 6.pdf

Petre Sergescu

- 10 -

Clasa a VII-a

1) a) Sa se arate ca : 2

5ab = 25m , unde a,b ∈N si m= ab ( ab +1).

b) Sa se calculeze suma cifrelor numarului N , undeori20063...33.... 333 33 3 N ++++= .

Prof.Gorun Sanda, Prof.Prajea Manuela 2) Sa se determine numarul de solutii numere intregi pozitive ale ecuatiei:

2006111

=+yx

.

Prof.Stretcu Daniel 3) In trapezul ABCD cu baza mare AB, diagonala CA este bisectoarea unghiului

< BCD.Daca M este mijlocul bazei mici CD, iar BM∩AD = {N}, sa se afle masura unghiului <ACN.

Prof.Dan Nedeianu Clasa a VIII-a

1) Determinati numerele intregi a, b, c care verifica simultan inegalitatile: a2 +b +3 <5a, b2 +c +3 <5b si c2 +a +3 <5c.

Prof.Dan Nedeianu 2) Fie ABCDA’B’C’D’ un cub cu muchia de 8 cm si E∈[A’D’], F∈[C’D’] astfel

incat D’E = D’F = 2cm. Paralela prin E la D’B intersecteaza (ABB’) in H, paralela prin F la D’B intersecteaza (BCC’) in I, iar planul determinat de dreptele EH si FI intersecteaza [BB’] in O. Sa se determine Aria[EHOIF].

Prof.Gheorghe Cainiceanu 3) O societate comerciala a cumparat trei feluri de aparate cu preturile de 23 lei, 45 lei si 67 lei ,

platind pentru toate suma de 854 lei.Care este numarul total al aparatelor cumparate? Prof.Eleodor Popescu

Clasa a IX-a

1) Sa se rezolve in Z* ecuatia: ⋅=⎭⎬⎫

⎩⎨⎧

++⎥⎦

⎤⎢⎣

⎡+

2511

yxxy

yx

Profesori Dana Paponiu, Stefan Marica 2) Fie poligonul A1A2…An inscris in cercul C de centru O (n∈Ν,n≥3). Daca punctele P1,P2,…,Pn

situate pe segmentele (A1A2), (A2A3),…,(AnA1) impart segmentele orientate in acelasi raport si OP1=OP2=…=OPn , sa se arate ca A1A2…An este poligon regulat.

Prof. Manuela Prajea 3) Demonstrati ca are loc relatia:

).,0(*,

2)1(

...6

33

21

1∞∈∈

++

+++

++

++

xsiNnxn

nnx

nxxx

p

Prof. Dana Paponiu Clasa a X-a

1) Fie ba, ∈(0,1)∪ (1,∞) si funcţia f: R→(0,∞) cu proprietăţile: (i) ∈21 , xx (0,∞) cu 21 xx ≠ implică f( 1x )≠ f( 2x ). (ii) funcţia g:(0,∞)→R, g( x )=f( log a x ) ∙ f( log b x ) este constantă. Să se calculeze produsul ab şi demonstraţi că există funcţii f care satisfac condiţiile din enunţ.

Prof.Dan Nedeianu

2) Fie ( )[ ] ( )[ ]20062006113113 iiiiz ++−+−++= număr complex.

Să se arate că z este un număr real din intervalul ( 30093010 2,2 −− ). Prof. Eleodor Popescu

Page 13: RMM 6.pdf

Petre Sergescu

H

- 11 -

SSM 3) a) Să se rezolve ecuaţia: 50382006 43 =−++++ xxx , x ∈R.

b) Să se arate că: 432

24

≥−

+x

x , ⎟⎠⎞

⎜⎝⎛ ∞∈ ,

23x .

Prof. Florentina Roman Clasa a XI-a

1) Fie ( )njiijaA

≤≤=

,1; ( )

njiijbB≤≤

=,1

; ( )njiijcC

≤≤=

,1; ( )jia ji ,max, = ; ( )jib ji ,min, = ;

⎪⎩

⎪⎨

=−−

≠−−=

jii

jijic ji ,12

;

, . Să se arate că ( )( )( )[ ] 0det ≥−−− BCAABCCAB .

Prof. Daniel Sitaru 2) Fie ( ) 0≥nnx , x0=a, x1=b, x2=c, x3=d, a, b, c,d ∈R şi ( ) 5,4321 ≥−+−= −−−− nxxxxx nnnnn .

a) Arătaţi că pentru orice valori ale constantelor a, b, c, d şirul ( )nx are subşiruri convergente. b) Să se determine inf nx şi sup nx . c) Să se determine a, b c, d astfel şirul ( nx ) să fie convergent.

Prof. Gheorghe Căiniceanu 3) Să se determine funcţiile continue f: R→R care satisfac relaţia:

( ) ( ) ( ) Rxxxx

xfxf ∈++++

= ;...........1 20052

2006 ⁄ { }1− .

Prof.. Manuela Prajea Clasa a XII-a

1) a) Fie m∈R* si f,g:I → R,)cos(sin2sin

cos)(xxmx

xxf++

= , )cos(sin2sin

sin)(xxmx

xxg++

= ,

unde I⊂R este un interval pe care f si g sunt definite. Calculati ∫ ∫− .)()( dxxgdxxf

b) Fie F:R→R,F(x) = 4 ⎥⎦⎤

⎢⎣⎡π2x +

⎪⎪⎩

⎪⎪⎨

>⎭⎬⎫

⎩⎨⎧+

≤⎭⎬⎫

⎩⎨⎧−

21

2,cos3

21

2,cos1

π

πxx

xx. Aratati ca F derivabila si calculati F’(x).

Prof.Giugiuc Constantin 2) Fie p∈N, p≥ 2 si G = { },...,, 21 pAAA )(2 RM⊂ un grup cu operatia de inmultire a matricelor.

Se stie ca ∑=

p

kkATr

1)( = 0. (Tr(Ak) = suma elementelor pe diagonala principala a matricei Ak).

a) Sa se arate ca multimea ⎭⎬⎫

==⎩⎨⎧

⎟⎟⎠

⎞⎜⎜⎝

⎛−

= 8.1,4cossin

sincoskt

ktktktkt

G π satisface cerintele din enunt.

b) Sa se demonstreze ca ∑=

=p

kk OA

12 (matricea nula de ordinul 2).

Prof. Gheorghe Cainiceanu 3) Fie ( o,G ) un grup abelian cu 4n+2 elemente, n∈N*. Presupunem cunoscut faptul ca ecuatia

x2n+1=e are 2n+1 solutii. a) Aratati ca ecuatia x2=e are 2 solutii. b) Daca grupul ( o,G ) este neabelian ramane adevarata cerinta a) ?

Prof. Leonard Giugiuc

Page 14: RMM 6.pdf

Petre Sergescu

- 12 -

Concursul Interjudeţean de Matematică “PETRE SERGESCU” ediţia a II-a, 20 ianuarie 2006, Drobeta Turnu-Severin

REZULTATE Clasa a IV-a

Numele şi prenumele Scoala Premiul Numele şi prenumele Scoala Premiul Ciobanu Diana Odobleja I Soare Daniela Şc 14 menţiune Medeleţ Miruna Carol Craiova II Ciută Cora Odobleja menţiune Osmani Amil Carol Craiova II Luţă Robert Carol Craiova menţiune Mitroi Laura Carol Craiova II Băloi Mihnea Şc Paulian menţiune Lupuleţi Alexandru Odobleja III Dragomir Andrei Odobleja menţiune Ţîru Vlad Carol Craiova III Duţă Anidora Şc 3 menţiune Călbureanu Diana Carol Craiova menţiune Mîrzoca Cezara Şc Paulian menţiune Guină Diana Alexandra Odobleja menţiune Răescu Adelina Larisa Şc 11 menţiune Ionaş Ioana Şc Paulian menţiune Spînu Antoanela P Sergescu menţiune Mema Şerban Odobleja menţiune Băloi Elisabeta Sc 2 menţiune Neagu Dragoş Odobleja menţiune Gheorghe Bogdan Şc 2 menţiune Teodorescu Alin Carol Craiova menţiune Gherghe Diana Şc 3 menţiune Caşotă Andreea Carol Craiova menţiune Mircea Andrei Sc 3 menţiune Mija Cătălina Carol Craiova menţiune Popescu Alin Şc 11 menţiune Petcu Oana Carol Craiova menţiune Beloineanu Andrei Carol Craiova menţiune Trancotă Diana Şc 6 menţiune Ciocan Andrei Şc 3 menţiune Piorescu Alexandru Carol Craiova menţiune Gegea Georgiana Odobleja menţiune Chicet Carina Şc 6 menţiune Popa Mihaela Odobleja menţiune Păuna Denisa Nicoleta Odobleja menţiune Tudor Georgiana Şc Paulian menţiune Puican Tiberiu Şc 2 menţiune Borugă Alin Şc 2 menţiune Cîrstov Gabriela Carol Craiova menţiune Marcu Bogdan Carol Craiova menţiune Marculescu Teodora Şc 6 menţiune Mendea Andreea Şc 3 menţiune Simion Claudia Odobleja menţiune Ene Bogdan Şc 6 menţiune Munteanu Floarea Odobleja menţiune Stanciu Vladimir Şc 6 menţiune Borcoş Ştefan Şc 3 menţiune Marghescu Luminiţa Şc 11 menţiune Mogoşanu Cosmin Şc Paulian menţiune Truşcă Alina Şc 6 menţiune Benga Andrei Şc 2 menţiune Ciotîrlă Cristian Şc Negreanu menţiune Ciuciu Laurenţiu Şc 2 menţiune Liliac Ramona Denisa Odobleja menţiune Bobiţi Ionuţ Adelin Odobleja menţiune Semen Alexandra Şc 2 menţiune Michiduţă Alexandru Carol Craiova menţiune Vladu Cătălin Şc Negreanu menţiune Voicu Robert Carol Craiova menţiune Bărbulescu Ramona Odobleja menţiune Geică Gabriela Carol Craiova menţiune Budecan Cosmin Şc 11 menţiune Oancea Mihaela Carol Craiova menţiune Munteanu Enia Maria Odobleja menţiune Olaru Alin Şc 6 menţiune

Clasa a V-a Numele şi prenumele Scoala Premiul Numele şi prenumele Scoala Premiul

Ştefan Andrei CNT I Raicu Maria Şc 11 menţiune Voicu Răzvan CNT II Badea Beatrice CNT menţiune Săceanu Andrei CNT III Pegulescu Vlad Daniel Vînju Mare menţiune Nicolae Andrei CNT menţiune Nicola Dorina Dl Tudor menţiune Nicola Silviu CNT menţiune Lacatusu Maria Sc.6 menţiune Chiriş Alexandra CNT menţiune Tănasie Denisa CNT menţiune Vîlcu Andrei CNT menţiune Anghel Cristian Vînju Mare menţiune Bărbulescu Cristian Odobleja menţiune Cazacu Cosmin CNT menţiune Băluţă Adrian CNT menţiune Mohora Eduard Odobleja menţiune Faer Eleonora CNT menţiune Oprea Alida CNT menţiune Gimoiu Ruxandra CNT menţiune Nuţă Flavius Andrei CNT menţiune Marghescu Andreea CNT menţiune Diaconu Paula Luiza Odobleja menţiune Aldea Loredana Motru menţiune Surdulescu Simona Şc 9 menţiune Viţian Sorina CNT menţiune Calafeteanu Liviu Şc 9 menţiune Filip Radu CNT menţiune Catanescu Raluca Sc.2 menţiune Georgescu Ana CNT menţiune Gîrbovan Paul CNT menţiune Nistor Ioana Andreea Odobleja menţiune Idita Marina Şc 9 menţiune

Page 15: RMM 6.pdf

Petre Sergescu

H

- 13 -

SSM Bălaşa Ciprian CNT menţiune Lăcraru Alexandru Odobleja menţiune Raicu Marian Motru menţiune Stîngă Adrian Şc 11 menţiune Turcu Izabela Lic Ţiţeica menţiune Tîrlui Valeria Lic Ţiţeica menţiune Aniţa Georgiana CNT menţiune Oancea Madalina Motru menţiune Asproniu Andreea CNT menţiune Tintos Adela Sc.6 menţiune Cîrstea Roxana CNT menţiune Truşcă Antonia Lidia Odobleja menţiune Pristoleanu Narcis CNT menţiune Zorilă Mihai CNT menţiune

Clasa a VI-a Numele şi prenumele Scoala Premiul Numele şi prenumele Scoala Premiul

Zugravu Rozalia CNT I Guran Maria Sc.2 menţiune Grosu Vlad CNT II Lupiţu Gabriela CNT menţiune Limbosu Maria CNT III Pănescu Dragoş CNT menţiune Nistor Mihaela Adriana Şc 14 III Ţintoş Mădălina CNT menţiune Cilioaica Rebeca CNT menţiune Arbănaşi Cornel Lic Ţiţeica menţiune Dăogaru Marian Motru menţiune Boescu Anca CNT menţiune Săndulescu Adela CNT menţiune Costea Alexandra CNT menţiune Băleanu Andrei Motru menţiune Drăguşin Cornel Şc 5 menţiune Drăghici Răzvan CNT menţiune Olaru Mihaela CNT menţiune Opriţa Geanina CNT menţiune Pîrvulescu Eleodor CNT menţiune Pavel Cristian CNT menţiune Popescu Denisa Şc 14 menţiune Toader Simona Sc.2 menţiune Rădulescu Silvia Şc 6 menţiune Ghinea Andrei Bogdan Şc 14 menţiune Sirbu Adrian Mihai Lic.Titeica menţiune Grecu Anda Teodora Lic Ţiţeica menţiune Tufaru Danusia Şc 6 menţiune Croitoru Andra Sc.2 menţiune Vişan Luana CNT menţiune Gavrilă Costinel Şc 11 menţiune Cojocaru Alex Sc 2 menţiune

Clasa a VII-a Numele şi prenumele Scoala Premiul Numele şi prenumele Scoala Premiul

Tiucu Andreea Carol Craiova I Nicolescu Adelina Carol Craiova menţiune Teşilă Bianca Lic Ţiţeica II Tiţă Andreea CNT menţiune Milici Alina Şc 5 III Papa Andreea CNT menţiune Mateiţă Diana Carol Craiova menţiune Agape Mihai CNT menţiune Sibinescu Ioana Motru menţiune Chereşdi Ion Lic Ţiţeica menţiune Furcuţă Ioana CNT menţiune Popescu Eduard CNT menţiune Papa Florin CNT menţiune Surugiu Mădălina Motru menţiune Sandu Loredana Carol Craiova menţiune Andreca Mihai CNT menţiune Stănişoară Sarah CNT menţiune Leonescu Larisa Şc 14 menţiune Budu Anca CNT menţiune Popa Bogdan Şc 14 menţiune Drăghia Miruna CNT menţiune Prunar Victor Loga Carans. menţiune Ţuculanu Andreea CNT menţiune Zanfir Cristian Doda Carans. menţiune Zorocliu Anda CNT menţiune

Clasa a VIII-a Numele şi prenumele Scoala Premiul Numele şi prenumele Scoala Premiul

Timofte Andrei Loga Carans. I Şeitan Mihaela CNT menţiune Carapencea C-tin CNT II Croitoru Răzvan CNT menţiune Duţă Adrian CNT III Milcu Roxana Loga Carans. menţiune Vlad Adina Loga Carans. menţiune Orbu Alexandra Lic Ţiţeica menţiune Pupăză Elena CNT menţiune Tudor Mihaela Motru menţiune Megan Ligia Loga Carans. menţiune Alex. Crivac Cristina Lic Ţiţeica menţiune Moatăr Alexandra Loga Carans. menţiune Duicu Vlad Motru menţiune Ploştinaru Diana Loga Carans. menţiune Ionaşcu Luana Motru menţiune

Clasa a IX-a Numele şi prenumele Scoala Premiul Numele şi prenumele Scoala Premiul

Trocan Irina CNT I Cruţan Bogdan CNT menţiune Pufu Cristian Carol Craiova II Drăgan Alin Oţelu Roşu menţiune Kremer Emanuela Loga Carans. III Eftinoiu Diana CNT menţiune Bobîlcă Oana CNT menţiune Staicu Alin Motru menţiune Oprea Radu CNT menţiune Gliga Anemona CNT menţiune Sibinescu Corina Motru menţiune Catană Nicu Motru menţiune Ciouca Eugen CNT menţiune Chilom Radu CNT menţiune

Page 16: RMM 6.pdf

Petre Sergescu

- 14 -

Lincan Dan CNT menţiune Nicolescu Alexandra CNT menţiune Ştefănoiu Anca CNT menţiune Robu Alex CNT menţiune Voicu Andreea CNT menţiune Roşu Maria CNT menţiune Gurgu Caius Loga Carans. menţiune Tuce Andreea Lic Ţiţeica menţiune Rogobete Roxana CNT menţiune Dunăreanu Lidia Lic Ţiţeica menţiune Sandu Andreea Lalescu Reşiţ menţiune Unguraş Dragoş Oţelu Roşu menţiune Mema Alexandra CNT menţiune Tişcă Mihaela Lalescu Reşiţ menţiune Mercioniu Marina Orşova menţiune Picu Vulpăşin Bianca CNT menţiune Răducu Alexandru CNT menţiune Săvoiu Aurel CNT menţiune Bechir Adriana CNT menţiune Mărinică Andreea Carol Craiova menţiune Cerga Alina CNT menţiune Vasilache Mădălina CNT menţiune Ciorobea Mihai CNT menţiune Ursachi Alexandru Vînju Mare menţiune Nica Flavius CNT menţiune Papală Doriana CNT menţiune Lungu Viorel Costin CNT menţiune Pasăre Andra Lalescu Reşiţ menţiune Zaharia Mihaela CNT menţiune Vlădoiu Alina CNT menţiune

Clasa a X-a Numele şi prenumele Scoala Premiul Numele şi prenumele Scoala Premiul

Mitucă Atena CNT I Popescu Anca Odobleja menţiune Tigora Andrei CNT I Puia Constantin Carol Craiova menţiune Răveanu Ioana CNT II Nistor Alexandru CNT menţiune Şuşelea Robert CNT II Jiplea Bogdan CNT menţiune Gogoloiu Gabriela Lic Ţiţeica III Ciocea Marina CNT menţiune Botea Silvia CNT III Belbu Loredana CNT menţiune Tuţă Leontin CNT menţiune Abagiu Aurel CNT menţiune Coandă Oana CNT menţiune Ghinea Constantin Dl Tudor menţiune Popovici Doru Lalescu Reşiţ menţiune Ghiga Octavian CNT menţiune Creţu Andrei CNT menţiune Mariescu Radu CNT menţiune Hinoveanu Cătălin CNT menţiune Păunescu Georgiana CNT menţiune Prundeanu Andreea CNT menţiune Hălălău Rodica CNT menţiune Căiniceanu Andrei CNT menţiune Pîrvănescu Izabela Lic Ţiţeica menţiune Dochin Luminiţa Doda Carans. menţiune Preşneanu Ruxandra CNT menţiune Rachieru Adrian CNT menţiune Vladu Margareta CNT menţiune Iacob Alexandra Doda Carans. menţiune Caragea Cătălin Motru menţiune

Clasa a XI-a Numele şi prenumele Scoala Premiul Numele şi prenumele Scoala Premiul

Rapcea Mihai CNT I Surdulescu Vlad CNT menţiune Bîzdoacă Mihai CNT II Busuioc Lucian CNT menţiune Bobiţi Ruxandra Lic Ţiţeica II Boară Andra CNT menţiune Dăgădiţă Monica CNT II Frumosu Sveta Orşova menţiune Roşu Ştefan CNT II Mîţoi Delia CNT menţiune Marcu George CNT menţiune Asproniu Gheorghe CNT menţiune Cucu Silviu Lalescu Reşiţ menţiune Dan Lorena CNT menţiune Pleşu Maria Strehaia menţiune Enache Bianca Doda Carans. menţiune Buna Mădălina Lalescu Reşiţ menţiune Glavici Iasmina CNT menţiune Năstase Liliana CNT menţiune Pascu Liviu CNT menţiune Turturea Roxana CNT menţiune Posniciuc Iulian CNT menţiune Aanicăi Simona Motru menţiune Săvoiu Lorena CNT menţiune Petria Andrada CNT menţiune Ungureanu Irina CNT menţiune Ghintaru Silviu Vînju Mare menţiune

Clasa a XII-a Numele şi prenumele Scoala Premiul Numele şi prenumele Scoala Premiul

Ungureanu Andrei CNT I Crăciunescu Mirela CNT menţiune Băzăvan Eduard Lic Ţiţeica II Jianu Amalia CNT menţiune Mirea Teodor CNT III Tîrţea Gabriela CNT menţiune Stoica George CNT menţiune Chilom George CNT menţiune Băcărin Dan CNT menţiune Nef Andrei CNT menţiune Paşov Iulia CNT menţiune Creţescu Oana CNT menţiune Buşoniu Andra Orşova menţiune Mareş Flavia CNT menţiune Chiş Andrei Lalescu Reşiţ menţiune Purdescu Monica CNT menţiune Ciocea Diana CNT menţiune Vîjaică Raluca CNT menţiune

Page 17: RMM 6.pdf

Petre Sergescu

H

- 15 -

SSM Simpozionul Naţional

“PETRE SERGESCU” REFERATE

I. O generalizare a problemei celor 12 bile

prof. Stoican Victor Grup Şcolar “C.Brâncoveanu”

Baia de Arama

Este cunoscută problema celor 12 bile: “ Se dau 12 bile, dintre care 11 au aceiaţi greutate iar a 12-a este de greutate diferită (nu se ştie dacă este mai grea sau mai uşoară faţă de celelalte). Din trei măsurări cu balanţa să se afle care este bila defectă si cum este faţă de celelalte”. Problema generală pe care ne-o propunem, este următoarea: “ Avănd o balanţă şi n măsurări, care este numărul maxim de bile m=f(n) dintre care m-1 au aceiaşi greutate iar a m-a bilă diferită de celelalte, astfel ca prin n măsurări să putem afla care este bila defectă”. Introducem funcţiile f:N→N, g:N→N şi h:N→N unde: f(n) – numărul maxim de bile pentru care din n încercări putem afla bila defectă, g(n) – numărul maxim de bile pentru care din n încercări să se poată afla bila defectă considerând că înaintea măsurărilor ni se mai dă o bilă normală. Introducem soluţia x>y (sau x<y) care înseamnă că am pus x bile cu potenţiale defecte pe primul taler şi y pe al doilea şi completăm volumul cu bile mai puţine astfel ca să fie acelaşi număr de bile pe ambele talere şi talerul cu cele y bile se va ridica( fiind mai uşor). h(n) – numărul maxim m = x + y astfel încât având la îndemână o relaţie de tipul x>y avem o metodă sigură de a afle care este bila defectă din cele x + y bile. Vrem să aflăm deci, care este numărul maxim de bile m = f(n) din care putem găsii sigur bila defectă având voie să facem n cântăriri. La prima căntărire punem A bile pe fiecare taler, B fiind numărul bilelor încă nefolosite, şi ne mai rămân n-1 cântăriri. f(n) = m = 2A + B Presupunem h(n) = x + y unde x<y. Fie x = x1+x2+x3 şi y = y1+y2+y3. Deci, x<y ⇒ x1+x2+x3 < y1+y2+y3. Fie

x = ⎪⎩

⎪⎨

−−−

cântăântăprimalanefolositebilelextalerdoileaalpetrecemlecarepebilelex

talerprimulperamancarebilelex

3

2

1

y = ⎪⎩

⎪⎨

−−−

nefolositebileleytalerprimulpetrecemlecarepebileley

talerdoileaalperăămâcarebileley

3

2

1

La prima cântărire vom pune pe primul taler x1 + y2 iar pe al doilea x2 + y1 bile. Avem trei posibilităţi: a) x1 + y2 < x2 + y1

x1 + x2 + x3 < y1 + y2 + y3 Evident, bila defectă nu poate fi printre cele x3 şi y3 dar nici printre x2 şi y2 deoarece dacă ar fi de exemplu printre cele x2, atunci din x1+y2<x2+y1 ar rezulta că este bila grea iar din relaţia x1+x2+x3<y1+y2+y3 că este bila uşoară. Deci, bila defectă poate fi ori în x1 şi este uşoară ori în y1 şi este bila grea, deci x1 < y1 (1) b) x1 + y2 > x2 + y1

x1+x2+x3< y1+y2+y3 de unde analog rezultă x2<y2 (2)

Page 18: RMM 6.pdf

Petre Sergescu

- 16 -

c) x1 + y2 = x2 + y1 x1+x2+x3 < y1+y2+y3 de unde rezultă x3< y3 (3) Contopind cazurile a), b) şi c) de mai sus, ajungem la concluzia că de la o relaţie de tipul x<y printr-o măsurare ajungem la una din relaţiile (1), (2), (3). h(n) = x + y , dar din n-1 cântăriri trebuie să găsim bila defectă în oricare din cele trei cazuri. Deci, putem considera ca x+y să fie h(n), adică maximul astfel încât din x<y să putem afla bila defectă din n cântăriri. Deci, x1+y1, x2+y2, x3+y3 trebuie să fie maximele astfel încât din cazurile x1<y1, x2<y2, x3<y3 să putem găsii sigur bila defectă din n-1 cântăriri. Deci, x1+y1 = x2+y2 = x3+y3 = h(n-1). Atunci, h(n) = x+y = x1 + x2 + x3 + y1 + y2 + y3 = 3h(n-1) Din h(1) = 3 şi h(n) = 3h(n-1) se verifică prin inducţie că h(n) = 3n. Cum 2A = A+A≤h(n-1) = 3n-1 şi cum 2A trebuie să fie numărul par maxim mai mic decât 3n-1, rezultă că 2A = 3n-1-1. Deci f(n) = 2A+B = 3n-1-1 + g(n-1). Pentru a afla o relaţie pentru g(n), procedăm analog cu deosebirea că trebuie să ţinem cont de faptul că trebuie să avem în plus o bilă normală. Deci g(n) = 2A + 1 + g(n-1) = 3n-1 + g(n-1). Din g(1) = 1 şi g(n) = 3n-1 + g(n-1) avem:

g(1) = 1 g(2) = 3 + g(1) g(3) = 32 + g(2) …………………………. g(n-1) = 3n-2 + g(n-2) g(n) = 3n-1 + g(n-1), adunând egalităţile, obţinem:

g(n) = 1+3 +32 + 33 + …= 3n-1 = 2

13 −n

. Deci, g(n) = 2

13 −n

şi f(n) =2

33 −n

.

În particular, “Problema celor 12 bile” având f(3) = 12, poate fi rezolvată prin 3 cântăriri.

Din 12 = F(3) = 2A + B avem 2A = 3n-1 – 1 ⇒ A = 4, B = g(2) = 2

132 − = 4

Deci, la prima cântărire se vor pune câte 4 bile numerotate a1, a2, a3, a4, şi respectiv b1, b2, b3b b4 pe cele două talere. Bilele normale le notăm cu P. Avem cazurile: 1. Dacă a1+a2+a3+a4 = b1+b2+b3+b4 atunci bila defectă nu poate fi decât în cele patru bile rămase, c1, c2, c3, c4. Punem pe talere pe c1+c2 şi respectiv pe c3+P şi putem avea cazurile: 1a. c1+c2 = c3+P şi rezultă că c4 este bila defectă pe care o comparăm prin ultima cântărire ( a treia) cu o bilă normală. 1b. c1+c2 > c3+P rezultă că la ultima cântărire vom pune pe talere bilele c1 şi c2. 2. a1 + a2 + a3 + a4 > b1 + b2 + b3 + b4

Conform cazului general, împărţim cele 8 bile în 3 grupe în mod cât mai echilibrate; 3,3,2. La a doua cântăriei punem pe cele două talere a1+a2+b3 şi respectiv b1+a3+P. Putem avea cazurile:

2a. a1+a2+b3 = b1+a3+P ⇒ a1, a2, a3, b1, b3 sunt normale şi a4+P<b2+b4. La ultima cântărire punem pe talere pe b2 şi b4.

2b. a1+a2+b3>b1+a3+P, a1+a2+a3+a4>b1+b2+b3+b4 (*) Presupunem b3 bilă defectă ⇒b3 este bila grea şi din (*) rezultă că b3 este bila grea , deci b3

=P(bilă bună). Analog se arată că a3 =P şi relaţiile 2b devin: a1+a2+P=b1+P+P, a1+a2>b1+P

Deci, la ultima cântărire punem pe talere pe a1 şi a2. 2c. a1+a2+b3<b1+a3+P, a1+a2+a3+a4>b1+b2+b3+b4 Analog cazurilor precedente rezultă că a1=P, a2=P, b1=P şi din P+P+b3<P+a3+P tezultă că

b3<a3. La ultima cântărire vom pune pe talerele balanţei pe b3 şi o bilă normală P.

Page 19: RMM 6.pdf

Petre Sergescu

H

- 17 -

SSM II. Elemente de probabilităţi geometrice

prof Popescu Octavian Tudor

Colegiul Economic F.S.Nitti Timisoara

Introducere Reamintim definiţia clasică a probabilităţii: Se consideră o urnă U care conţine n bile, dintre care m albe şi n-m negre Se extrage la întâmplare o bilă. Fie evenimentul “bila extrasă este albă.” Definiţie: Se numeşte probabilitatea evenimentului A raportul dintre numărul cazurilor favorabile realizării evenimentului A şi numărul cazurilor egal posibile.

Deci: ( )nmAP = .

Această definiţie se poate folosi numai în experimentele cu evenimente elementare egal posibile. (cu probabilităţi egale). Plecând de la definiţia clasică a probabilităţii putem rezolva ( intuitiv deocamdată) probleme apelând la probabilităţile geometrice. Spre exemplu: La o ţintă circulară de rază R se aruncă cu o săgeată, la întâmplare. Să se determine:

a) probabilitatea ca săgeata să se înfigă în interiorul unui pătrat înscris în cercul de rază R . b) Probabilitatea ca săgeata să se înfigă întrun triunghi echilateral înscris în cerc. Se ştie că

toate săgeţile ating ţinta şi poziţiile punctelor de înfigere a săgeţilor pe ţintă sunt egal posibile.

a) Avem evenimentul A : “săgeata se înfige în pătratul înscris în cercul de rază R ”

( ) =APππ22

cerc. Ariapăătra

cercului punctele toatepăătratulu punctele toate

favorabile caz posibile caz nr.

2

2

====RRAria

n

b) A: „săgeata se înfige în triungiul înscris în cercul de rază R .”

( )ππ 4332

33

cerc. Aria triunghi

2

2

===R

RAriaAP .

Rezolvarea acestei probleme a plecat de la principiul definiţiei clasice a probabilităţii. Vom furniza în cele ce urmează suportul matematic riguros care ne îndrumă la rezolvarea

acestor tipuri de probleme.

Probabilităţi geometrice Orice familie de figuri geometrice F din spaţiul euclidian rΕ este dată de ecuaţii de forma: (1) ( ) 0,...,;,..., 11 =qrxxF αα , lrj −≤≤1 , 11 −≤≤ rl unde qαα ,...,1 sunt parametri. Dacă dăm valori concrete parametrilor qαα ,...,1 atunci obţinem o figură din respectiva familie. Deci există o corespondernţă biunivocă între familia de figuri F şi o mulţime D0 ⊂ qB a spaţiului qΕ cu elemente de forma ( )qαα ,...,1 . Cu alte cuvinte o discuţie asupra familiei de figuri F din Spaţiul Euclidian, se reduce la o discuţie asupra mulţimii D 0 a parametrilor ( )qαα ,...,1 .

Iar dacă F1 este o submulţime a lui F atunci lui F1 îi corespunde în qΕ un domeniu D⊂D 0 . Unei familii de figuri F i se poate asocia un grup de invarianţă şi anume un grup de

transformări care transformă orice figură din F într-o altă figură din F (în cazul în care acest grup există).

Ex: Grupul de invarianţă al familiei dreptelor din plan este grupul transformărilor ortogonale.

Page 20: RMM 6.pdf

Petre Sergescu

- 18 -

În spaţiul rΕ o familie de transformări este dată de funcţii de forma: rra Ε→ΕΤ : ; ( )saaa ,...,1= cu legea funcţiei dată de

(2) ( )srii aaxxff ,...,;,..., 11= ri ≤≤1 unde ( )saaa ,...,1= sunt parametrii transformării.

Deci ( ){ }ssrrar aaaG Ε∈=Ε→ΕΤ= ,...,: 1

rG este grup dacă ( )cbassbas îacba Τ=ΤΤΕ∈∃Η∈ΤΤ⇒Ε∈ oo .., Dacă punem condiţia ca rG să fie grup invariant al familiei F, acest lucru înseamnă

(conform definiţiei grupului invariant şi ţinând cont de (1) şi (2)) ( ) ( )21111 ,...,;,...,,...,;,..., ββαα rjqrj yyFxxF = ; lrj −≤≤1 11 −≤≤ rl

unde ( )sqii aa ,...,;,..., 11 ααϕβ = . ( 1β sunt găsiţi în funcţie de parametrii familiei qαα ,...,1 şi parametrii transformării saa ,...,1 ) Cu alte cuvinte unui grup invariant de transformări din rΕ corespunzători unei familii de figuri F îi corespunde biuivoc un alt grup de transformări ale paramtrilor în spaţiul qΕ . Acest nou grup o să-l notăm sΗ deoarece depinde de s parametrii ( ) ssaa Ε∈,...,1 . Adică ( ){ }ssqqas aaa Ε∈=Ε→Ε=Η ,...,: 1ϕ sΗ este grupul transformărilor care transformă parametrii unei figuri din F în parametrii altei figuri din F. Urmărind ca la o deplasare ale figurilor din familia F rezultatul calculelor să nu se schimbe, putem să dăm următoarea definiţie: Definiţie: O integrală ( ) qq

Ddd αααα ...,...,... 11Φ∫∫=Ι se numeşte invariant integral al

grupului de transformări sΗ dacă ( ) ( ) qq

Dqq

Ddddd ββββαααα ...,...,......,...,... 11

'11 Φ∫∫=Φ∫∫

unde qBD ⊂ este domeniul parametrilor ( )qαα ,...,1 corespunzător unei subfamilii F’,iar D’=domeniul parametrilor ( )qββ ,...,1 obţinut prin aplicarea transformărilor (2). (deplasarea lui D) Se poate demonstra că o condiţie necesară şi suficientă ca integrala I să fie invariant integral grupului sΗ este ca

(3) ( ) 02

1

=Φ∂∂∑

=kkt

kxξ , st ≤≤1

unde ktξ sunt coeficienţii transformărilor infinitezimale ale grupului sΗ . Acest sistem de ecuaţii cu derivate parţiale se numeşte sistemul lui Deitheil. În continuare ne va interesa situaţia când grupurile de transformări admit un invariant integral unic, adică atunci când sistemul lui Deitheil admite o soluţie unică Φ , abstracţie făcând de un factor constant. Putem presupune că Φ (soluţia sistemului lui Deitheil) este nenegativă, căci dacă Φ furnizează un invariant integral, atunci şi Φ furnizează un invariant integral.

Considerăm qB⊂0D şi ( ) +→∩ RD qBP 0:ν ( )Dν = ( ) qqD

dd αααα ...,...,... 11Φ∫∫

ν este o măsură pozitivă. Pentru că între submulţimi de figuri ale lui F şi submulţimile domeniului 0D există o corespondenţă biunivocă, putem defini măsura unei submulţimi 1F a familiei F ( )1Fµ ca fiind ( )Dν iar ( ) ( )0Dνµ =F

Page 21: RMM 6.pdf

Petre Sergescu

H

- 19 -

SSM Făcând o schimbare de variabile ( )qkk αααα ,...,1= qk ≤≤1 se observă că

( ) ( ) ( )( ) q

q

qq

Dqq

Ddd

D

Ddd αα

αα

αααααααα ...

,...,

,...,,...,......,...,... 1

1

11

'11 Φ∫∫=Φ∫∫

adică ( ) ( ) ( )1Fµνν == DD ⇒ măsura unei mulţimi de figuri 1F ale unei familii F nu depinde de invariantul integral cu care se construieşte măsura respectivă. (la o deplasare a figurii rezultatul calculelor nu se schimbă). Dacă ( ) 1,...,1 =Φ qαα atunci ( ) =∫∫= q

DddD ααν ...... 1 volumul domeniului D .

Dacă 1F ⊂ F este o submulţime de figuri ale familiei F căreia în spaţiul parametrilor îi corespunde domeniul D , atunci probabilitatea ca un element luat la întâmplare din F să fie din 1F este

(4) ( ) ( )( )

( )( )

( )( ) qq

D

qqD

dd

dd

DDPP

αααα

αααα

νν

µµ

...,...,...

...,...,...

11

11

00

Φ∫∫

Φ∫∫====

FFF 1

1

Prin această construcţie se obţine σ – câmpul de probabilitate ( ,0D ,0 qB∩D )P . Cu alte cuvinte pentru rezolvarea unei probleme de probabilităţi geometrice referitoare la o familie de figuri date, se determină invariantul integral corespunzător grupului de transformări considerat, apoi se determină submulţimea din spaţiul parametrilor ale cărei puncte generează submulţimea de figuri a mulţimii de figuri date şi apoi se calculează probabilitatea cerută cu formula (4).

Aplicaţie Sisteme de puncte din plan. Fie în planul euclidian de coordonate carteziene ortogonale (x, y) familia de figuri formată din mulţimea punctelor ( )21,ααM ; deci ecuaţiile familiei sunt: (5) 1α=x , 2α=y cu 1α , 2α parametri. Grupul de invarianţă al acestei familii este grupul transformărilor din plan de ecuaţii: 'xx = cos 'y−θ sin a+θ 'xy = sin 'y+θ cos b+θ Aplicând acest grup familiei ecuaţiile ( ) devin: 1' β=x , 2' β=y unde: (6) 11 βα = cos 2βθ − sin a+θ 12 βα = sin 2βθ + cos b+θ Acesta e grupul ),( 212 ααH ataşat grupului de invarianţă faţă de familia (5 ). Ţinând seama de dezvoltările în serie:

sin ...;!5!3

53 θθθθ +−= cos ...

!3!21

32 θθθ +−=

relaţiile (6) devin: ...0211 ++−= aββα ...0122 +++= bββα coeficienţii transformărilor infinitezimale ale acestui grup sunt: ,111 =ξ 012 =ξ , ,213 βξ −= ,021 =ξ ,122 =ξ 123 βξ = Sistemul lui Detheil ( ) se scrie:

Page 22: RMM 6.pdf

Petre Sergescu

- 20 -

⎪⎪⎪

⎪⎪⎪

=∂Φ∂

+∂Φ∂

=∂Φ∂

=∂Φ∂

02

0

0

212

2

1

ββ

ββ

β

β

şi are soluţia =Φ const. Având în vedere (4) putem lua .1=Φ Deci măsura mulţimii F de puncte din planul xOy ai cărei parametrii variază într-o

mulţimee mărginită 20 β⊂D este ( ) 210

ββµ ddD∫∫=F sau, dacă ţinem seama de (5) ( ) ( )00

DSdxdyD =∫∫=Fµ unde ( )0DS reprezintă aria domeniului 0D din plan.

Exemple:

1. Pe segmentul OA de lungime l situat pe axa Ox se alege la întâmplare punctul B. Să se determine

probabilitatea ca cel mai mic dintre segmentele OB şi BA să aibă lungimea mai mare ca 3l .

Fie evenimentul A: cel mai mic dintre segmentele OB şi OA are o lungime mai mare ca 3l .

notăm cu { }OAOBx ,min=

atunci 2lx ≤ reprezintă domeniul ⎥⎦

⎤⎜⎝⎛=

2,00

lD şi 3lx > ceea ce înseamnă ⎥⎦

⎤⎜⎝⎛=

2,

3llD

deci ( ) ( )( ) 3

1

2

320

=−

== l

ll

DvDvAP

2. Pe un segment AB de lungime l sunt fixate la întâmplare punctele C şi D. Care este probabilitatea ca punctul C să fie situat mai aproape de D decât de A? Poziţiile punctelor C, D sunt egal posibile.

Fie punctele C, D în interiorul segmentului AB, unde A(0,0), B(l,0) cu 0≥l , C(x,0), D(y,0). Considerăm evenimentul E: punctul C să fie situat mai aproape de D decât de A

avem [ ]lx ,0∈ , [ ]ly ,0∈ . deci [ ] [ ]llD ,0,00 ×= iar x şi y mai trebuie sa satisfacă condiţia yxx −>

adică ( ) [ ] [ ]{ }yxxllyxD −>×∈= :,0,0,

deci ( ) ( )( ) 4

32

22

0

=⎟⎠⎞

⎜⎝⎛ +

===l

lll

uluiAriaPatratuluiAriaTrapez

DvDvEP

Page 23: RMM 6.pdf

Petre Sergescu

H

- 21 -

SSM 3. Un indicator de semnale se compune din două dispozitive. Fiecare din cele două semnale este egal posibil în orice interval de timp T. Indicatorul acţionează dacă diferenţa între momentele de sosire ale semnalelor este mai mică ca ( )Ttt < . Să se determine probabilitatea ca indicatorul să acţioneze în timpul T, dacă oricare din cele două dispozitive trimite un semnal.

Considerăm evenimentul E: indicatorul acţionează în timpul T

Notăm cu x momentul sosirii semnalului de la dispozitivul nr.1 şi cu y momentul sosirii semnalului de la dispozitivul nr.2. atunci ( )Tx ,0∈ şi ( )Ty ,0∈ adică [ ] [ ]TTD ,0,00 ×= pentru producerea evenimentului E se impune condiţia tyx <−

adică ( ) [ ] [ ]{ }tyxTTyxD <−×∈= :,0,0,

( ) ( )( )

( )( )

22

22

0

222

TtTt

T

tTT

uluiAriaPatratsuratateruluiHaAriaPatrul

DvDvEP −

=

−−

===

4. Se aleg la întâmplare două numere pozitive x şi y mai mici sau egale cu 2. să se determine

probabilitatea ca produsul lor să nu depăşească 1, iar xy să nu depăşească pe 2.

E: produsul numerelor xy nu depăşeşte 1, iar xy nu depăşeşte 2.

avem [ ]2,0∈x , [ ]2,0∈y rezultă că [ ] [ ]2,02,00 ×=D

iar ( ) [ ] [ ]⎭⎬⎫

⎩⎨⎧

≤≤×∈= 2,1:2,02,0,yxxyyxD

deci ( ) ( )( ) 8

2ln31

0

+===

uluiAriaPatrattaAriaHasura

DvDvEP

5. Să se determine probabilitatea ca lungimea unei coarde într-un cerc de rază R să fie cuprinsă între două numere a şi b. Considerăm într-un cerc de rază R, o coardă aleasă arbitrar, de lungime l. atunci [ ]Rl 2,0∈ Vom restrânge domeniul însă, la un semicerc determinat în felul de mai jos, tocmai pentru a păstra invariantul egal cu 1. În semicercul de diametru paralel cu coarda considerată, există doua coarde paralele deasemenea cu diametrul şi coarda considerată, ce au lungimile a respectiv b.Mai considerăm şi raza cercului ce trece perpendicular pe coardele considerate şi care le intersectează în punctele M şi N.

[ ]RD ,00 = iar NsegmentulMD =

Deci ( ) ( )⎟⎟⎠

⎞⎜⎜⎝

⎛−−−==≤≤

441 2

22

2 bRaRRR

iMNsegmentuluvblaP

Page 24: RMM 6.pdf

Petre Sergescu

- 22 -

III. Asupra unor identităţi trigonometrice condiţionate

Lucian Dragomir Grup Şcolar Industrial

Oţelu-Roşu, Caraş-Severin Propunem în nota de faţă obţinerea unor cunoscute identităţi trigonometrice pe cale nonstandard , folosind proprietăţi geometrice ale triunghiului . Este vorba despre :

( 1 ) 2

cos2

cos2

cos4sinsinsin CBACBA ⋅⋅⋅=++ ;

( 2 ) CBACBA sinsinsin42sin2sin2sin ⋅⋅⋅=++ ; ( 3 ) tgCtgBtgAtgCtgBtgA ⋅⋅=++ , egalităţi adevărate în orice triunghi ABC (deci π=++ CBA ). O bună cunoaştere a formulelor trigonometrice şi deprinderea de a le aplica conduce la obţinerea acestor identităţi pe cale “standard“, aşa cum sunt prezentate, de exemplu, în [1], pag. 205 sau în [3], pag. 196 şi 228 .

În cele ce urmează vom folosi binecunoscuta proprietate geometrică : Dacă M este un punct în interiorul unui triunghi ABC , iar x , y , z sunt distanţele de la M la

laturile acestuia ( care au lungimile a , b , respectiv c ) , atunci ( * ) Sczbyax ⋅=++ 2 , unde S este aria triunghiului . Particularizând poziţia punctului M în relaţia precedentă vom obţine , pe rând , identităţile anunţate ( i ) Dacă M = I (adică M este centrul cercului înscris), folosind notaţiile uzuale, avem evident :

2sin

2sin

2sin4 CBARrzyx ⋅⋅⋅====

Egalitatea ( * ) devine succesiv , folosind şi teorema sinusurilor : CBARrcba sinsinsin22)( 2 ⋅⋅⋅⋅=⋅++ ⇔

2sin

2sin

2sin)sinsin(sin8 2 CBACBAR ++ =

2cos

2cos

2cos

2sin

2sin

2sin32 2 CBACBAR

Doar o simplificare conduce acum la identitatea ( 1 ) . □ ( ii ) Dacă M = O ( centrul cercului circumscris ) , avem :

CRzBRyARx cos,cos,cos === Egalitatea ( * ) conduce astfel la :

( ) CBARCcBbAaR sinsinsin4coscoscos 2 ⋅=⋅+⋅+⋅ , de unde , cu teorema sinusurilor , obţinem imediat identitatea ( 2 ) . □ ( iii ) Dacă M = H (ortocentrul triunghiului ABC) şi D, E, F sunt picioarele înălţimilor din A, B, respectiv C, avem :

CBRDHx coscos2 ⋅⋅== , ACREHy coscos2 ⋅⋅== , BARFHz coscos2 ⋅⋅== .

Aceeaşi relaţie ( * ) conduce la : CBABACACBCBA sinsinsincoscossincoscossincoscossin ⋅⋅=⋅⋅+⋅⋅+⋅⋅ De unde , prin împărţire cu 0coscoscos ≠⋅⋅ CBA ajungem la tgCtgBtgAtgCtgBtgA ⋅⋅=++ , adică ( 3 ) . □

Bibliografie: [1] Becheanu Mircea , Enescu Bogdan –Manual pentru clasa a X a , Ed. Teora , 1999 [2] Lalescu Traian – Geometria triunghiului , Ed.Tineretului , 1958 [3] Panaitopol Laurenţiu , Bălună Mihai , Enescu Bogdan – Manual pentru clasa a X a , Ed. Gil , 2000 [4] Vodă Viorel Gh. – Vraja geometriei demodate , Ed. Albatros , 1983

Page 25: RMM 6.pdf

Petre Sergescu

H

- 23 -

SSM IV. Generator de probleme

drd. Laviniu Bejenaru

C.N. “TRAIAN”

Deschizând mai multe numere din revista dedicată elevilor (serioşi şi capabili) ”Gazeta de matematică”, prescurtată în această prezentare prin GM, se pot distinge o serie de probleme comune. Deoarece generaţiile de elevi se schimbă, iar ei trebuie să înveţe cam aceleaşi noţiuni, este cumva normal să se cam repete o serie de probleme scrise, desigur, sub alte forme. Atât pentru liceu cât şi pentru şcoala generală.

Mă voi opri asupra unor probleme pentru gimnaziu cât şi asupra unor aspecte legate de categoria de probleme discutată, legate de raţionament. O categorie lucrativă pentru mulţi elevi este tipul problemelor în care se dă un număr scris sub forma unei expresii barate formată cu cifre zecimale gen abcd , unde a, b, c, d aparţin bazei zece, desigur, deci sunt cifre între 0 şi 9, cu a ≠ 0. Dându-se anumite cerinţe sau restricţii asupra cifrelor, se cere să se determine care sunt aceste numere sau câte sunt cele care verifică proprietăţile impuse. Desigur, numărul de 4 cifre nu este semnificativ, putând fi mai multe sau mai puţine; însă sub aspectul calculului matematic adresat elevilor nu prea multe.

Pot interveni şi probleme în care apar şi alte elemente, cum ar fi puterile. În acest caz, nu mai apare restricţia ca prima dintre cifre să fie nenulă, în lipsa altor precizări. Una din aceste probleme este problema E:12649 din GM Anul CVIII, nr.12/2003. Enunţul acestei probleme este următorul:

E:12649. Determinaţi a, b, c, d ∈ N, ştiind că .375432 =+++ dcba Rezolvarea normală pentru un elev de gimnaziu este dată de următoarea idée de bază: se

observă că toate elementele din suma stângă sunt pozitive, se determină valorile puterilor a, b, c, d maxime dincolo de care suma din stanga ar depăşi valoarea din dreapta şi se trece la o serie de vcerificări pentru egalitate. Valorile maxime sunt: a maxim este 5, b maxim este 3, c maxim este 2, d maxim este 2. S-a avut în vedere şi faptul că celelalte elemente din sumă au cel puţin valoarea 1. Soluţia de verificare, computerizată, este prezentată în fişierul E _12649.cpp de mai jos. Rezolvând problema direct şi/sau computerizat, obţinem soluţiile (0,3,1,1); (1,2,0,2); (2,3,0,1); (3,1,0,2); (3,3,0,0); (5,1,0,0). Această problemă se poate adapta, generând noi probleme. Propun următoarea problemă:

(***) Determinaţi a, b, c, d, e ∈ N, ştiind că 4375432 =++++ edcba Soluţiile acestei probleme obţinute foarte uşor sunt în număr de 7 şi anume: (0,2,0,2,1); (2,1,1,2,1); (2,2,1,2,0); (2,3,1,0,1); (4,1,2,0,1); (4,2,2,0,0); (5,0,1,1,0) . O altă problemă generalizabilă este E:13031 din GM Anul CX, nr.9/2005, număr aniversar,

110 ani de Gazeta Matematică. La mulţi ani! Enunţul ei este E:13031. Determinaţi elementele mulţimii ( ) ( ) ( ){ }128| =−⋅−⋅− xzzyyxxyz .

Prezentată spre rezolvare elevilor de gimnaziu, aceştia trebuie să verifice relaţia folosind anumite abilităţi, cum ar fi observaţia că numărul din dreapta este par, deci produsul de cifre din stânga trebuie să fie par, sau mai mult, că este o putere a lui 2. Aceste observaţii mai reduc numărul de cifre x, y, z care trebuie verificate în relaţia de mai sus. Se obţin soluţiile: (1,5,9); (4,8,0); (5,9,1); (8,0,4); (9,1,5). Corectitudinea poate fi verificată imediat folosind calculatorul printr-un program corespunzător, prezentat în fişierul E_13031.cpp. Putem genera foarte uşor multe alte probleme cu soluţii cu tot fără efort, computerizat. Voi propune o nouă problemă:

(***) Determinaţi numărul de soluţii şi soluţiile mulţimii ( ) ( ) ( ){ }1024)(| =−−⋅−⋅− xttzzyyxxyzt

Page 26: RMM 6.pdf

Petre Sergescu

- 24 -

Utilitatea verificării este importantă, mai ales că soluţiile corecte se obţin instantaneu. Acestea sunt în număr de 13 şi anume: (1,5,1,9); (1,9,1,5); (1,9,5,9); (4,0,8,0); (4,8,0,8); (5,1,9,1); (5,9,1,9); (8,0,4,0); (8,0,8,4); (8,4,8,0); (9,1,5,1); (9,1,9,5); (9,5,9,1).

Utilitatea considerării unor astfel de probleme este atât la nivelul profesorilor cât şi la nivelul elevilor. La nivelul profesorilor, obţinerea de probleme variate, orientate pe o anumită temă, este o necesitate, cu atât mai mult cu cât obţinerea soluţiilor corecte este imediată. Din punctul de vedere al elevilor, variaţia enunţurilor în anumite limite sau exactitatea soluţiilor nu sunt elemente care îi lasă indiferenţi pe cei care doresc să se perfecţioneze.

Concluzia este că se poate verifica corectitudinea soluţiilor multor probleme şi se pot genera o serie de probleme noi, în tiparele existente sau în tipare noi, destul de uşor de obţinut, la nivel gimnazial. Şi pentru liceu, aceste aspecte se pot discuta. Nu în ultimul rând, aş putea prezenta şi alte probleme care pot fi generalizabile şi pot avea soluţii calculate computerizat, în ideea că atât elevii cât şi profesorii îşi pot uşura activitatea şi pot varia conţinutul în anumite limite.

E:12869. Să se afle toate numerele de forma abcd , ştiind că 79

=−

+

cdabcdab .

E:12997. Aflaţi numerele de forma abc pentru care are loc egalitatea: babcabc ⋅⋅= 2 .

E_12649.cpp

Soluţii E_12649

E_13031.cpp

Soluţii E_13031

Soluţii E_12869

Soluţii E_12997

#include<iostream.h> #include<conio.h>

long a,b,c,d,nrsol,p1,p2,p3,p4; long Putere(long m,long n) //returneaza m^n { long p,k; if(n==0) return 1; else { p=1; for(k=1;k<=n;k++) p=p*m; return p; } } void main(void) { for(a=0;a<=5;a++) for(b=0;b<=3;b++) for(c=0;c<=2;c++) for(d=0;d<=2;d++) { p1=Putere(2,a); p2=Putere(3,b); p3=Putere(4,c); p4=Putere(5,d); if((p1+p2+p3+p4)==37) { cout<<”Solutia ”<<++nrsol<<” : ” ; cout<<”(”; cout<<a<<","<<b<<","<<c<<","<<d; cout<<”)”; cout<<endl; } } getch(); } (0,3,1,1); (1,2,0,2); (2,3,0,1); (3,1,0,2); (3,3,0,0); (5,1,0,0)

#include<iostream.h> #include<conio.h> long x,y,z,nrsol,ValExp; void main(void) { for(x=0;x<=9;x++) for(y=0;y<=9;y++) for(z=0;z<=9;z++) { ValExp=(x-y)*(y-z)*(z-x); if( (ValExp == 128) && (x != 0) ) { cout<<"Solutia "<<++nrsol<<" : "; cout<<"(" ; cout<<x<<","<<y<<"," <<z; cout<<")"; cout<<endl; } } getch(); } (1,5,9); (4,8,0); (5,9,1); (8,0,4); (9,1,5) (1,6,0,2); (2,4,0,3); (3,2,0,4); (4,0,0,5);: (4,8,0,6); (5,6,0,7); (6,4,0,8); (7,2,0,9); (8,0,1,0); (8,8,1,1); (9,6,1,2) (3,1,2)

Page 27: RMM 6.pdf

Petre Sergescu

H

- 25 -

SSM V. PRIMUL CONCURS JUDETEAN DE GEOMETRIE

GHEORGHE TITEICA

Profesor pensionar STEFAN MARICA Cu prilejul implinirii a 100 de ani de la nasterea marelui matematician severinean Gheorghe Titeica (nascut la 4 octombrie 1873) ,Inspectoratul Scolar Mehedinti ,Filiala Mehedinti a Societatii de Stiinte Matematice din Romania si catedra de matematica a Liceului Traian ,au initiat in 1973 Concursul de geometrie Gheorghe Ţiteica pentru elevii claselor VII-XII. Din comisia de organizare a concursului au facut parte urmatoarele cadre didactice: Profesor Mircea Chivu-Inspector General Adjunct Profesor Dorin Radoslav-Inspector de matematica Profesor Stefan Marica-Responsabil al concursului pentru clasele VII-VIII Profesor Paunescu Alexandru-Responsabil al concursului pentru clasele IX-XII. Datele concursului Pentru clasele VII-VIII a doua duminica a lunii octombrie la Scoala Generala Nr.1, unde a fost elev Gheorghe Titeica Pentru clasele IX-XII a treia duminica a lunii octombrie la Liceul Traian. Cateva din prevederile regulamentului acestui concurs: 1) Pot participa la concurs elevii scolilor din judetul Mehedinti care au aparut ca rezolvitori sau propunatori de probleme in revistele de specialitate, cu deosebire in Gazeta Matematica. 2) In scopul antrenarii elevilor pentru studierea si aprofundarea cunostiintelor matematice, elevii vor expedia comisiei de organizare si desfasurare a concursului cel putin un exercitiu sau problema originala. 3) Deoarece concursul are loc la inceputul anului scolar, programa de concurs este cea studiata in anii scolari precedenti. 4) Fiecare concurent va avea de rezolvat 3 probleme si anume: o problema din Gazeta Matematica publicata in ultimii doi ani, o problema sau exercitiu propus de un profesor de matematica din Mehedinti, o problema din culegerea Gheorghe Titeica, sau din manualele scolare. 5) Lucrarile scrise vor fi corectate de cate doi profesori, iar cele mai bune vor fi revizuite de o alta comisie (aceasta in scopul unei evaluari corecte si pe acesta baza stabilirea castigatorilor) Experienta concursului a determinat comisia de organizare si desfasurare sa modifice unele cerinte ale regulamentului. Deoarece problemele propuse de concurenti nu erau dintre cele mai reusite s-a renuntat la aceasta prevedere. Din cauza ca la Scoala Generala Nr.1 nu erau conditii normale pentru desfasurarea competitiei, intreg concursul s-a desfasurat in liceul Traian. Incepand cu anul scolar 1978-1979, Facultatea de Matematica a Universitatii din Craiova a organizat Concursul Interjudetean de Matematica Gheorghe Titeica. Din aceasta cauza comisia judeteana a hotarat incetarea concursului propriu incepand cu 1980. Avem satisfactia ca am fost cei dintai care am organizat un concurs de geometrie care a purtat numele unui matematician de talie mondiala.

Page 28: RMM 6.pdf

Petre Sergescu

- 26 -

In cele ce urmeaza prezentam rezultatul primului concurs de geometriE Gheorghe Titeica desfasurat pentru elevii din clasele VII-VIII. Extras din Procesul Verbal 14-Octombrie 1973: PREMIUL I Balu Cristina Sc.Gen.3,Profesor I.Gaban Glavici Marius Lic.Traian,Profesor St.Marica Constantinescu Violeta Sc.Gen.14, Profesor Baluta Poganu PREMIUL II Stelescu Stefan, Sc.Gen.5, Profesor C.Covrig Gurbina Mircea Sc.Gen.Nr.2, Profesor R.Bazacov Ivaschescu Livia Liceul Traian, Profesor St.Marica Buse Iustin ,Sc.Gen.9, Profesor El.Vicea PREMIUL III Serbanescu Cristian, Liceul Traian,Profesor Fl.Nica Dragomir Nicolae, Sc.Gen.9, profesorA.Gugu Preotesoiu Pompiliu, Sc.Gen.4,Profesor Ana Vladica Balu Marinela,Sc.gen.Nr.6, Profesor El.Gimanca Serbulescu Mihaela,Sc.Gen.6, Profesor el.Gimanca MENTIUNI Rotaru Dan, Liceul Traian, Profesor St.Marica Vaduva Cristian,Sc.Gen.9, Profesor El.Vicea Derjac Simona,Sc.Gen.6, Profesor Palina Iorga. Ceilalti 55 de concurenti au obtinut un punctaj inferior.....

Page 29: RMM 6.pdf

FOŞTI ELEVI MEHEDINŢENI

- 27 -

H SSM

De la Severin la Rutgers

Nu stiu altii cum sunt, dar eu, cand ma gandesc la casa si familia in care am crescut si la educatia pe care am primit-o, realizez cat de mult m-au influentat in alegerea unei directii pe plan professional. In momentul de fata lucrez la obtinerea unui doctorat in “Mathematics Education” (Stiintele Educatiei in Matematica) in cadrul universitatii Rutgers din New Jersey, Statele Unite. Adeseori ma intreb ce m-a adus pe aceste meleaguri, si inevitabil ma uit la trecut. Am crescut intr-o familie de matematicieni (atat ambii parinti cat si unul dintre bunici fiind profesori de matematica de liceu), alaturi de o sora geamana (Simona) si o sora mai mica (Speranta). Tot ce imi aduc aminte despre experientele cu matematica in scoala primara si gimnaziala este ca intelegeam conceptele si rezolvam problemele cu relativa usurinta. Sa fie acest lucru datorat, in parte, unei “mosteniri genetice”? N-am nici cea mai vaga idee. Cert este ca si surorile mele au manifestat aceeasi abilitate matematica, in conditiile in care nu lucram la matematica aproape deloc inafara scolii. Nu am facut niciodata meditatii, dar prezenta a doi profesori de matematica in propria casa a ajutat cu siguranta. Ajutorul dat de parintii nostri consta in “indicatii subtile” oferite atunci cand ne impotmoleam la vreo problema din tema. Aceste indicatii erau destul cat sa ne scoata din “blocaj”, dar nu contineau niciodata intreaga solutie. Uitandu-ma inapoi, imi dau seama ca formularea acestor “indicatii subtile” este o arta, si numai cineva cu ani de experienta in predarea matematicii o poate stapani. Avand in vedere plenitudinea de “resurse matematice” avute la indemana in copilarie, este curious ca nici una dintre noi, cele trei fete, nu a manifestat un interes deosebit fata de matematica in clasele 1-12 (facute la Scoala Generala Nr. 2, Liceul Gheorghe Titeica si Liecul Traian). Zambesc si acum cand imi amintesc ca singurul fel in care am folosit maldarele de culegeri de matematica prezente in casa noastra a fost ca suport pentru cartile de joc. Au existat ambitii temporare de a obtine rezultate bune la olimpiade de matematica si fizica (Simona si Speranta participand chiar la fazele nationale ale olimpiadelor de matematica si respectiv fizica); eu nu am avut ambitii in aceasta directie, ducandu-ma la olimpiade municipale si judetene mai mult ca sa multumesc profesorii si parintii. Am avut parte de profesori exceptionali (prof. Coada Carmen in gimnaziu si prof. Cainiceanu George in liceu), si simteam ca se asteptau ca noi, elevii buni ai clasei, sa reprezentam scoala la aceste concursuri. S-a intamplat sa nu am nici o motivatie personala sa-mi compar cunostintele matematice cu altii, dar ce consider ca am invatat in anii de gimnaziu si liceu de la acesti profesori este ca matematica este un domeniu al gandirii logice si al cunoasterii prin explorare. Anii de facultate (Universitatea Bucuresti, Facultatea de Matematica) nu au consolidat aceasta perceptie a matematicii. Majoritatea cursurilor luate acolo m-au tarat intr-o lume abstracta in care nu mai stiam de ce studiez acest domeniu. Consider ca am “vegetat” acei 4 ani – m-am lasat dusa de curentul facultatii, pregatindu-ma pentru examene mai mult din

Page 30: RMM 6.pdf

FOŞTI ELEVI MEHEDINŢENI

- 28 -

obisnuinta decat din interes. In ultimul an de facultate am inceput sa fiu interesata de studii in strainatate, si m-am hotarat sa fac un master in Romania pentru a avea mai multe sanse sa fiu acceptata la o universitate buna in Statele Unite, si mai ales sa primesc ajutor financiar. In timpul acestui masterat am aflat accidental (datorita Simonei, sora geamana) despre o pozitie de instructor de matematica la un colegiu de arte din Vermont, Statele Unite. Am aplicat in ultimul minut si m-au angajat! Au urmat doi ani de imersiune intr-o cultura total straina mie pana in acel moment: m-am trezit brusc in pozitia de profesoara, cu care nu eram obisnuita, intr-o tara unde nu cunoasteam pe nimeni. Asa am descoperit ca imi priesc provocarile, si ca ma ajuta sa “cresc” atat pe plan profesional cat si personal. Experienta acelor doi ani m-a ajutat in decizia de a-mi continua studiile la nivel de doctorat, in domeniul stiintelor educatiei in matematica. Asa am ajuns la Rutgers University, in New Jersey (la aproximativ o ora de New York). In Statele Unite se pune mult accent pe “constructivism” in educatie – ideea principala este ca elevii trebuie sa-si “construiasca” propriile idei ca sa priceapa matematica cu adevarat. Din acest punct de vedere, rolul profesorului este acela de “guide on the side” (ghidul de pe margine) si nu de “sage on the stage” (inteleptul de pe scena). Cu alte cuvinte, profesorul trebuie sa creeze situatii in care elevul este lasat sa exploreze o problema noua sau un concept nou, intervenind doar cand e necesar. Departamentul de educatie de la Rutgers primeste fonduri de milioane de dolari de la diverse organizatii pentru a investiga cum se pot aplica diferite versiuni ale constructivismului in scoli. De ce? Pentru ca teste internationale arata ca elevii americani sunt sub nivelul mediu la matematica (clasele 8 si 12) al tarilor industrializate. Momentan, nu exista consens in mediile academice in privinta modului in care sistemul educational american poate fi imbunatatit.

Asadar, treburile sunt complicate la nivel macro intr-un sistem educational necentralizat. In studiile mele, eu ma concentrez pe educatie la nivel micro – cum afecteaza diferitele tipuri de interactiune dintre elev si profesor procesul de invatare, ce fel de activitati trezesc interesul elevilor, etc. Inca nu am decis care va fi subiectul tezei mele de doctorat: poate fi pe teme legate de intrebarile mentionate mai sus, sau pe teme privind calitatea programelor de calificare a profesorilor. Cu putin noroc, voi absolvi in 2008. Nu m-am decis inca ce voi face dupa terminarea doctoratului. Acum ca am vazut cum ti se poate schimba cursul vietii in cateva zile (vezi episodul cu Vermont), nu ma grabesc sa fac planuri pe termen lung!

Iuliana Radu Graduate School of Education 10 Seminary Place New Brunswick, NJ 08901 USA [email protected]

Page 31: RMM 6.pdf

CERCUL DE MATEMATICA

H SSM

- 29 -

Temă pentru grupa de performanţă la clasa a- V -a

Metode de rezolvare a problemelor de matematica

Prof. Victor Saceanu

Rezolvarea unei probleme de aritmetica, daca nu se apeleaza la metodele elgebrice, devine dificila la nivelul claselor IV-VI, in cazul cand nu se cunoaste metoda sau metodele prin care aceasta poate fi rezolvata. De aceea in cele ce urmeaza, voi cauta sa prezint metodele de baza ce se pot folosi in rezolvarea problemelor de aritmetica, urmate de 1-2 exemple concrete Consider ca este necesar acest lucru, deoarece atat programa cat si manualele de matematica de la nivelul claselor IV-VI, prezinta succinct sau nici nu amintesc de aceste metode. 1.Metoda figurative (grafica) – se aplica in rezolvarea problemelor in care se cunosc:

a) Suma si diferenta a doua marimi: “O barac merge in sensul curentului unei ape cu 16km/h, iar contra curentului cu 12km/h.Care este viteza barcii in apa statatoare si care este viteza apei?” Solutie: Fie: ------------- - viteza barcii in apa statatoare =Vb -------- - viteza apa = Va ------------------ = 16 km/h ------------ De unde deducem ca 2xVb=16+12→Vb=14km/h→Va=2km/h Obs: In acest tip de probleme daca deducem suma cu diferenta obtinem de doua ori marimea mai mare, iar daca le scadem obtinem de doua ori marimea mai mica.

b) Suma sau diferenta si raportul celor doua marimi “ Ce ora este acum, daca partea ramasa din zi este de 6x1/2 ori mai mica decat partea care a trecut ?” Solutie: Fie: --------- = partea ramasa = 1p │ ----------------------------------- = partea trecuta │ =24h Atunci 1p+6x1/2p=7x1/2 parti 24:7x1/2=3x1/5 ore 24-3x1/5=20x4/5 ore Este ora 20 si 48 de minute

“Intr-o clasa, numarul elevilor absenti este de 1/6 din numarul celor prezenti . Cati elevi sunt in clasa, daca numarul celor prezenti este cu 25 mai mare decat numarul celor absenti ? Solutie: Fie: ------ -elevi absenti │ ------------------------------ = elevi prezenti │diferenta = 25 de elevi 1p-1/6p=5/6p 25:5/6=30 elevi sunt in clasa

c) Suma mai multor marimi si diferentele sau rapoartele ce exista intre ele. “ Suam a trei numere naturale a,b,c este 900. Aflati numerele stiind ca b este de 3 ori mai mare ca a si cu 25 mai mic decat c.

Page 32: RMM 6.pdf

CERCUL DE MATEMATICA

- 30 -

Solutie: Fie: ------ = a │ --------------------- = b │ S = 900 ------------------------ = c │

Avem 7p+25=900→7p=875→1p=125 Deci numerele sunt a = 125; b = 375; c=400

2) Metoda comparatiei ( aducerii la acelasi termen de comparative ) Aceasta metoda consta in: a) Eliminarea unei marimi prin scadere: Exp: “Daca 20 de caiete si 35 de creioane costa 54lei noi, iar 15 caiete si 9 creioane costa 26,70 lei, sa se afle pretul uniu caiet si al unui creion.”

Solutie : Avem asezarea: 20caiete----------35creioane--------54lei 15caiete-----------9creioane---------26,70lei Facem ca marimea caiete, de exemplu, sa aiba aceiasi valoare in ambele cazuri si atunci luam prima data de 3ori mai mult si a doua oara de 4ori mai mult si obtinem: 60caiete-----------105creioane---------162lei 60caiete-----------36creioane----------106,80lei Prin scadere obtinem ca: 69creioane costa 55,20lei, de unde 1 creion costa 0,80lei . Inlocuind in una din situatii creioanele obtinem ca un caiet costa 1,30lei.

d) Eliminarea unei necunoscute prin inlocuirea ei: Exp: “Un costum de haine costa cat doua uniforme scolare, iar o uniforma costa cat cinci camasi. Sa se afle pretul fiecaruia din aceste obiecte, stiind ca 40de costume, 150 uniforme si 120de camasi costa impreuna 81280lei.” Solutie: Avem ca 40costume costa cat 40x2=80uniforme Uniforme avem acum : 80+150=230 care ele costa atat : 230x5=1150camasi. Deci in final avem : 1150+ 120=1270 camasi care costa 81280lei, de unde o camasa costa 64lei, o uniforma costa 5x64=320lei si un costum costa 2x320=640lei. 3. Metoda falsei ipoteze – consta in aceea ca ambele marimi sunt presupuse la fel si apoi se deduce diferenta care apare, di ferenta din care se poate afla marimea care a ramas. Exp: “Intrebat fiind, cati porumbei si cati iepuri are, un elev a raspuns: In total sunt 51de capete si 132de picioare.Cati prumbei si cati iepuri are elevul?”

Solutie: Presupunem ca ar avea numai prumbei. Atunci ar fi in total 51x2=102 picioare, deci mai raman in plus 132-102=30picioare. Acestea provin din faptul ca un iepure are in plus 4-2=2picioare. Atunci cele 30picioare le impartim in grupe de cate 2 picioare si obtinem 15iepuri si 51-15=36porumbri Exp: “Intr-un bloc sunt 39apartamente cu 4 si 3 camere. Cate apartamente de fiecare fel sunt, daca numarul total al camerelor este 141 ?” Solutie : Presupunem ca toate apartamentele sunt cu 3camere si am avea 39x3=117camere, deci mai raman in plus 141-117=24camere .Acestea provin din faptul ca avem si apartamente cu 4 camere, deci cu una in plus. Cele 24camere ne dau si cele 24apartamenete cu 4camere iar 39-24=15 apartamente cu 3camere.

Page 33: RMM 6.pdf

CERCUL DE MATEMATICA

H SSM

- 31 -

4. Metoda drumului invers(retrograda) – consta in aceea, ca in rezolvarea problemelor, o luam cu rationamentul de la datele din finalul lor, pana atunci ajungem la inceput, adica la cele cerute in problema.

Exp: “O taranca a vandut oua la 3cumparatori astfel: primului jumatate din oua ce le avea si inca o jumatatede ou la al doilea, jumatate din oua ramase si inca o jumatate de ou, iar la al treilea jumatate din oua cu care mai ramansese si inca o jumatate de ou si astfel le-a vandut pe toate. Cate oua a vandut taranca ?” Solutie: Daca al 3-lea comparator nu lua jumatate de ou atunci el ar fi cumparat jumatate din cantitatea care era si ar fi ramas in cos o cantitate de oua egala cu aceea cumparata de el, adica o jumatate de ou si ouale sau terminat, inseamna ca el a cumparat 1 ou, pe care l-a gasit in cos dup ace a cumparat al 2-lea. Deci al doilea a gasit in cos 1x1/2+1x1/2=3 oua si el a cumparat 1x1/2+1/2= 2 oua. Atunci primul a gasit in cos 3x1/2 + 3x1/2 =7 oua si a cumparat 3x1/2 +1/2=4oua. Deci taranca a avut 7oua Exp: “La un centru de vanzare a painii inainte de inchidere erau 4cumparatori care au cumpart toata cantitatea de paine.Sa se afle cate paini au fost,daca fiecare comparator a acumparat jumatate din painea care se mai gasea in momentul in care i-a venit randul si inca cate o jumatate de paine” Solutie: Daca al 4-lea comparator a terminat painea ce mai ramansese, inseamna ca el a cumparat 1/2 +1/2 =1 paine acre ramansese de la al 3-lea. Acesta a gasit 1x1/2 +1x1/2=3paini ce ramasesera de la al 2-lea si el a cumparat 1x1/2+1/2=2paini. Al 2-lea inseamna ca a gasit 3x1/2+3x1/2=7paini si a cumparat 3x1/2+1/2= 4 paini. Daca primul i-a lasat celui de-al 2-lea 7paini inseamna ca el a gasit 7x1/2+7x1/2=15 paini , si a cumparat 7x1/2+1/2 =8paini. Deci au fost 15 paini. Obs: Rezolvarea unei probleme de aritmetica se poate face, fie aplicand una din metodele expuse, dar si combinate,deoarece problemele de aritmetica nu pot fi clasificate nici dupa datele lor nici dupa al cerintelor lor. Prin cele expuse in acest material, consider ca nu au adus o noutate in rezolvarea problemelor de aritmetica ci am dorit sa reamintesc cele mai importante metode re rezolvare,spre a fi folosite la clasa sau la pregatirea loturilor olimpice, evitandu-se metodele algebrice. Bibliografie: 1. Metodica de predare a matematicii – E.D.P – 1954 2. Probleme de matematica pentru gimnaziu – I.Petrica s.a 3. Culegere de probleme de matematica – Bucuresti – 1987 4. Gazeta matematica – 1980 – 2005.

Page 34: RMM 6.pdf

CERCUL DE MATEMATICA

- 32 -

Tema pentru grupa de performanta la clasa a- VII -a

Formule de calcul prescurtat

Angela Niţoiu & Manuela Opriţa Colegiul Tehnic Decebal

1. an – bn= (a – b )(an-1 + an-2b + … + abn-2 + bn-1 ) Demonstraţie Pornim de la membrul drept şi ajungem la membrul stâng : (a – b )(an-1 + an-2b + … + abn-2 + bn-1 ) = an + an-1b + an-2b2 + … +a2bn-2 + abn-1- an-1b – - an-2b2 - … - a2bn-2 - abn-1 – bn = an – bn

2. a2k+1 + b2k+1 = (a + b)(a2k – a2k-1b + a2k-2b2 + … + b2k) Demonstraţie Pornim de la dreapta la stânga şi înmulţim termen cu termen obţinem: (a + b)(a2k – a2k-1b + a2k-2b2 + … + b2k) = a2k+1 – a2kb + a2k-1b2 + … +ab2k + a2kb + a2k-2b3 + … - ab2k + b2k+1 = a2k+1 + b2k+1

3. (a + b)0 = 1 (a + b)1 = a + b (a + b)2 = (a + b)(a + b) = a2 + 2ab + b2

(a + b)3 = (a + b)2(a + b) = (a2 + 2ab + b2)(a + b) = a3 + 3a2b + 3ab2 + b3

(a + b)4 = (a + b)3(a + b)=(a3 + 3a2b + 3ab2 + b3)(a + b)=a4 + 4a3b + 6a2b2 + 4ab3 + b4

(a +b)5 = (a + b)4(a + b)=( a4 + 4a3b + 6a2b2 + 4ab3 + b4) (a + b) = a5 + 5a4b + 10a3b3 + 10a2b3 + 5ab4 + b5

(a + b)6 = (a + b)5(a + b)=( a5 + 5a4b + 10a3b3 + 10a2b3 + 5ab4 + b5)(a + b) =a6 + 6a5b + 15a4b2 + 20a3b3 + 15a2b4 + 6ab5 + b6

Coeficienţii acestor binoame ( coeficienţi binomiali) se pot calcula din triunghiul lui Pascal după cum urmează: 1 n ═ 0

1 1 n ═ 1 1 2 1 n ═ 2 1 3 3 1 n ═ 3 1 4 6 4 1 n ═ 4 1 5 10 10 5 1 n ═ 5 1 6 15 20 15 6 1 n ═ 6 ………………………………. Exerciţiu: Calculaţi: (a + b)7 . Rezolvare: Se observă pe baza exemplelor calculate anterior şi din triunghiul lui Pascal următoarele: (a + b)7 = (a + b)6(a + b)=( a6 + 6a5b + 15a4b2 + 20a3b3 + 15a2b4 + 6ab5 + b6)(a + b) = a7 + 7a6b + 21a5b2 + 35a4b3 + 35a3b4 + 21a2b5 +7ab6 + b7 4. Identitatea lui Lagrange :

(i) (a2 + b2)(c2 + d2) = (ac + bd)2 + (ad – bc)2

(ii) (a2 + b2 + c2)(m2 + n2 + p2) = (am + bn + cp)2 + (an – bm)2 + (ap – cm)2 + (bp – cn)2

(iii) Generalizare ∑∑n

1-

2i

1-

2

i

n

ii ba ═

2

1-⎟⎠

⎞⎜⎝

⎛∑n

iiiba + ( )

2

1∑

≤≤≤

−nji

ijji baba

Aplicaţii:

Page 35: RMM 6.pdf

CERCUL DE MATEMATICA

H SSM

- 33 -

1. Arătaţi că:

a) 12)12005( 2006 M− b) 9)54( 20052005 M+

2. Să se calculeze folosind triunghiul lui Pascal: a) ( )432 −x b) ( )525 xxy −

3. Dacă 31=+

aa , calculaţi : .1,...,1,1,1,1

44

33

22

nn

aa

aa

aa

aa

aa +++++

4. Dacă 2=+ab

ba , calculaţi 33

66

baba + .

Bibliografie: [1] C. Năstăsescu, C. Niţă ş.a., Exerciţii şi probleme de algebră, EDP , Bucureşti, 1981 [2] Revista de matematică Minimath, Nr.1 /2005, Editura Reprograph Tema pentru grupa de performanta la clasa a- VIII –a

Probleme de algebră cu soluţii geometrice

prof. Mihai Octavian Ungureanu

P.1. Determinaţi numerele reale strict pozitive x şi y care au suma S şi produsul P. Soluţie SyxDCBDyDCxBD =+=+== ,,

PxyDCBDADAmABC

BCAD IT

o==⋅=⇒

⎪⎭

⎪⎬⎫

=∆

⊥2

..

^90)(,

Triunghiul ABC poate fi construit dacă şi numai dacă paralela la BC intersectează semicercul de diametru S.

PxyyxS 22 =≥+=⇒

PDCBDADSBCAO =⋅=== ,22

242

222 PSDOADAODO −=⇒−=

24

24

2

2

PSSCODODCy

PSSDOBOBDx

−+=+==

−−=−==

Page 36: RMM 6.pdf

CERCUL DE MATEMATICA

- 34 -

P.2. Determinaţi numerele reale strict pozitive x şi y care au diferenţa d şi produsul p. Soluţie presupunem Pxydyxyx ==−> ,, Punctele M, A, B din figură sunt coliniare

yxAByMBxMA −=== ,,

222

2

dyxABOT

pMTpxyMBMAMT

=−

==

=⇒==⋅=

24

90)(,2..^ pd

MOTmMOTPT

o +=⇒=∆

2424

2

2

dpdOAMOMAx

dpdOBMOMBy

++=+==

−+=−==

P.3. Determinaţi numerele reale strict pozitive x şi y care au suma S şi diferenţa d. Soluţie Fie x ≥ y, S = x + y, d = x – y A, M, B, C coliniare, M mijlocul [AC] AM = x, MB = y AB = AM + MB = x + y = S BC = MC – MB = x – y = d x = AM = AC/2 = (AB+BC)/2 = (S+d)/2 y = MB = MC - BC = (S+d)/2 – d = (S - d)/2

P.4. Determinaţi numerele reale strict pozitive x şi y care au produsul p şi raportul k.

Soluţie kyx

DCBDpxyDCBDyDCxBD ====⋅== ,,, .

kACABk

BCCDBCBD

ACABpDCBDAD =⇒=

⋅⋅

==⋅= 2

2

;

ACABADBD

ACAB

ADBDDACDBA =⇒=⇒∆∆ ~

pkkpACABADBDx =⋅===

ABACADDC

DCAD

ACABDACDBA =⇒=⇒∆∆ ~

kp

kp

ABACADDCy =⋅===

1

P.5. Determinaţi numerele reale strict pozitive x şi y care au suma S şi raportul k.

Soluţie kACABSyxBCyDCxBD ==+=== 2

2

,,, (aceeaşi figură ca la problema 4)

1122

2

2

22.

+=−=⇒

+⋅=

+⋅=⋅===

kSxSy

kkS

ACABABBC

BCABBC

BCABBDx

cateteiT

Page 37: RMM 6.pdf

CERCUL DE MATEMATICA

H SSM

- 35 -

P.6. Determinaţi numerele reale strict pozitive x şi y care au diferenţa d şi raportul k.

Soluţie dyxPBPAAByPBxPA =−=−=== ,,

)2(,2

,2

problemaxyPTyxPOdOT =+

==

yxyxxy

POTOPTTDPOT

+−

=⋅

=⇒∆)(

2

22222

)()(

2 yxyxxyyxTDTODOTDO

+−

−⎟⎠⎞

⎜⎝⎛ −

=−=⇒∆

)(2)( 2

yxyxDO+−

=⇒

⎪⎪⎩

⎪⎪⎨

+=⇒

=+

=+

=+

⇒===⇒

⎪⎪⎭

⎪⎪⎬

+−

=+=

+−

=−=

22

2

2

22

2

2

2

1

1;

)(

)(

dk

kTA

ABTA

TBTATA

DBDADA

kk

TBTA

DBDAk

yx

DBDA

yxyxxOADODA

yxyxyDOBODB

)1(2)1(;

)1(2)1(;

1

22

2

22

2

2

−+

==+−

=−=+

==⋅

=⋅

=kkd

DOTOPO

kkdOADADO

kkd

ABTA

TBAB

TBTA

TBDBTADA

1;

1 −=+==

−=−==

kkdABPBPAx

kdOBPOPBy

Tema pentru grupa de performanta la clasa a-IX-a

ALGORITMUL „PLICULUI”

prof. DRAGA-TATUCU MARIANA

prof. DRAGA-TATUCU PORFIREL Se consideră N ∈≥ nn ,3 , n este impar, ansamblul crescător al numerelor reale { }2...21 naaaA <<<= , ale cărei elemente sunt în progresie aritmetică cu 0>r . Se cere proiectarea unui algoritm pentru generarea cu elementele ansamblului A a unei matrici

pătratice nnaA ij ×= )( , care să verifice simultan egalităţile: ∑∑∑∑=

+−===

====n

inini

n

iii

n

iij

n

jij Saaaa

11,

111

,

unde Sn este o valoare ce se va preciza.

Descriere algoritm.

Elementele ansamblului A precizat fiind în progresie aritmetică, rezultă ∑⋅+

=2

)( 21 2 naa

A n .

Deoarece elementele matricei nnaA ij ×= )( care se cere generată, verifică condiţiile din

enunţ, rezultă ∑⋅+

==2

)(;1 21 naa

SAn

S nnn .Orice matrice pătratică A care verifică condiţiile

Page 38: RMM 6.pdf

CERCUL DE MATEMATICA

- 36 -

precizate în enunţ se numeşte pătrat magic de ordin n, având suma magică Sn care depinde de elementele ansamblului A.

Dacă n este par, generarea unui pătrat magic A cu elementele unui ansamblu A se face cu ajutorul algoritmului permutărilor.

Pentru cazul 3≥n , număr natural impar, pentru generarea unei matrici A cu elementele ansamblului A care să verifice condiţiile precizate în enunţ, vom utiliza metoda teraselor datorită lei Bachet (sau metoda plicului).

Metoda teraselor constă în următoarele: se consideră un caroiaj pătratic de dimensiuni

)12()12( −×− nn de tipul următor: În caroiajul considerat, se numeşte origine a acestuia, mijlocul oricărei laturi frontiere al lui. Deci, există patru origini posibile.

Se defineşte sens de terasare (de generare) sensul considerat de la originea aleasă spre mijlocul uneia din laturile frontieră cea mai apropiată. Deci, din fiecare origine pot fi considerate două sensuri de terasare.

Pentru a face o alegere, vom considera că mijlocul laturii frontieră cea mai din stânga este originea acestuia, iar sensul de la originea aleasă

spre mijlocul laturii frontieră cea mai de sus este sensul de terasare sau sensul de generare în caroiaj.

Odată fixată originea şi sensul, pe tot parcursul terasării, sensul de terasare trebuie menţinut. Terasarea elementelor ansamblului în caroiaj se va face în ordinea crescătoare a valorilor lor, în n terase paralele, fiecare terasă având n – elemente distincte din ansamblu.

Terasarea se va face astfel:

În urma terasării tuturor elementelor ansamblului A, în caroiajul considerat se obţine un romb valoric de forma:

Unind mijloacele laturilor rombului valoric şi ducând diagonalele pătratului obţinut rezultă

patru triunghiuri valorice exterioare pătratului şi patru triunghiuri valorice interioare. Triunghiurile valorice obţinute au următoarele proprietăţi: oricare două triunghiuri valorice,

unul exterior şi unul interior pătratului, sunt complementare valoric, în sensul că dacă o poziţie precizată dintr-un triunghi valoric exterior este ocupată de un element al ansamblului A, poziţia corespunzătoare din oricare triunghi valoric, interior pătratului, este vidă. Aceasta înseamnă că, dacă un triunghi valoric exterior este suprapus pe oricare triunghi valoric interior se va obţine un triunghi valoric plin, adică un triunghi în care toate poziţiile sale sunt ocupate de elemente distincte ale ansamblului A considerat.

an .

. a2

.

a2n

a1 an+2 2na an+1

22 +−nna

12 +−nna

2

2

4

4

1 1 3 3

1………………..n…...………….2n-1

1 . . n . . . 2n-1

Page 39: RMM 6.pdf

CERCUL DE MATEMATICA

H SSM

- 37 -

Dacă se numerotează cu 1 – 4 triunghiurile valorice exterioare pentru a obţine un pătrat valoric plin ale cărui elemente să reprezinte o matrice nnaA ij ×= )( , care să verifice condiţiile din enunţ, suprapunerea triunghiului exterior peste triunghiul valoric interior trebuie să se facă astfel: 11→ din dreapta 22 → de jos 33 → din stânga 44 → de sus

Fiind dat ansamblul A cu 3≥n număr natural impar, cu ajutorul metodei teraselor pot fi generate 8 matrici distincte având aceeaşi sumă magică Sn.

Caz particular. Să se exemplifice algoritmul

proiectat pentru următorul caz: { } 5,49,...,7,5,3,1 == nA . Rezolvare:

1252

5)491(5 =

⋅+=S

Matricea căutată este:

⎟⎟⎟⎟⎟⎟

⎜⎜⎜⎜⎜⎜

=

451933721113592347371254913327411539

294317315

A

9

7 19

5 31 17 43 29

3 39 15 41 27 3 39

1 13 49 25 1 37 49

11 47 23 9 35 11 47

21 7 33 19 45

31 43

41

Page 40: RMM 6.pdf

CERCUL DE MATEMATICA

- 38 -

Tema pentru grupa de performanta la clasa a-IX-a

TEOREMA LUI CASEY

eleva Otilia Stretcu, Liceul Teoretic ,,Gh. Ţiţeica”

Pentru a prezenta subiectul acestei teoreme este necesară prezentarea următoarelor leme: LEMA 1 Se dau cercul C (O,R) şi cercurile C (O1,R1), C (O2,R2) tangente interioare cercului C (O,R), în

punctele A, respectiv B. Să se arate că distanţa dintre cercurile C (O1,R1) şi C (O2,R2) (lungimea tangentei exterioare comune) este :

dAB )r)(Rr(RR

AB21 −−= .

Demonstraţie: Din trapezul dreptunghic O1O2B1A1 avem:

d 2AB =A1B1

2 =O1O22 –(r1-r2)2.

Din ∆O1OO2 obţinem:

O1O 22 = OO1

2 +OO22 - cosαOOOO2 21 ⋅⋅⋅ =

=(R-r1)2 +(R-r2)2- cosα)r-(R)r(R2 21 ⋅⋅−⋅ . (2)

Cum αcos22 222 RRAB −= ⇒ 2

22

2RAB2R cos −

=α .

Înlocuim (3) în (2) şi apoi (2) în (1) şi obţinem:

dAB )r)(Rr(RR

AB21 −−= .

LEMA 2 Se dau cercul C (O,R) şi cercurile C (O1,R1), C

(O2,R2) tangente exterioare cercului C (O,R), în punctele A, respectiv B. Să se arate că distanţa dintre cercurile C (O1,R1) şi C (O2,R2) este :

dAB )r)(Rr(RR

AB21 ++=

Demonstraţie: Analog cu Lema 1, dar (2) se înlocuieşte cu

cosα)rR()r(2)()(OO 212

22

1221 ⋅+⋅+−+++= RrRrR

obţinându-se în final relaţia dAB )r)(Rr(RR

AB21 ++=

TEOREMA LUI CASEY Dacă cercurile C (O1,R1), C (O2,R2), C (O3,R3), C (O4,R4) sunt tangente interior

cercului C (O,R) (sau exterior) în punctele A,B,C,D∈ C (O,R) astfel ca ABCD să fie

patrulater convex şi dacă notăm cu dAB distanţa dintre cercurile C (O1,R1), C (O2,R2) şi analoagele, atunci avem:

BDACADBCCDAB dddddd =⋅+⋅ . Demonstraţie:

Se exprimă BDACADBCCDAB d,d,d,d,d,d conform formulei (4) (sau (5)) şi se înlocuiesc în relaţia:

(2’)

(5)

(1)

(3)

Page 41: RMM 6.pdf

CERCUL DE MATEMATICA

H SSM

- 39 -

BDACADBCCDAB dddddd =⋅+⋅ obţinându-se relaţia lui Ptolemeu. Observaţie:

1) Dacă toate cele patru cercuri degenerează în puncte găsim chiar teorema lui Ptolemeu, iar din aceasta în cazul patrulaterului dreptunghic obţinem teorema lui Pitagora.

2) Din a doua teoremă a lui Ptolemeu, care afirmă că dacă ABCD este patrulater inscriptibil,

atunci DCDABCBACDCBADAB

BDAC

⋅+⋅⋅+⋅

= se poate demonstra o generalizare a acestei relaţii şi anume:

BCDABCBA

CDCBADAB

BD

ACdddddddd

dd

⋅+⋅⋅+⋅

= .

APLICAŢII:

1) Fie triunghiul ABC înscris în cercul O (O,R) şi un punct D pe latura [BC]. Se consideră cercurile C şi D tangente la O şi AD; primul cerc este tangent şi la BD iar al doilea la DC. Să se arate că cercurile C şi D sunt tangente dacă şi numai dacă m( BAD∠ )=m( CAD∠ ).

(D. Brînzei) Rezolvare:

Se consideră punctul E, intersecţia dintre AD şi cercul O. Fie M punctul de tangentă al cercului C cu BD, iar N punctul de tangenţă al cercului D cu DC. Aplicăm teorema Casey pentru cercurile degenerate B, C, E şi C şi avem: EPBCBMCECMBE ⋅=⋅+⋅ , unde P este punctul de tangenţă al cercului C cu AD. Din teorema Casey pentru cercurile degenerate B, C, E şi D obţinem,

EQBCBNCECNBE ⋅=⋅+⋅ , unde Q este punctul de tangenţă al cercului D cu AD. Cercurile C şi D dacă şi numai dacă P=Q. Scăzând relaţiile precedente rezultă:

BM)(CECN)-CM(BE −⋅=⋅ BN sau BE = CE , adică m( BAE∠ )=m( CAE∠ ).

2) Fie cercul Γ circumscris unui triunghi ABC şi I centru cercului înscris în triunghi. Se consideră cercul tangent laturilor AC, BC, respectiv în D,E şi tangent cercului Γ . Să se arate că I este mijlocul segmentului DE.

Rezolvare: Este suficient să demonstrăm că I ∈[DE]. Notând BE=x şi AD=y, din teorema lui Casey

aplicată punctelor A, B, C şi cercului Γ rezultă: xb +za =(a-x)c. (6) Din CE=ED ⇒ a – x = b – y ⇒ z = b – a + x. (7)

Din (6) şi (7) obţinem: x=p

b)a(p − şi y = p

a)b(p − . (8)

Din teorema transversalei I ∈[DE] ⇔IC

IC'ABDCADBC'

ECBEAC' =⋅+⋅ (9)

unde CI }{C'AB =∩ .

Din teorema bisectoarei ⇒ba

cIC

IC'+

= , înlocuim în (9)⇒ba

cyb

yba

caxa

xab

cb 2

+=

−⋅

++

−⋅

+

⇔ cbabppb

a)ba(pabappa

b)ba(p=

+−−

++−− , evident adevărată, după ce am folosit (8).

Observaţie: Pentru a exersa aplicarea teoremei lui Casey se recomandă rezolvarea

problemelor din bibliografie. BIBLIOGRAFIE:

[1] M. Ganga, ,,Probleme elementare de matematică pentru gimnaziu şi liceu”, Ed.Mathpress 2003. [2] L. Nicolescu şi V. Boskff, ,,Probleme practice de geometrie”, Editura Tehnică, Bucureşti 1990. [3] Gheorghe Ţiţeica, ,,Probleme de geometrie”, Editura Tehnică, Bucureşti, 1982. [4] V. Vornicu, ,,Olimpiada de matematică - de la provocare la experienţă”, Editura Gil, 2003

Page 42: RMM 6.pdf

CERCUL DE MATEMATICA

- 40 -

Tema pentru grupa de performanta la clasa a-X-a

COMENTARII METODICE ASUPRA UNOR PROBLEME DATE LA OLIMPIADELE SUA

Prof.dr.Gheorghe Cainiceanu C.N.Traian, Drobeta Turnu Severin

§1 Introducere De-a lungul timpului am fost martorii mai multor tipuri de “filosofii”; de la credinta ca noi

suntem cei mai grozavi de pe pamant pana la paguboasa insinuare ca suntem mici si saraci si neimportanti. Scopul acestui articol este sa arate cum pe o structura ce imita metodele romanesti de selectie ,o organizare superioara si o treaba bine facuta pot in timp da rezultate remarcabile... Sa nu uitam ca in urma cu 15 ani SUA nu prea conta in lupta pentru medalii la Olimpiada Internationala de Matematica iar in prezent intr-un clasament cumulat pe anii 1997-2001, SUA se afla pe locul doi, Romania situindu-se pe locul 8. In SUA selectia lotului national se desfasoara dupa urmatorul algoritm: American Mathematics Competitions AMC 10/12 (Olimpiada locala- prin corespondenta, subiecte unice, concurs deschis unui numar mare de scoli agreate din lume) American Invitational Mathematics Competitions AIME (Olimpiada judeteana- prin corespondenta, participa primii aproximativ 1% din concurentii de la AMC 10 si 5% din concurentii de la AMC 12). USAMO (Olimpiada Nationala- participa doar elevii americani care au intrunit un punctaj cumulat de peste 200p din maximul de 300p realizabil). Test Selection (baraje). MOSP (Mathematical Olimpiad Summer Program- Pregatirea lotului). In articolul de fata ma voi opri asupra a doua probleme din faza AIME, alese oarecum la cei doi poli de dificultate. Mentionam ca regulamentul concursului prevede ca elevii sa scrie pe fisa de concurs doar un rezultat numeric de trei cifre la fiecare din cele 15 probleme ce treebuiesc rezolvate in trei ore.

§2 Problema 1 AIME 1998 Pentru cate valori ale lui k numarul 1212 este cmmmc al numerelor 66 , 88 si k ?

Comentariu Este de remarcat faptul ca acesta problema este practic accesibila unui elev chiar de clasa a V-a. De ce o astfel de problema? Iata ce scrie d-l Steven dunbar Directorul AMC intr-o scrisoare adresata parintilor elevilor participanti la AMC: “...problemele sunt in afara tiparelor (outside of the box)...chiar daca fiul sau fiica dumneavoastra rezolva o singura problema sau doua din concurs trebuie sa fiti mandrii de el caci aceste probleme sunt altfel decat problemele obisnuite de la clasa...” Solutie 66=26-36; 88=224 ;1212=224-312. desigur vom lua k=312-2p cu p lund valori de la 0 la 24 deci raspunsul este “025”

§3 Problema 13 AIME 1998 Daca {a1,a2,...an} este o multime de numere reale cu elementele ordonate strict crescator vom denumi CPS a sa numarul a1i + a2i2 + a3i3 +....anin. Fie Sn suma CPS corespunzatoare tuturor submultimilor nevide ale ale multimii {1,2,…n}. Daca S8 =-176- 64i si S9 = p+qi determinati |p| +|q|.

Page 43: RMM 6.pdf

CERCUL DE MATEMATICA

H SSM

- 41 -

Comentariu Se observa deja un nivel de dificultate sporit ,in primul rand pentru intelegerea corecta a enuntului. Se simte si se va confirma pe parcursul rezolvarii ca problema se adreseaza unui elev de cel putin clasa a X-a, fiind necesare cunostinte de numere complexe, dar si de combinatorica pentru numararea submultimilor. Solutie Analizam posibilitatea calculului lui S9.Multimea {1,2,...9} are 29 – 1 submultimi nevide.Sumele care apar din cauza submultimilor de un element le vom nota cu T1

(1) , T1(2) , ....T1

(9)

si desigur ∑=

9

1

)(1

k

kT = 45i. Sumele datorate submultimilor de doua elemente le vom nota T2(1,2) , T2

(1,3)

,.....,T2(8,9). De exemplu T2

(k,l) = ki+li2 si ∑≤<≤ 91

),(2

lk

lkT = ∑≤<≤ 91 lk

(ki – l).

Evident S9 = ∑ T1 + ∑ T2 + ∑ T3 +….∑ T8 + T9. Apar doua intrebari " Nu este prea complicat sa calculam aceste sume ? " "De ce am primit in enunt S8? " Esta firesc chiar sa ne intrebam daca se poate generaliza problema. Se naste astfel ideea de a incerca gasirea unei relatii de recurenta intre Sn+1 si Sn. Sa urmarim din cine este formata Sn+1. Submultimile lui {1,2,....,n,n+1} sunt toate submultimile lui {1,2,...,n} si apoi toate submultimile lui {1,2,...,n} la care se adauga n+1. Vom evita submultimea vida care nu are o CPS. Pentru inceput constatam ca Sn+1 = Sn + Un+1. Sa ne apucam de calculul lui Un+1. O submultime ce contine n+1, si are k+1 elemente este de forma a1<a2<...<ak<n+1.

CPS = a1i + a2i2 + ....+akik + (n+1)ik+1, Deci Un+1 este formata din CPS-uri date de toate submultimile nevide ale lui {1,2,...n}si la fiecare din ele (n+1)ik+1 in numar egal cu Cn

k. Deci Un+1=(n+1)i+Sn+(n+1)i2 Cn

1+(n+1)i3 Cn2+....+(n+1)in Cn

n-1+ (n+1)in+1Cnn =

Sn+(n+1)i(1+i)n. In concluzie Sn+1 = 2Sn + (n+1)i(1+i)n ,deci

S9 = 2 S8 + 9 i(1+i)8 = - 352 +16i, si deducem ca |p| + |q| = 368. Rezolvarea problemei s-a incheiat dar ea a deschis urmatoarea intrebare: putem gasi formula termenului general pentru Sn din moment ce avem o relatie de recurenta? Raspunsul este da, iar calea de urmat este urmatoarea:

Sn = 2 Sn-1 + i n (1+i)n-1 2 Sn-1= 22 Sn-2 + 2 i (n-1) (1+i)n-2 .................... 2n-2 S2 = 2n-1 S1 + 2n-2 i- 2 -(1+i)1 prin adunare vom obtine: Sn = 2n-1S1 + i[20n(1+i)n-1 + 21(n-1)(1+i)n-2 + ....+2n-2-2(1+i)1]

Cum S1=i deducem: Sn = i[20n(1+i)n-1 + 21(n-1) (1+i)n-2 + ....+2n-2-2(1+i)1 + 2n-1-1-(1+i)0]=

=i-2n-1[n(2

1 i+ )n-1 + (n-1) (2

1 i+ )n-2 + ....+ 1-(2

1 i+ )0].

Cu notatia z= 2

1 i+ avem ca Sn = 2n-1-i (1+2z+3z2+...+nzn-1) şi prin inmultire cu 1-z

(1-z) Sn = 2n-1i [1+z+....+zn-1 – nzn] = 2n-1i [z

z n

−−

11 - nzn].

Dupa o noua inmultire cu (1-z)-1 = 1+i , obtinem

Sn = 2n-1i [2i(1-(2

1 i+ )n) – n(1+i)(2

1 i+ )n]= ....=(n+1+i)(1+i)n-1 – 2n.

Sa verificam acum rezultatul obtinut pentru cazul n=8: S8 = (9+i)(1+i)7 – 256 = - 176 – 64i.

Page 44: RMM 6.pdf

CERCUL DE MATEMATICA

- 42 -

Sa remarcam in finalul acestei prezentari frumusetea si abordabilitatea problemelor, naturaletea lor, deschiderea lor spre generalizari si incitarea elevului spre o cercetare mai profunda dupa ce a obtinut rezolvarea. Mai este de remarcat faptul ca unele probleme pot avea abordari informatice, lucru care apropie elevii, cunoscuta fiind pasiunea crescanda a lor pentru calculator.

BIBLIOGRAFIE [1] American Invitational Mathematical Examination AIME 1998 [2] Titu Andreescu, Zuming Feng USA and International Mathematical Olympiads 2001

Published and distributed by The Mathematical Association of America 2001 Tema pentru grupa de performanta la clasa a-X-a

Tetraedrul şi capcanele lui

Prof: Ion Chilea Elev: Aida Giurcan

Motto: ,,Geometria tetraedrului are strălucirea şi duritatea diamantului”

Definiţia 1 Se numeşte ,,înălţime” a unui tetraedru perpendiculara dusă dintr-un vârf al tetraedrului pe planul feţei opuse. Notaţii Vom nota tetraedrul de referinţă (A)= 4321 AAAA . Planul feţei opuse vârfului iA , i= 4,1 va fi notat ( ) ( )khj

i AAAA = , {i,j,k}={1,2,3,4}. Faţa opusă vârfului iA este un triunghi ce se va nota cu

( ) khji AAAA ∆= şi aria sa cu 4,1, =iSi . Unghiul dintre planele ( )iA şi ( )jA va fi notat cu ijα . Lungimea muchiei ji AA se va nota cu ija , iar piciorul înălţimii duse din vârful iA va fi notat cu

4,1, =iH i . Lungimea segmentului ii HA va fi notat cu 4,1, =ihi .

Vom enunţa în continuare câteva teoreme referitoare la volumul tetraedrului. Teorema 1 Înălţimile unui tetraedru sunt invers proporţionale cu ariile feţelor corespunzătoare, adică 44332211 hShShShS === Demonstraţia o lăsăm pe seama cititorului. Definiţia 2 Prin volumul tetraedrului (A) înţelegem numărul notat V=v ( )4321 AAAA şi

definit prin V= 4,1,3

=ihS ii .

Teorema 2. Fiind dat tetraedrul (A) şi punctele 3,1,4 =∈ iAAB ii are loc relaţia:

( )( ) 321

321

3214

4321

bbbaaa

BBBAvAAAAv

⋅⋅⋅⋅

= unde am notat ii AAa 4= şi cu .3,1,4 == iBAb ii

Teorema 3. (Dostor- 1967). Volumul V al tetraedrului (A) este dat de formula:

V=hk

ijji

aSS3

sin2 α⋅, {i,j,h,k}= {1,2,3,4}.

Demonstraţie. Cu notaţiile din figura 1, avem:

34

122

34

12113412111

sin2sin22

sina

Sa

PAaPAh ααα =⋅=⋅= de unde, în virtutea definiţiei 2, rezultă:

V=34

122111

3sin2

3 aSShS α

= .

Page 45: RMM 6.pdf

CERCUL DE MATEMATICA

H SSM

- 43 -

Consecinţe:

1. În tetraedrul (A), are loc relaţia:

24321

2314

2314

2413

2413

3412

3412

94

sinsinsinsinsinsin VSSSSaaaaaa

⋅=

⋅⋅

=⋅⋅

=⋅⋅

αααααα (teorema sinusurilor în tetraedru)

Demonstraţie. Din teorema 3, avem:

V=12

3443

34

1221

3sin2

3sin2

aSS

aSS αα

= de unde 1234

341243212

9sinsin4

aaSSSSV

⋅⋅

=αα

din care

obţinem: 24321

3412

3412

94

sinsin VSSSSaa

=⋅⋅

αα.

Teorema cosinusului. Teorema 4 (a proiecţiilor).Într-un tetraedru (A), are loc relaţia

ikkihhijji SSSS ααα coscoscos ++= , {i,j,k,h}= {1,2,3,4}.

Demonstraţie. Dacă 1H este proiecţia lui 1A pe faţa 1S avem: ( ) ( ) ( ) 14413312232

142

143

11 coscoscos ααα SSSAAHsAAHsAAHsS ++=++= .

Definiţia 3 Vectorul notat cu iS , având direcţia şi sensul vectorului ii HA şi modulul iS ,

poartă numele de vector al feţei ( ) 4,1, =iAi . Teorema 5 (a ,,ariciului”). 04321 =+++ SSSS .

Demonstraţie. Notăm jiij AAa = . Avem 131242 aaS ×= , 121432 aaS ×= , 141342 aaS ×= şi

132412 aaS ×= = ( ) ( ) 13121214131412131214 aaaaaaaaaa ×−×−×=−×− .

Din aceste relaţii avem: ( )=+++ 43212 SSSS

013141314131214131214131212141314 =×−×=×+×+×+×−×−×= aaaaaaaaaaaaaaaa Teorema cosinusului În tetraedrul (A) au loc relaţiile: (a) 233213311221

23

22

21

24 cos2cos2cos2 ααα SSSSSSSSSS −−−++=

(Carnot) (b) 3443

24

331221

22

21 cos2cos2 αα SSSSSSSS −+=−+ .

Demonstraţie: (a) Se foloseşte teorema „ariciului” pentru 4S , şi apoi calculăm 44

24 SSS ⋅=

(b) Din teorema „ariciului” avem: ( )4321 SSSS +−=+ apoi ridicăm la pătrat.

Teorema 6 (Schwering). Dacă în tetraedrul (A) notăm cu 2

4321 SSSSS +++= , atunci are

loc relaţia:

( ) ( )( )( )( )2

cos169 34122

432143212

43122 αα ±

−−−−−=± SSSSSSSSSSSSaaaV .

Demonstraţie. Avem conform teoremei 2, 12

3443

34

1221

3sin2

3sin2

aSS

aSSV αα

== .

Page 46: RMM 6.pdf

CERCUL DE MATEMATICA

- 44 -

Tema pentru grupa de performanta la clasa a-XI-a

Asupra teoremei lui Knaster Elev Rapcea Mihai Clasa a XII-a CNT

În acest articol ne propunem să analizăm o problemă de punct fix dată ca teoremă în

lucrarea „Manual pentru grupele de performanţă”, clasă a XI-a. Vom începe prin a da definiţia unui punct fix al funcţiei.

Fie RA ⊆ o mulţime, AAf →: o funcţie. Ax ∈0 se numeşte punct fix al funcţiei dacă

( ) 00 xxf = . Observaţie: o funcţie f are cel puţin un punct fix dacă şi numai dacă graficul funcţiei f

intersectează prima bisectoare. În lucrarea mai sus menţionată, teorema lui Knaster este enunţată în felul următor: Fie ],[],[: babaf → o funcţie monotonă. Atunci există ],[ bac∈ astfel încât f(c)=c. Demonstrăm acest rezultat pentru o funcţie monoton crescătoare. Fie B= })(:{ xxfAx ≥∈ . Cum aaf ≥)( rezulta ca Ba∈ si deci ≠B . Fie c=supB.

Deoarece Bxxc ∈≥ , si f crescatoare, rezulta ca ,),()( Bxxfcf ∈≥ deci ,,)( Bxxcf ∈≥ adica cBcf =≥ sup)( . Atunci ),())(( cfcff ≥ deci cBcfBcf =≤⇒∈ sup)()( . Din relatiile

ccfccf ≤≥ )(,)( rezulta ca .)( ccf = Cu toate acestea, teorema este enuntata gresit deoarece aceasta nu se verifica pentru functii

monoton descrescatoare. In acest sens, oferim un comtraexemplu mai jos folosind metoda grafica.

O functie care ilustreaza contraexemplul de mai sus, data prin lege de asociere este:

⎩⎨⎧

∈+−∈+−

=→]2,1(,3]1,0[,2

)(],2,0[]2,0[:2321

xxxx

xff

Page 47: RMM 6.pdf

CERCUL DE MATEMATICA

H SSM

- 45 -

Tema pentru grupa de performanta la clasa a-XII-a

Caracteristica unui inel prof. Nedeianu Dan,

Grupul Scolar “Domnul Tudor”

În cele ce urmează ne propunem să aprofundăm noţiunea de caracteristică a unui inel (întâlnită în algebra superioară) prin unele probleme propuse la diverse concursuri şcolare. Definiţie Fie (A ,+,∙) un inel, cu elementul unitate A1 şi elementul nul A0 . 1º. Dacă există *Ν∈n cu AAn 01• = (adică

444 3444 21orin de

AAAA 11...11 ++++ = A0 ), atunci cel mai mic n cu

această proprietate se numeşte caracteristica inelului A. 2º. În caz contrar, adică An 01• A ≠ , *Ν∈n , spunem că inelul A are caracteristica zero. Notaţie Dacă inelul A are caracteristica *Ν∈n , notăm char(A)=n. Observaţii: 1) Dacă char(A)=n≥1, atunci n este ordinul elementului A1 în grupul (A,+). 2) Inelul Z are caracteristica zero, căci n∙1≠0, *Ν∈n . 3) Pentru n≥2, inelul nΖ are caracteristica n, căci grupul ( nΖ ,+) este ciclic generat de 1̂ , şi deci

ordinul elementului 1̂ în acest grup este n. Propoziţia 1 Un inel integru are caracteristica zero sau un număr prim. Demonstraţie: Arătăm că în cazul când caracteristica este nenulă, aceasta este un număr prim. Fie deci (A,+,∙) un inel integru cu char(A)=n≥1. Presupunem prin absurd că n nu este prim ⇒ n= 21 nn ⋅ cu 1n , Ν∈2n şi 1< 1n < n, 1< 2n < n; atunci avem că:

AAn 01 =⋅ ,Ù ( )21 nn ⋅ ∙ AA 01 = Ù444 3444 21

ori nn de

AAAA

21

11...11⋅

++++ = A0 Ù(444 3444 21

ori n de

AAAA

1

11...11 ++++ )∙

(444 3444 21

ori n de

AAAA

2

11...11 ++++ )= A0 Ù ( 1n ∙ A1 )∙ ( 2n ∙ A1 )= A0 . Cum A este inel integru (adică nu are divizori

ai lui zero), deducem că ( 1n ∙ A1 )= A0 sau ( 2n ∙ A1 )= A0 ,ceea ce contrazice faptul că *Ν∈n este cel mai mic cu proprietatea n∙ AA 01 = . Consecinţă: Caracteristica unui corp este zero sau un număr prim. Demonstraţie: evidentă, căci un corp este şi inel integru. Propoziţia 2 Fie A un inel comutativ de caracteristică p număr prim. Arătaţi că: a) Funcţia f:AÆA, f(x)= px este un endomofism al inelului A (endomorfismul lui Frobenius) b) Pentru *Ν∈n şi orice Ayx ∈, avem egalitatea: ( ) nnn ppp yxyx +=+ . Demonstraţie: a) Avem f(xy)= ( ) ( ) ( ) Ayxyfxfyxxy ppp ∈⋅== ,, si f(1A)=1A. ( ) ( ) ppp

ppp

pp

ppp yxyCyxCyxCxyxyxf +++++=+=+ −−−−− 1122211 ...

Dar p prim divide pe App

pp

pp

ppppp xyCyxCyxCCCC 0...,...,, 1122211121 ====⇒ −−−−− şi atunci

( ) ( ) ( ) Ayxyfxfyxyxf pp ∈+=+=+ ,, , deci f este endomorfism.

Page 48: RMM 6.pdf

CERCUL DE MATEMATICA

- 46 -

b) Se observă că ( ) ( ) npdef

xxfffxg =⎟⎠⎞

⎜⎝⎛=

orin ...ooo , care este tot endomorfism, şi atunci

( ) ( ) ( ) ,,, Ayxygxgyxg ∈+=+ adică exact relaţia cerută. Propoziţia 3 Fie K un corp comutativ de caracteristică *Ν∈p . Să se arate că există un singur morfism de grupuri de la grupul aditiv (K,+) la grupul multiplicativ ( )⋅,*K Demonstraţie S-a văzut că ( ) Kyxyxyx ppp ∈+=+ ,, (din propoziţia precedentă). Atunci ( ) ( ) pppp yxyx 1−+=− ; pentru p≥3, p fiind prim ⇒ ( ) ppp yxyx −=− , iar pentru p=2 ⇒

( ) 22222 yxyxyx −=+=− (căci Ky 02 2 = ) Cu alte cuvinte am dedus că ( ) Kyxyxyx ppp ∈−=− ,, . Fie f:(K,+) Æ ( ⋅,*K ) un eventual morfism. Arătăm că este cel constant. Într-adevăr, pentru x K∈ , avem p∙x= K0 şi deci K1 =f( K0 )=f(px)=f(x) p ⇒ 0 K =f(x) p - K1 =(f(x)- K1 ) p , de unde f(x)= K1

Kx∈ .

Propoziţia 4 Fie A un inel finit cu cel puţin două elemente. Atunci char(A) divide card(A), card(A) fiind numărul de elemente din A. Demonstraţie Presupunem că card(A)=n≥2, iar char(A)=m≥2, m, n *Ν∈ . Se ştie că notând cu H subgrupul ciclic generat de elementul K1 A∈ (al grupului (A,+)), din teorema lui Lagrange: card(H)|card(A) Dar card(H)=

mchar(A)

=, căci H={ A1 , A1 + A1 , …,

444 3444 21ori 1-m

11...11 AAAA ++++ }, deci m|n.

Propoziţia 5 Dacă (A,+,∙) este inel finit cu p elemente, p prim, atunci A este corp comutativ cu char(A)=p. Demonstraţie: Fie n=char(A), deci elementul A1 ∈A are ordinul n în grupul (A,+); fie H{ A1 , A1 + A1 , …,

444 3444 21ori 1-n

11...11 AAAA ++++ }, subgrupul generat de A1 , în grupul (A,+) atunci card(H)|card(A), deci n|p şi

cum p prim ⇒ n=p, deci char(A)=p ⇒ A=H={ A1 , A1 + A1 , …, 444 3444 21

ori 1-n

11...11 AAAA ++++ }.

Definim f: AZ p → , ( )44 344 21

ori i

1...11ˆAAAif +++= , evident izomorfism de inele ⇒ A corp comutativ.

Propoziţia 6 Fie A inel, AA 10 ≠ , cu proprietatea că grupul elementelor inversabile ale inelului are un număr impar de elemente. Atunci char(A)=2. Demonstraţie: Dacă (U(A,∙)) este grupul elementelor inversabile din A, din enunţ card(U(A))=2n+1, n *Ν∈ ; avem că - A1 ( )AU∈ ⇒ ( ) 11 12 =− +n

A Ù - A1 = A1 Ù A1 + A1 =0 ⇒ char(A)=2. S-a folosit că într-un grup(G,∙) finit cu n elemente şi e element neutru, avem că Gxexn ∈= , . Propoziţia 7 Fie A inel cu propietatea că ( ) *Nn∈∃ fixat astfel ca Axxx n ∈= ,2 . Atunci char(A)=2. Demonstraţie:

Page 49: RMM 6.pdf

CERCUL DE MATEMATICA

H SSM

- 47 -

Avem (- A1 ) n2 = - A1 ⇒ A1 = - A1 ⇒ A1 + A1 = 0 ⇒ char(A)=2. Observaţie: În particular orice inel boolean (inel A în care x 2 = x, Ax∈ ) este de caracteristică 2. Mai mult, un inel boolean este şi comutativ, pentru că ( ) yxyx +=+ 2 implică

yxyyxxyx +=+++ 22 , deci xy+yx=0 Ayx ∈ , şi cum xy+xy=0, rezultă xy=yx Ayx ∈ , . În continuare concretizăm noţiunea de caracteristică a unui inel şi proprietăţile sale, prin unele aplicaţii, formulate sub formă de probleme de concurs (olimpiade locale, judeţene, concursuri şcolare interjudeţene, etc.). Exemplul 1 Fie (A,+,∙) inel cu AA 01 ≠ , cu proprietatea că există A∈α unic astfel ca ( )*012

AA =+−αα Să se arate că: a) char A=3 b) Axpxx A

pA

p ∈Ν∈=⇔= ,,00 *2 (OLM Vaslui 2000) Soluţia: a) Avem ( ) ( ) 01111111 222 =+−=++−+−−=+−−− AAAAAAA αααααααα şi din unicitatea (*)⇒ AA 11 =+⇒−= αααα . Dar cum ( ) ( ) ααααααα ⇒−=−=⇒−= AAAAA 11 si 111 2 este element inversabil al inelului cu

αα −=−A11 .

În fine din AAAAAAAAA 01111 adica ,1111(*)

11 =++⇒−=−==+⇒⋅=+ −− αααααα c.c.t.d.

b) „⇒ ” Presupunem 02 =px , *, Ν∈∈ pAx

Avem ( ) ( ) =++++++=+−−−−− Ap

Appp

AAp

Ap

A xxxxxx 111111 22 ( ) ( ) ( ) 0111 2 =++++++= pppp

AAA xxxx pentru că K1 + K1 + K1 = K0 . Din unicitatea (*), avem că A

pA

pA xx 011 =⇒−==−− α

Reciproc „⇐ ” este evident. Exemplul 2 Fie (A,+,∙) inel cu proprietatea că Axxx ∈= ,24 . Să se arate că

444 3444 21ori 12

11...11 AAAA ++++ =0.

(OLM Galaţi 2005)

Soluţie: Avem ( ) ( ) Axxx AA ∈+=+ ,11 24 , deci AA xxxxxx 121464 2234 ++=++++ şi deci Axxxx A ∈=++ ,0264 23 ; pentru x= A1 ⇒ 4∙ A1 +6∙ A1 +2∙ A1 =0, adică

444 3444 21ori 12

11...11 AAAA ++++ = A0 .

(amintim că ;4 33333 xxxxx +++= xxxxxxxxxx +=+++++= 2;6 2222222 etc)

Exemplul 3 Fie (A,+,∙) un inel în care A1 + A1 este inversabil. Demonstraţi că cel puţin una dintre următoarele afirmaţii este adevărată: a) char A= *Ν∈n b) A conţine o infinitate de elemente inversabile

(OLM Argeş 2000) Soluţie: Presupunem că a) este falsă şi arătăm că b) este adevărată, arătând inductiv (după *Ν∈n ) că A

n 12 ⋅ este element inversabil. Pentru n=1 avem A1 + A1 = 12 ∙ A1 este inversabil ⇒ ( ) Au∈∃ a.î. ( A1 + A1 )u= A1 , adică u+u= A1 (*)

Page 50: RMM 6.pdf

CERCUL DE MATEMATICA

- 48 -

Presupunem că pentru *Ν∈k , elementul k2 ∙ A1 este inversabil şi arătăm că elementul 12 +k ∙ A1 este inversabil: Fie Ax∈ astfel că ( k2 ∙ A1 )∙x=x∙( k2 ∙ A1 )= A1 . Atunci ( k2 ∙x)= (x∙ k2 )=1 A (**)

Şi deci k2 ∙x∙u=u; adun u ⇒ k2 ∙x∙u+u=u+u(*)

= A1 Ù ( k2 -1)x∙u+xu+u= A1 ⇒ ( k2 -

1)(xu)+(x+ A1 )∙u= A1 (**)

⇒ ( k2 -1)x∙u+(x+ k2 x)u= A1 ⇒ 12 +k ∙x∙u= A1 ⇒ ( 12 +k ∙ A1 )∙(xu)= A1 . Analog se arată că (x∙u)∙ ( 12 +k ∙u)= A1 , deci 12 +k ∙ A1 este element inversabil al lui A. Deci am arătat că n2 ∙ A1 este element inversabil al lui A, *Ν∈n şi cum a) este falsă ⇒ m2 ∙ A1 ≠2 n ∙ A1 , *, Ν∈ nm , m≠n, deci mulţimea U={2 n ∙ A1 | *Ν∈n } are toate elementele distincte două câte două şi conţine o infinitate de elemente, toate inversabile.

Exemplu 4 Dacă (A,+,∙) inel de caracterisrică zero şi x,y,z ∈A, cu proprietatea că

Azyxzzyyxx 0,,, 222 =++=== , să se arată că x=y=z= A0 . (IMC 2000)

Soluţie Avem x+y=-z ⇒ ( ) yxyyxxyxyxzzyx −−=+++−−===+ 2222 deci , şi astfel

=+++ yxxyyx 22 A0 ; înmulţesc cu x la stânga: =+++ xyxyxxyx 22 22 A0 ⇒ =++ xyxxyx 32 A0 (1).

Înmulţind cu x la dreapta ⇒ =++ 22 32 xyxxyxx A0 , adică =+ xyxx 42 A0 (2). Din (1) şi (2) ⇒ xy=xyx şi folosind (2) ⇒ 2x+4xy= A0 . char A=0 ⇒ x+2xy= A0 ⇒ 2xy=-x Atunci 4xyxy= 2x =x; dar 4xyxy=4xyy=4xy=-2x, deci x=-2x ⇒ x+x+x= A0 ⇒

( A1 + A1 + A1 )∙x= A0 0CharA=

⇒ x= A0 . Analog avem şi că y=z= A0 . Se observă că este suficient ca char A { }3,2∉ . Exemplu 5 Fie K un corp de caracteristică p, cu )4(mod1≡p a) Demonstraţi că - K1 este pătratul unui element din K b) Arătaţi că orice element nenul din K se scrie ca suma a trei pătrate nenule de elemente din K. c) Rămâne, în general, această scriere adevărată pentru elementul nul din K ? Soluţie a) Fie m *Ν∈ , p=4m+1 Din teorema Wilson, p|(p-1)!+1, deci (p-1)!+ K1 = K0 în K, deci (4m)!=- K1 în K. Apoi K1 =-4m, 2=-(4m- K1 ), ..., 2m=-(2m+ K1 ) în K ⇒ înmulţite:

A1 ∙2∙3∙...∙(2m)=(4m)(4m- K1 )...(2m+ K1 ) şi înmulţim cu membrul drept ⇒ (4m)!=[(4m)(4m- K1 )…(2m+ K1 )] 2 , adică - K1 =[(4m)(4m- K1 )…(2m+ K1 )] 2 , deci - K1 este pătrat perfect în K b) Fie b=2 1− , deci ( K1 + K1 )b= K1 , b+b= K1 (*) şi avem: b≠-b; atunci a şi b comută căci din (*) înmulţind cu a la stânga, respectiv la dreapta avem că ab+ab=a=ba+ba Ù ( K1 + K1 )ab=( K1 + K1 )ba ⇒ ab=ba, *Ka∈ variabil şi 2ab=a. Dacă a≠-b⇒a+b≠ K0 şi a=2ab=(a+b) 2 -a 2 -b 2 =(a+b) 2 +a1

2 +b 12 , căci- K1 este pătrat perfect în K.

Dacă a=-b ⇒ a≠b şi a=2ab+b 2 -(a-b) 2 =a 2 +b 2 +c 2 , căci - K1 este pătrat perfect în K. c) În Z 5 , 0̂ nu se poate scrie ca suma a trei pătrate perfecte nenule.

Page 51: RMM 6.pdf

CERCUL DE MATEMATICA

H SSM

- 49 -

În încheiere, expunem cititorului un set de probleme propuse (PP) legat de tema acestui articol. PP 1) Fie (A,+,∙) inel cu 6 elemente şi A={n AAAAN 01...11

orin de

* =+++∈44 344 21

}

a) Să se arate că 6min =∈

nAn

b) A este un inel comutativ cu divizori ai lui zero. (OLM Prahova 2003)

PP 2) Spunem că inelul A are proprietatea P dacă pentru orice Ax∈ , există un Ν∈xn impar, cu proprietatea { }AA

nxx 1,0∈ a) Să se arate că un inel cu proprietatea P este corp. b) Să se arate că orice corp finit în care A1 + A1 = A0 are proprietatea P. c) Să se dea exemplu de corp infinit în care A1 + A1 = A0 şi care nu are proprietatea P.

(Concurs “Gh. Lazăr”, 2003)

PP 3) Fie (A,+,∙) inel, ca astfel 2,2,, , ≥≥Ν∈∈ knknAa44 344 21

orin de

1...11 AAA +++ =0, Ak aa 1+= .

Să se arate că: a) oricare ar fi *NS ∈ , există Ν∈+110 ,...,, kPPP astfel că:

1110 ... −−+++= k

kS aPaPPa

b) există *Nm∈ astfel încât Ama 1=

(OJM 2002) Bibliografie [1] M.Ţena – Manual pentru clasa a XII-a, Editura Gil Zalău 2003 [2] C.Năstăsescu, C.Nită ş.a. – Manual pentru clasa a XII-a, E.D.P. Bucureşti 2002 [3] M.Antronache, M.Ţena ş.a. – Olimpiadele de matematică 2003 (cls. XI-XII) – Editura Gil 2003 [4] A.Izmail, G.Kreindler ş.a. – Olimpiadele de matematică 2004 (cls. XI-XII) – Editura Gil 2004. [5] D.Brânzei, C.Stoica ş.a. – Matematică în concursurile şcolare, Ed. Paralela 45, Piteşti 2000. Tema pentru grupa de performanta la clasa a-XII-a

TEOREMELE LUI SYLOW

prof. Dana Paponiu CNT

Fie G un grup finit. Din teorema lui Lagrange, ordinul unui subgrup al lui G divide ordinul grupului G. Reciproca teoremei lui Lagrange este falsa, adica daca n este un divizor al lui G nu rezulta neaparat ca G are un subgrup de ordin n. Teoremele lui Sylow permit sa se stabileasca existenta subgrupurilor de ordin n daca n este o putere a unui numar prim si dau informatii importante despre aceste subgrupuri. Fie p un numar natural prim. Daca rpG m= ,unde m si r sunt numere naturale nenule si

p nu divide r , vom numi p-subgrup Sylow al lui G orice subgrup de ordin p m al sau. T1. Pentru orice grup finit G si orice numar prim p ce divide G exista un p-subgrup Sylow al lui G.

Page 52: RMM 6.pdf

CERCUL DE MATEMATICA

- 50 -

T2. Fie G un grup finit si p un numar prim ce divide G . Daca H este un p-subgrup Sylow

al lui G, iar J un p-subgrup al lui G, atunci exista Gg ∈ astfel incat HggJ 1−≤ . T3. Fie pn numarul p-subgrupurilor Sylow distincte ale unui grup finit G. Atunci

)(: HNGn Gp = , unde H este un p-subgrup Sylow particular al lui G, pn divide HG : si )(mod1 pn p ≡ .

Aplicatii. A1. Fie G un grup de 6009 elemente. Aratati ca G are un singur subgrup de ordin 2003.(V. Vornicu, lista scurta O.N.M. 2003) Solutie.Teorema 3 a lui Sylow ne spune ca numarul 2003-subgrupurilor lui G este congruent cu 1 (mod 2003), deci el nu poate fi decat 1.

A2. Fie p un numar prim, G un p-grup cu ∈= mpG m , N. Sa se demonstreze ca pentru

fiecare { }mi ,...,2,1,0∈ exista un subgrup normal iG al lui G astfel incat ii pG =

si GGGG m =<<<= ...1 10 .

A3. Fie p un numar prim, iar G un grup finit astfel incat ∈mGpm , N. Sa se demonstreze

ca G are un subgrup de ordin mp . Solutie. Fie H un p-subgrup Sylow al lui G cu tpH = ; atunci tm ≤ si exista (conform A2) HJ ≤

astfel incat mpJ = . Ca urmare a acestui exercitiu putem trage concluzia ca pentru p-grupurile finite este adevarata reciproca teoremei lui Lagrange.

A4. Fie G un grup finit, p un divisor al lui G , iar H un p-subgrup Sylow al lui G.

Sa se demonstreze ca daca pH = , atunci numarul elementelor de ordin p din G este egal cu )1( −pn p .

A5. Aratati ca orice grup de ordin 255 este ciclic. Solutie. Fie G astfel incat ,pqG = P un p-subgrup Sylow al lui G si Q un q-subgrup Sylow al lui

G. Deoarece pP = si qQ = , deducem ca P si Q sunt ciclice, adica >=<>=< yQxP , cu

ord(x)=p si ord(y)=q. Conform teoremei lui Lagrange, deducem ca 1=∩QP . Avem qn p si )(mod1 pn p ≡ . Deoarece )(mod1 pq ≡ rezulta ca 1=pn si GP ≤ ; analog GQ ≤ . Din GP ≤

obtinem ca Pyyxxyxyx ∈= −−−− )( 1111 si din GQ ≤ obtinem ca Qyxyx ∈−− 11 )( , deci QPyxyx ∩∈−− 11 . Rezulta succesiv ca yxxy = ,

pqxyord =)( , >=< xyG , G este grup ciclic.

Bibliografie: [1] Gh.Vraciu Elemente de teoria grupurilor finite [2] D.Busneag Probleme de algebra

Page 53: RMM 6.pdf

CERCUL DE MATEMATICA

H SSM

- 51 -

Tema pentru grupa de performanta la clasa a-XII-a

Asupra polinomului caracteristic

prof. Daniel Sitaru, CN Economic „Th. Costescu”,

Fie V o mulţime nevidă, ale cărei elemente le vom nota cu cba ,, …. şi (K,+, *) un corp comutativ cu elementele notate α, β, γ, …….. (exceptând elementul nul şi elementul neutru pe care le vom nota cu 0, respectiv 1). Presupunem că pe V este definită relaţia de egalitate a elementelor sale. Definiţia 1 Spunem că mulţimea V are structură de spaţiu vectorial (liniar) peste corpul K dacă este înzestrată cu două legi de compoziţie: I ) O lege de compoziţie internă, + : V x V → V, numită adunare, în raport cu care V este grup II ) O lege de compoziţie externă * : K x V → V, numită înmulţire cu scalari, care verifică axiomele:

1. (α β) a = α (β a ) 2. α ( a + b ) = α a + α b 3. (α + β) a = α a + β a 4. 1* a = a , oricare ar fi a , b ∈ V şi α ,β ∈ K. Elementele unui spaţiu vectorial se numesc vectori; elementele corpului K se numesc scalari Elementul neutru al grupului (V, +) se numeşte vectorul nul (şi îl notăm cu 0 ) al spaţiului

vectorial V. Fie V,W două spaţii vectoriale peste acelaşi corp K. Definţia 2 Aplicaţia f: V →W se numeşte aplicaţie liniară (homomorfism liniar, operator

liniar) dacă: a) f este aditivă, adică f( x + y ) = f ( x ) + f ( y ), oricare ar fi x , y ∈ V. b) f este omogenă, adică f (α x ) = α f( x ), oricare ar fi α ∈ K ; x ∈ V. Dacă W = V spunem că f este un endomorfism liniar sau operator liniar al spaţiului vectorial V. Notăm End(V) mulţimea endomorfismelor spaţiului vectorial V. Definiţia 3 Vectorul nenul x ∈ V se numeşte vector propriu al operatorului liniar f dacă

există un scalar λ ∈ K astfel încât f ( x ) = λ x . Un scalar λ ∈ K se numeşte valoare proprie a operatorului liniar f dacă există un vector x ∈ V \ { 0 } pentru care f ( x ) = λ x .

Proprietatea 1 a) Dacă x ∈V\{ 0 } este vector propriu al lui f, atunci există un unic scalar λ∈K pentru care

f( x )=λ x . Cu alte cuvinte, oricărui vector propriu îi corespunde o singură valoare proprie b) Oricărei valori proprii îi corespund o infinitate de vectori proprii.

Definiţia 4 Fie B1 = { 1a , 2a , …., na } şi B2 = { 1b , 2b , … , nb } baze în spaţiile vectoriale

V, respectiv W şi f: V → W o aplicaţie liniară. Scriind vectorii f ( 1a ); f ( 2a );…; f ( na ) în raport cu baza B2 avem:

f ( ia ) = ai1

1b + ai2 2b + … + ai

m mb ; i = 1,2, …, n. Matricea A ale cărei coloane sunt coordonatele imaginilor prin f ale vectorilor bazei B1 în

raport cu baza B2 se numeşte matricea aplicaţiei liniare f în raport cu bazele B1 şi B2. Vom avea:

Page 54: RMM 6.pdf

CERCUL DE MATEMATICA

- 52 -

A = ⎟⎟⎟⎟

⎜⎜⎜⎜

mn

mm

n

n

aaa

aaaaaa

21

222

21

112

11

Definiţia 5 Fie B = { 1a , 2a , …., na } o bază a spaţiului vectorial V şi f ∈ End (V). Atunci

oricare ar fi i = 1, 2, …, n avem f ( ia ) = aik ka .

Prin matricea operatorului liniar f în raport cu baza B vom înţelege matricea:

A = ⎟⎟⎟⎟

⎜⎜⎜⎜

nn

nn

n

n

aaa

aaaaaa

21

222

21

112

11

Proprietatea 2 Fie A ∈ Mn(k) matricea operatorului liniar f ∈ End (V) în raport cu baza B = { 1a , 2a , …., na }a lui V. Au loc următoarele afirmaţii:

a) Un scalar λ ∈ K este valoarea proprie a lui f dacă şi numai dacă det ( A – λ In ) = 0 . b) Un vector 0x = x0

i ia ∈ V este vector propriu al lui f, corespunzător valorii proprii λ, dacă şi numai dacă vectorul coloană 0

~x = ( x01, x0

2, …, x0n)t format cu coordonatele sale,

este o soluţie nenulă a sistemului liniar şi omogen : (A - λ In) x~ = 0~

Proprietatea 3 Fie f ∈ End (V) şi A, B matricile lui f în raport cu bazele B = { 1a , 2a , ….,

na }, respective B1 = { 1b , 2b , … , nb } ale lui V. Atunci det (A - λ In) = det (B - λ In). Definiţia 6 Polinomul Pf (λ) = det (A - λ In) se numeşte polinom caracteristic al operatorului

liniar f. Ecuaţia Pf (λ) = 0 se numeşte ecuaţia caracteristică a lui f. Proprietatea 4 Un scalar λ ∈ K este valoarea proprie a lui f dacă şi numai dacă este o

rădăcină (în K) a ecuaţiei caracteristice Pf (λ) = 0. Practic, pentru aflarea valorilor proprii şi a vectorilor proprii pentru un operator liniar

f∈ End (V), cu V finit dimensional, se procedează astfel: 1) Se fixează o bază B = { 1a , 2a , …., na } în V 2) Se scrie matricea lui f în raport cu baza B. 3) Se calculează polinomul caracteristic Pf (λ) = det (A - λ In) 4) Se rezolvă în K ecuaţia caracteristică Pf (λ) = 0, adică se determină valorile proprii λ1,

λ2, …, λm; m≤ n (egalitate pentru K = C). 5) Pentru fiecare valoare proprie λi, i = 1, 2, …, m se determină vectorii proprii, adică

vectorii nenuli ix prin rezolvarea sistemului liniar şi omogen (A – λi In) x~ = 0~ Aplicaţia 1 Fie f∈End (R2) care în raport cu baza canonică 1e = (1,0); 2e = (0,1) a lui R2 este definit

prin: f ( x ) = ( x1 – 2 x2, 2 x1 – 4 x2), pentru x = (x1,x2) ∈ R2. Se cer valoriile proprii şi vectorii proprii pentru f.

Soluţie: f ( 1e ) = (1,2); f ( 2e ) = (-2,-4). Matricea lui f în raport cu baza { 1e , 2e }este:

A = ⎟⎠

⎞⎜⎝

⎛−−

4221

Page 55: RMM 6.pdf

CERCUL DE MATEMATICA

H SSM

- 53 -

Polinomul caracteristic al lui f este :

Pf (λ) = det ⎟⎟⎠

⎞⎜⎜⎝

⎛−−

−−λ

λ42

21 = λ2 + 3 λ

Ecuaţia caracteristică este λ2 + 3 λ = 0. Valorile proprii sunt soluţiile acestei ecuaţii: λ1 = -3; λ2 = 0. Determinarea unui vector propriu pentru λ1 = -3;

⎟⎟⎠

⎞⎜⎜⎝

⎛−−

1224

⎟⎟⎠

⎞⎜⎜⎝

⎛2

1

xx = ⎟⎟

⎞⎜⎜⎝

⎛00

, adică: 2x1 – x2 = 0.

Alegem x1 = α – necunoscută secundară şi rezultă x2 = 2α; α ∈ R. Un vector propriu este 1x =(1,2). Orice alt vector propriu este de forma x = α 1x ; α ∈R*. Analog, pentru λ2 = 0 avem 2x =(2,1). Orice alt vector propriu este de forma x = α 2x ; α ∈R*.

Aplicaţia 2 Fie f∈End (R3) care în raport cu baza canonică din R3 are matricea:

A = ⎟⎟⎟

⎜⎜⎜

− 110110001

. Se cer valorile şi vectorii proprii.

Soluţie: Singura rădăcină reală a polinomului caractristic:

Pf (λ) = λ

λλ

−−−

110110001

=( 1 – λ )[(1 – λ)2 +1] este λ = 1. Rezultă că f are valoarea proprie λ=1.

Formăm sistemul ⎟⎟⎟

⎜⎜⎜

− 010100000

⎟⎟⎟

⎜⎜⎜

3

2

1

xxx

= ⎟⎟⎟

⎜⎜⎜

000

.

Se observă că x1 este necunoscută secundară, iar x2 = x3 = 0. Deci, un vector propriu ataşat valorii proprii λ = 1 este a = (1,0,0), ceilalţi vectori proprii fiind de forma x = α a ; α ∈R*. Aplicaţia 3 Fie V un spaţiu vectorial peste R şi f∈End (V) care în raport cu baza B = { 1a , 2a } a lui V

are matricea A = ⎟⎟⎠

⎞⎜⎜⎝

⎛ −2452

Întrucît polinomul caracteristic Pf (λ) = λ

λ−−−

2452

= (2- λ)2 +20 nu are rădăcini în R

rezultă că f nu are valori proprii şi vectori proprii. Aplicaţia 4 Fie V un spaţiu vectorial peste R şi f∈ End (V) care în raport cu baza B = { 1a , 2a , 3a , 4a }

are matricea A =

⎟⎟⎟⎟⎟

⎜⎜⎜⎜⎜

−−−

−−

2112101224101201

.

Se cer valorile şi vectorii proprii.

Page 56: RMM 6.pdf

CERCUL DE MATEMATICA

- 54 -

Soluţie: Polinomul caracteristic al lui f este : Pf (λ) =

λλ

λλ

−−−−−

−−−−

211211224101201

= (λ-1)4

Rezultă că f admite valoarea proprie λ = 1, multiplă de ordinul 4. Sistemul care dă coordonatele x1, x2, x3, x4 ale vectorilor proprii ( în raport cu baza B) ataşaţi valorii proprii λ = 1 este:

(S) ⎩⎨⎧

=+−−=−

0202

4321

43

xxxxxx

. Alegând x1, x2 ca necunoscute secundare şi x3, x4 principale,

obţimem:

⎪⎪⎩

⎪⎪⎨

+−=+−=

=

21

213

22

1

2442

xxxxxx

xxx

cu x1,x2 ∈R.

Prin urmare, un sistem fundamental de soluţii ale sistemului (S) este: 1a = (1,0,-2,-4);

2a =(0,1,1,2). Vectorii 1a , 2a sunt vectori proprii ai lui f, ceilalţi vectori proprii fiind de forma a =α 1a + β 2a , unde α, β ∈R încât a ≠ 0 .

Teorema Cayley – Hamilton Fie Pf (λ) polinomul caracteristic al lui f∈ End (V). Dacă în raport cu o bază oarecare f are matrice A ∈Mn (C) atunci Pf (An) = 0n Aplicaţia 5:

Fie A = ⎟⎟⎠

⎞⎜⎜⎝

⎛dcba

matricea unei aplicaţii f∈End (V).

Pf (λ) = λ

λ−

−dc

ba = (a- λ)(d- λ)-bc = ad - aλ - d λ + λ2 – bc = ad – bc - (a+d) λ + λ2

Pf (A) = 02 ⇒ A2 – (a+d) A + (ad-bc) I2 = 02 ⇒A2 – TrA A + detA I2 = 02 Aplicaţia 6

Fie A = ⎟⎟⎟

⎜⎜⎜

ihgfedcba

matricea unei aplicaţii f∈End (V).

Pf (λ) = (a – λ) (e- λ)(i- λ) + bfg + cdh – cg(e- λ) – hf(a- λ) – bd(i- λ) = - λ3 + λ2(a+e+i) – λ(ae+ai+ei-cg-hf-bd) + aei + bfg + cdh – cge –ahf –bdi.

Pf (λ) = - λ3 + λ2 TrA – λ (det A1 + det A2 + det A3) + det A unde det A1 = edba

; det A2=

ihfe

; det A3 = igca

;

Pf (A) = 03, de unde: A3 – Tr A A2 + (det A1 + det A2 + det A3) A – det A I3 = 03

Bibliografie: [1] I.Vladimirescu – „Note de curs” – Reprografia Univ. din Craiova 1987 [2] I.R. Şafavevici – „Noţiunile fundamentale ale algebrei” – Ed. Academiei 1989

Page 57: RMM 6.pdf

CERCUL DE MATEMATICA

H SSM

- 55 -

mATh Studio

...sau despre limitările matematicii făcute pe hârtie Bocşe Bogdan elev clasa a XII-a CNT

A folosi un calculator în rezolvarea unor probleme de matematică este de multe ori

considerat un demers nenecesar, care poate împiedica o învăţare corectă a materiei. Când problemele ce apar în domeniile mai avansate ale matematicii se complică, folosirea unei maşini de calcul devine inevitabilă. Există multe cazuri, mai ales în simularea unor procese reale, când o soluţie analitică, exactă nu mai poate fi obţinută; drept urmare, e nevoie de o soluţie aproximativă, numerică.

Acest proiect al meu, mATh Studio, este un set de instrumente ce permite rezolvarea numerică a unor astfel de probleme care nu admit soluţii exacte. Printre acestea se pot enumera integrări multiple şi derivări parţiale, convoluţii, estimări de serii, rezolvări numerice de ecuaţii diferenţiale şi de ecuaţii diferenţiale cu derivate parţiale, transformări Fourier rapide, analiza (statistică, diferenţială) a seturilor de date.

Dat fiind faptul că majoritatea conceptelor aici

prezentate depăşesc programa de liceu, mă voi limita la a arăta aplicaţiile lor practice.

1. Propagarea oscilaţiilor unui fir semi-elastic,

considerând forma iniţială dată, pierderile de energie şi oscilaţii definite la capete (figura 1).

2. Propagarea undelor unei membrane semi-elastice,

considerând forma iniţială, pierderile de energie şi oscilaţii suplimentare definite la margini.

3. Studiul evoluţiei sezoniere a populaţiilor dintr-un

ecosistem pe baza relaţiilor pradă-prădător, folosind ecuaţiile Lotka-Volterra (figura 2).

4. Studiul gradului de fericire al individului într-o serie

de circumstanţe definite, prin măsura în care sunt favorabile sau nefavorabile. (figura 3)

5. Studiul unor probleme din cinematică, cum ar fi

aruncarea pe oblică la viteze foarte mari, considerând rezistenţa la înaintare proporţională cu pătratul sau cu cubul vitezei.

Figura 1

Figura 2

Figura 3

Page 58: RMM 6.pdf

CERCUL DE MATEMATICA

- 56 -

De asemenea, mATh Studio pune la dispoziţia utilizatorului numeroase module de vizualizare pentru diferite tipuri de seturi de data: curbe explicite şi parametrice (în plan sau în spaţiu), suprafeţe explicite sau parametrice (figura 4) şi diagrame de variaţie. Toate modulele de vizualizare pot fi folosite atât pentru seturi de date statice sau animate (care conţin un parametru suplimentar, pentru timp).

O facilitate importantă a programului este că permite relaţionarea seturilor de date. Mai exact, datele de ieşire obţinute folosind unul dintre modulele de calcul pot fi folosite ca date de intrare pentru alt modul. Astfel se pot defini întregi scheme de calcul, care pot fi salvate, editate şi folosite ulterior.

În concluzie, mATh Studio, e un utilitar ce permite experimentarea cu diverse concepte matematice ce nu pot fi dezvoltate în absenţa unui sistem de calcul. Mai mult, programul permite aplicarea acestor concepte în diverse domenii, de la fizică şi astronomie, până la sociologie şi epidemiologie.

Din partea redactiei

Lucrarea completă a fost prezentată în cadrul CONCURSULUI NATIONAL „INFOEDUCATIE” - Gălăciuc 2006 şi a obţinut Premiul I.

Figura 4

Page 59: RMM 6.pdf

PROBLEME PROPUSE

- 56 -

Elevii vor rezolva probleme de la clasa pe care o urmează şi de la clasa inferioară. Se pot rezolva şi problemele date la Ediţia a II-a a concursului „Petre Sergescu” - pagina 9. Soluţiile redactate pe foi format A5 se vor preda profesorului îndrumător.

IV.1. Aflaţi numerele naturale a, b, c care îndeplinesc simultan condiţiile:

a) al doilea număr este de trei ori mai mare decât primul, b) al treilea număr este jumătatea celui de-al doilea, c) suma numerelor este mai mare decât 7 şi cel mult egală cu 44.

inst. Maria Ungureanu IV.2. Vlad are un număr de bile. El observă că le poate grupa câte 6 dar şi câte 10,

iar diferenţa dintre grămezi este 2. Câte bile are Vlad? inst. Maria Ungureanu

IV.3. Două veveriţe consumă rezerva de alune în 6 luni. În cât timp vor termina

rezerva trei veveriţe? inst. Maria Ungureanu

IV.4. Radu are de 7 ori mai multe timbre decât colegul său. Dacă ar fi avut cu 48 de

timbre mai puţine, iar colegul său cu 36 de timbre mai multe, atunci Radu ar fi

avut de trei ori mai multe timbre decât colegul său. Câte timbre are fiecare? inst. Maria Ungureanu

IV.5. La concursul „Cangurul” pentru fiecare din cele 20 de probleme propuse, rezolvată corect, se acordă 10 puncte, iar pentru fiecare problemă nerezolvată se

scad 5 puncte. Ana a obţinut 125 puncte. Câte probleme a rezolvat corect ? inst. Niculina Opriţa

IV.6. Aflaţi numerele naturale a, b, c care îndeplinesc simultan condiţiile: a) a + c = 20, b) a – b = 5, c) b – c = 5.

inst. Niculina Opriţa IV.7. Diferenţa a două numere este 226 iar primul mărit cu 2 este de 8 ori mai mic

decât al doilea. Care sunt numerele? inst. Niculina Opriţa

IV.8. Răsturnatul semisumei a două numere este 24, iar a patra parte din primul

este egală cu a treia parte din al doilea. Care sunt numerele? inst. Mariana Cornea

IV.9. Tatăl are 46 de ani iar fiul are 19 ani. Cu câţi ani în urmă tatăl era de patru ori

mai în vârstă decât fiul ? inst. Mariana Cornea

IV.10. Produsul vârstelor a trei fraţi este 72, iar suma vârstelor este mai mică decât

15. Câţi ani are fiecare, ştiind că nu sumt gemeni ? inst. Mariana Cornea

V.1. Să se afle numerele naturale prime n pentru care numerele n+1, n+3, n+5, n+21 sunt simultan prime.

prof Calafeteanu Gheorghe V.2. Aflaţi cel mai mic nr. natural care are exact 20 divizori.

prof. Calafeteanu Gheorghe V.3. Aflati a,b,c,d,e ∈N care verifică

2 2a b+ + 2 2c d+ + 2e =2007 prof. Victor Săceanu

V.4. Arătaţi ca nu există n∈N astfel încât 4n -3 sa fie pătrat perfect prof. Vctor Săceanu

Page 60: RMM 6.pdf

PROBLEME PROPUSE

H SSM

- 57 -

V.5. Determinaţi a,b,c ∈N ştiind ca verifică relaţia:3 4a b b c+ ++ +6c a+ + 36 prof.Victor Săceanu

V.6. Aratati ca daca m este minimul dintre numerele 31338 si 24446, atunci m > 22007. prof. Alexandru Szoros

V.7. Aflati numerele naturale n ce verifica egalitatea 2n + n5 = n2 + 5n . prof. Alexandru Szoros

V.8. Sã se afle restul împãrţirii numãrului n=20052006 + 2007 2006 +2004 la 2006 prof. Marica Ştefan

propusă OJ V.9. Împãrţind n numere naturale consecutive la n obţinem resturi a cãror sumã

este 55. a) Sã se afle n b) Sã se afle cele n numere naturale ştiind cã suma dintre cel mai

mic si cel mai mare este 230 prof. Marica Ştefan

propusă OJ V.10. Pe un raft al unei biblioteci sunt asezate in ordine 10 carti, astfel ca numarul

de pagini pentru doua carti ‘vecine’(alaturate) sa difere cu 1. Pot fi in total, pe raft 2006 pagini? Dar 2005?

prof. Dan Nedeianu propusă OJ

V.11. Se consideră mulţimea M={n /Ν∈ n }2006≤ a) Stabiliţi dacă se poate scrie M ca reuniune de submulţimi disjuncte Mk astfe încât în fiecare mulţime Mk să existe un element egal cu suma celorlalte. b) Să se demonstreze că din oricare 1005 elemente din M se pot alege două elemente diferite astfel încât suma lor să fie 2006.

prof. Dana Paponiu propusă OJ

VI.1. Arătaţi că fracţia : 17572

2

2

++

nn

este ireductibilă. prof. Calafeteanu Gheorghe

VI.2. Să se arate că oricare ar fi 5 numere bipătrate perfecte (adică egale cu puterea

a 4 a unor numere întregi) există 2 a căror diferenţă se divide cu 10. prof. Calafeteanu Gheorghe

VI.3. Aflaţi măsurile unghiurilor unui triunghi ABC ştiind ca :m(∠A)=[m(∠C)] si m(∠B)=2m(∠C)

prof. Victor Săceanu VI.4. Găsiţi o partiţionare a unei suprafeţe triunghiulare dreptunghice, in 5

suprafeţe triunghiului isoscel nedistincte. prof. Victor Săceanu

VI.5. Fie triunghiul ABC cu m(∠A)=900, AB=1 , AC= 3 , D-mijlocul lui [BC] si F∈(AC)

Arătaţi ca : BF+FD=minimă, daca AC=3⋅AF prof. Victor Săceanu

VI.6. Fie multimile A = {6n+1| n∈N* } si B = {6n-1| n∈N* }. Aratati ca : 1) A ∩ B = � ; 2) x,y∈A ⇒ xy∈A ; 3) x,y∈B ⇒ xy∈A ; 4) Cel putin una din multimile A, B contine o infinitate de numere prime.

prof. Alexandru Szoros

Page 61: RMM 6.pdf

PROBLEME PROPUSE

- 58 -

VI.7. Rezolvati in N ecuaţia 20062006

2007...2

31

2200732 =

+++

++

+ nnn.

prof. Alexandru Szoros VII.1. Rezolvaţi ecuaţia 0

23)1)(2005)(2006(

2 +−−−−

xxxxx

prof. Calafeteanu Gheorghe VII.2. In dreptunghiul ABCD cu BC > BC, fie E∈(AB), F∈(BC), CE∩DF={P}, iar MN

mediatoarea lui [EC], N∈(EF). Daca NC este tangenta cercului circumscris ∆CDP, arătaţi ca DE ⊥EF.

prof. Victor Săceanu VII.3. Rezolvaţi in Z X Z ecuaţia: 3x – y + xy = 2010

prof. Victor Săceanu VII.4. Aflaţi x∈Ndin egalitatea: 3 13 x+ – 2 ⋅ 23 x – 2 ⋅ 3x =2007

prof. Victor Săceanu VII.5. Demonstrati ca exista o infinitate de numere naturale a cu proprietatea ca

multimea M = {7a+3, 7a+4, 7a+5, 7a+6 } contine exact un patrat perfect. prof. Alexandru Szoros

VII.6. Determinati numerele intregi x, y, z ce verifica simultan inegalitatile : x2 +3y +25 < 11z; y2 +5z – 11< 9x; z2 +7z –3<7y.

prof. Alexandru Szoros VII.7. Demonstrati ca pentru a∈Z expresia a6 +4a4 – 5a2 este divizibila cu 36.

prof. Alexandru Szoros VII.8. Sa se afle cifrele x,y,z care satisfac egalitatea

0,x(y1)+0,2y(3z)+0,4z5(yx)=1,430(32). prof. Eleodor Popescu

propusă OJ VII.9. a) Fie ABCD un trapez cu AB| |CD, AB=b<a=CD şi AC∩BD={0}. Dreapta d

trece prin O şi intersectează [CD] şi [AB] în E respectiv F. Notăm CE=x. Să se

determine x pentru care aria [FADE]>aria[ECFB]. b) Paralelogramul MNPQ este decupat din interiorul paralelogramului ABCD. Să se determine o dreaptă care împarte figura rămasă în doua figuri de aceeasi arie.

Justificaţi răspunsul. prof. dr. Gh. Căiniceanu

propusă OJ VII.10. Fie a , b, c ∈Z/ si numerele A = 7a³-b+c , B = 5b³+a+c.

Demonstrati ca A M 6⇔ B M 6. prof. Dan Nedeianu,

propusă OJ VIII.1. Descompuneţi în factori 2a2-a-3.

prof. Calafeteanu Gheorghe VIII.2. Calculaţi maximul expresiei: E(x) = 2005 – 9x2 + 6x.

prof. Calafeteanu Gheorghe

VIII.3. Se dă fracţia 1)23(3

2)33)(23()(

++−++

=xx

xxxF

a) Simplificaţi fracţia

b) Aflaţi pentru ce valori întregi ale lui x fracţia Ζ∈++

1343

xx

c) Calculaţi F(0,(3)) prof. Calafeteanu Gheorghe

Page 62: RMM 6.pdf

PROBLEME PROPUSE

H SSM

- 59 -

VIII.4. Fie a, b > 0 cu 1a

+ 1b

Arătaţi ca : 3

1a

+ 3 3

1a b

+ 3

1b

> 14

prof. Victor Săceanu VIII.5. In triunghiul ascuţitunghic ABC, fie (AA` bisectoarea unghiului A ,

A`∈(BC). Demonstraţi ca I este centrul cercului înscris in triunghiul ABC daca si

numai daca ``

DAEA

=pa

prof. Victor Săceanu VIII.6. Fie triunghiul dreptunghic isoscel ABC cu m(∠A)=900, D∈(AB) , E∈(AC),

DK⊥BE, AL⊥BE, (K, L∈(AC) ) si CL = LK. Se duce MB⊥ (ABC), MB=BC. Aflaţi

m(∠ (MC,DC)) prof. Victor Săceanu

VIII.7. In triunghiul ABC, AM mediana, G∈(AM), GE ║ AB, GF ║ AC, E, F∈(BC). Sa se demonstreze ca G este centrul de greutate al triunghiului ABC dacă

şi numai dacă aria[GEF]=19⋅ [ABC]

prof. Victor Săceanu VIII.8. Rezolvati in R ecuatia (x2 +x +1)3 + (x2 -x +1)3 = 3x4 +3x2 +2.

prof. Alexandru Szoros VIII.9. In trapezul ABCD oarecare cu baza mare CD se dau [AB] =[BC]=a,

m(BCD)=60°, M mijlocul lui [CD], BM∩AD={N}, D∈[NA]. Pe planul trapezului (ABCD) se ridica perpendiculara [VA]=a. Sa se calculeze distanta de la punctul V la dreapta CN.

Prof.Dan Nedeianu propusă OJ

VIII.10. Aratati,ca daca numarul natural x se scrie ca o suma de trei patrate perfecte,atunci nx se scrie si el ca o suma de trei patrate perfecte,oricare ar fi n

∗∈N . prof. Victor Saceanu

propusă OJ IX.1. Demonstrati ca in orice triunghi ABC are loc inegalitatea

142

+⋅≥⎟⎟⎠

⎞⎜⎜⎝

⎛++

rR

rr

rr

rr

a

c

c

b

b

a .

prof. Alexandru Szoros

IX.2. Sa se arate ca daca x1 ,x2,…….,xn ∈ (0,2π ) atunci

sin(x1+x2)∙sin(x2+x3)∙……∙sin(xn-1+xn)∙sin(xn+x1) ≥ ≥ sin2x1∙sin2x2 ∙…………∙sin2xn

prof. Daniel Sitaru propusă OJ

IX.3. Dacă x, y, z ∈ (1,∞), să se demonstreze inegalităţile:

a) 91

11

11

1 222

≥−+−

+−+−

+−+−

zzz

yyy

xxx

b) 6111≥

−+

−+

− zz

yy

xx

prof. Băloi Valeria propusă OJ

Page 63: RMM 6.pdf

PROBLEME PROPUSE

- 60 -

IX.4. Se considera functiile f, g:N*→N* definite prin:f(n)=∑=

n

kk

1)(ϕ si g(n)=[ n ] unde

φ(k) reprezinta numarul divizorilor lui k. Sa se arate ca (-1) )(nf =(-1) )(ng prof. Manuela Praja

propusă OJ IX.5. Fie *, Nba ∈ Arătaţi că ecuaţia 12)()( 22 −=−+− abbxax nu are rădăcini

raţionale. ***

Clasa a-X-a X.1. Aflati a ∈R stiind ca sistemul 2007x +2007-x = y - |x| ; ayx =+++ 14 32

admite o unica solutie reala. prof. Alexandru Szoros

X.2. Fie z1, z2, z3 numere complexe nenule, distincte câte două şi de module egale.

Dacă

3

2123

2

3122

1

3221 ,,

zzzz

zzz

zz

zzz ×

+ sunt numere reale, atunci 43

42

41 zzz ==

sau 1321 =×× zzz . prof. Manuela Prajea

propusă OJ X.3. Daca a, b, c ∈ C* sunt numere complexe distincte doua cate doua astfel incat

(a-b)³=(b-c)³=(c-a)³, sa se arate ca cba −−2 = cab −−2 = bac −−2 . prof. Dan Nedeianu

propusă OJ X.4. Să se rezolve inecuaţia: )23( − x - )257( − x 2≥

prof. Eleodor Popescu propusă OJ

X.5. Demonstraţi inegalitatea 2

11

loglog naxn

kx

n

kka k

≥⋅∑∏==

unde *,,1,1, Nnnkxa k ∈=>

prof. Ovidiu Ticusi propusă OJ

XI.1. Sã se determine funcţiile f:R*+→R continue care satisfac relaţia

*,)(),(1)()( +∈+⎟⎟⎠

⎞⎜⎜⎝

⎛⎟⎠⎞

⎜⎝⎛=+ Ryxxyf

yaf

xafyfxf unde a>0, a≠ 1.

prof. Manuela Prajea XI.2. Fie f : (a,b) →R o functie nemarginita. Sa se arate ca exista un sir ( xn )

convergent cu proprietatea ca (f(xn)) sa fie un sir cu limita ∝. prof. Gheorghe Căiniceanu

XI.3. Sa se arate ca exista o functie bijectiva f : (a,b) →R pentru orice a, b numere reale.

prof. Gheorghe Căiniceanu

XI.4. Se consideră şirul 1n )(x ≥n definit prin x1=21 si xn+1=xn²+xn.

Să se calculeze ⎥⎦

⎤⎢⎣

⎡+

+++

++ 1

1...1

11

1

21 nxxx.

prof. Manuela Prajea

Page 64: RMM 6.pdf

PROBLEME PROPUSE

H SSM

- 61 -

XI.5. Să se arate că ⎭⎬⎫

⎩⎨⎧ >

++

++

+0,,| cba

bac

cab

cba =[

23 ,2)

prof. Manuela Prajea

XI.6. Să se calculeze cos2x ştiind că ∑=

∞→=

n

kn 0

2n 5xsinlim .

prof. Manuela Prajea XII.1. Se consideră polinoamele f=8x³-6x-1, g=4x³-4x şi mulţimea

M=⎭⎬⎫

⎩⎨⎧ ∈ ][|)

9(cos XQhh π

.

a) Să se arate că f(cos9π )=0.

b) Să se arate ca cos9π∉Q.

c) Dacă h∈Q[X] şi h(cos9π

)=0, să se arate că h este divizibil cu f.

d) Să se arate că M este spaţiu vectorial real în raport cu operaţiile uzuale.

prof. Manuela Prajea

XII.2. Se consideră şirul ( ) 0≥nnI definit prin ∫= 40

xdxtgIn

a) Să se arate că In + In-1 = 12

1−n

, () n∈N*.

b) Să se arate că sirul ( ) 0≥nnI este convergent şi să se calculeze nnI

∞→lim .

c) Să se calculeze )12

1)1(...51

311(lim 1

−−+−+− −

∞→ nn

n.

prof. Manuela Prajea

XII.3. Sa se calculeze: ( ).,0,sin2006cos π∈∫ xdx

xx

prof. Dana Paponiu XII.4. Fie RRf →:: funcţie care admite primitive, şi F o primitivă a sa pentru care

avem relaţia Rxx

xxfxF ∈≤+

+ )(,0cos1

2sin)()( 2 . Să se arate că nu există )(lim xFx ∞→

.

prof. Dana Paponiu

XII.5. Fie { }RRR 3 ∈=+−−→= xxfxfxffA ,01)(4)(2)(: 2 . Stabiliti: a) cate elemente are multimea A; b) cate functii din A admit primitive.

prof. Dana Paponiu

Page 65: RMM 6.pdf

Rezultate obtinute de elevii mehedinteni la concursuri de matematica in anul 2006 OIM Ungureanu Andrei Bogdan medalie de aur prof. Prajea Manuela Bazavan Eduard medalie de aur prof. Stretcu Daniel OBM Bazavan Eduard medalie de aur prof. Stretcu Daniel OLIMPIADA NATIONALA Ungureanu Andrei Bogdan premiul I prof Prajea Manuela medalie de aur SSM Bazavan Eduard premiul al III-lea prof. Stretcu Daniel medalie de aur SSM Nicolescu Alexandra mentiune prof. Gimoiu Iuliana medalie de argint Tesila Bianca medalie de bronz SSM prof. Zaman Irina Carapencea Constantin medalie de bronz SSM prof. Paponiu Dana Tigora Andrei medalie de bronz SSM prof. Cainiceanu Gheorghe Pasov Iulia medalie de bronz SSM prof.Popescu Eleodor CONCURSUL NATIONAL AL. MYLLER, Iasi Croitoru Razvan premiul al III-lea prof. Paponiu Dana Carapencea Constantin mentiune prof. Paponiu Dana Pit-Rada Andrei mentiune prof. Paponiu Dana CONCURSUL INTERJUDETEAN GH.TITEICA, Craiova Pit-Rada Andrei mentiune prof.Paponiu Dana Tigora Andrei mentiune prof.Cainiceanu Gheorghe CONCURSUL INTERJUDETEAN ION CIOLAC , Craiova Rapcea Mihai premiul I prof.Paponiu Dana CONCURSUL NATIONAL SCOALA CU CEAS, Rm. Vilcea Proba individuala Sbarcea Alexandra premiul al III-lea prof. Prajea Manuela Gimoiu Ruxandra premiul al III-lea prof. Gimoiu Iuliana Andreescu Madalina premiul al III-lea prof. Antonie Rodica Mitroi Roxana mentiune prof. Cainiceanu Gheorghe Asproniu Robert mentiune prof. Cainiceanu Gheorghe

Page 66: RMM 6.pdf

Nicoara Calin mentiune prof. Paponiu Dana Seitan Mihaela mentiune prof. Paponiu Dana Duta Adrian mentiune prof. Paponiu Dana Proba La ceas Voicu Razvan premiul al IIII-lea prof. Prajea Manuela Grosu Vlad premiul al II-lea prof. Cainiceanu Gheorghe Papa Florin premiul al III-lea prof. Paponiu Dana Nicoara Calin premiul al II-lea prof. Paponiu Dana BARAJUL NATIONAL CANGURUL (clasele VII-XII), Bucuresti Bazavan Eduard premiul I prof.Stretcu Daniel Cepesi Cristian premiul al II-lea prof.Pupaza Ecaterina Carapencea Constantin premiul al II-lea prof.Paponiu Dana Sosu Cristian premiul al II-lea prof.Paponiu Dana Croitoru Razvan premiul al III-lea prof.Paponiu Dana Rosu Stefan premiul al III-lea prof.Paponiu Dana BARAJUL JUDETEAN CANGURUL (clasele V-VI) Clasa a II-a Popescu Maria Sc.nr.3 Popescu Cristiana Lic. I.St.Paulian Vitus Eduard Sc.nr 13 Popescu Simona Sc. Nr 2 Clasa a III-a Prencea Cassian Lic. I.St.Paulian Marghescu Gabriel Nu a mentionat scoala Dirpes Iulian Sc. Nr 2 Marculescu Cristina Sc. Nr 13 Clasa a IV-a Dragomir Andrei Lic. I.St.Paulian Mema Serban Lic. I.St.Paulian Floarea Ionut Strehaia Mihutescu Diana Strehaia Stoichita Andreea Strehaia Ciuta Cora Lic. I.St.Paulian Clasa a V-a Doana Cristina Fara Mentiune de scoala Rapcea Lucian Vinju Mare Baluta Adrian C.N.Traian Stanciu Alexandru Gen.6 Arbanasi Emil Gh.Titeica Nuta Flavius C.N.Traian Nicola Silviu C.N.Traian Borcean Diana Severinesti

Page 67: RMM 6.pdf

Ciontea Stefan Gh.Titeica Stefan Andrei C.N.Traian Clasa a VI-a Grosu Vlad C.N.Traian Clain Andrei Catalin C.N.Traian Rosca Ioana C.N.Traian Pirvulescu Eleodor C.N.Traian Ciocoteala Alexandru C.N.Traian Gevelica Simon C.N.Traian Tudosie Dorin Elevi care au primit diplome din partea American Mathematics Competitions AMC 12,AIME Ungureanu Andrei Bogdan 239p CNT prof. Prajea Manuela Carapencea Constantin 168p CNT prof. Paponiu Dana Pasov Iulia 159,5p CNT prof.Popescu Eleodor Mateisescu Alexandru 159p CNT prof. Giugiuc Leonard Meilescu Simona 159p CNT prof. Paponiu Dana Sfetcu Adina 155,5p CNT prof. Paponiu Dana Rapcea Mihai 155p CNT prof. Paponiu Dana Chilom George 152,5p CNT prof.Popescu Eleodor Turturea Roxana 150,5p CNT prof. Gimoiu Iuliana Rosu Stefan 150p CNT prof. Paponiu Dana Croitoru Razvan 148p CNT prof. Paponiu Dana Golenteanu Magdalena 141p CNT prof. Prajea Manuela Achimescu Andreea 140p CNT prof. Prajea Manuela Togoe Bogdan 138p CNT prof. Prajea Manuela Bazavan Cristina 135,5p CNT prof. Prajea Manuela Bocse Bogdan 134,5p CNT prof. Paponiu Dana Bejinaru Amalia 122p CNT prof. Paponiu Dana Dagadita Monica 122p CNT prof. Paponiu Dana AMC 10, AIME Tigora Andrei 210p CNT prof. Cainiceanu Gheorghe AMC 8 Carapencea Constantin 21p CNT prof. Paponiu Dana Andreca Mihai 17p CNT prof. Paponiu Dana Pit-Rada Andrei 17p CNT prof. Paponiu Dana Capraru Alexandru 16p CNT prof. Paponiu Dana

Page 68: RMM 6.pdf

Duta Adrian 16p CNT prof. Paponiu Dana Zorocliu Andra 16p CNT prof. Paponiu Dana Agape Mihai 15p CNT prof. Paponiu Dana Grosu Vlad 13p CNT prof. Cainiceanu Gheorghe Asproniu Robert 10p CNT prof. Cainiceanu Gheorghe Vasiu Mihnea 10P CNT prof. Antonie Rodica Prunescu Flavius 10p CNT prof. Cainiceanu Gheorghe Boescu Anca 9p CNT prof. Cainiceanu Gheorghe Lungu Amelin 9p CNT prof. Cainiceanu Gheorghe CALIFICATI LA BARAJUL NATIONAL CANGURUL (clasele VII-XII)

Cepesi Cristian Decebal VII Bobiti Ruxandra Titeica XI Carapencea Constantin CNT VIII Rosu Stefan CNT XI Croitoru Razvan CNT VIII Bazavan Eduard Titeica XII Sosu Cristian CNT VIII Ungureanu Andrei CNT XII Stretcu Otilia Titeica VIII Enescu Flavius CNT XII Cretu Andrei CNT X Chilom George CNT XII Coanda Oana CNT X Priboi Cristian CNT XII

Elevi calificati la Concursul anual al rezolvitorilor Gazetei Matematice, Pitesti , august 2006 Pit-Rada Andrei VIII CNT Raveanu Ioana X CNT

Page 69: RMM 6.pdf

Concursuri

- 62 -

Olimpiada de matematicã - etapa judeţeană 11.03.2006 -

Clasa a V-a

Numele şi prenumele Scoala Pr. Profesor Numele şi prenumele Scoala Pr. Ţîrlui Valeria Ţiţeica I Tătucu Mariana Badea Beatrice CNT M Băcanu Alexandru CNT II Gimoiu Iuliana Nistor Oana Ţiţeica M Pirici Giulia Ţiţeica III Tătucu Mariana Safta Roxana Ţiţeica M Pristoleanu Narcis CNT III Prajea Manuela Tănasie Denisa CNT M Ştefan Andrei CNT M Rapcea Răzvan Lucian Lic.Vj.Mare M Gimoiu Ruxandra CNT M Buşe Ştefan Şc.14 M Săceanu Andrei CNT M Cătănescu Raluca Şc.2 M Sbîrcea Alexandra CNT M Filip Radu CNT M Mitran Dragoş CNT M Hinoveanu Elena CNT M Nicolae Andrei CNT M Negrea Gabriela Ţiţeica M Zamfirescu Simona CNT M Nuţă Flavius Andrei CNT M Răducu Andreea CNT M Păcurariu Adrian Şc.Orşova M Anghel Dumitru Şc.Vj.Mare M Păvălaşc Bogdan Ţiţeica M Viţian Elena Sorina CNT M Arbănaşi Emil Marius Ţiţeica M Florescu Alexandru Ţiţeica M Axinte Dana Şc.2 M Ghiţu Alexandru Ţiţeica M Bîtmălai Adrian Şc.15 M Marghescu Andreea CNT M Calafeteanu Liviu Şc.C.Negreanu M Voicu Răzvan CNT M Doană Cristina Ţiţeica M Drigă Darie CNT M Mohora Eduard Lic.Odobleja M Bobei Paula Şc.14 M Nicola Silviu CNT M Georgescu Ana CNT M Anghel Cosmin Şc.Vj.Mare M Ţolea Cristian Ţiţeica M Clasa a VI-a

Numele şi prenumele Scoala Pr. Profesor Numele şi prenumele Scoala Pr. Frăţilă Remus Şc.2 I Coada Carmen Spîrlea Octavian Şc.14 M Nistor Mihaela Şc.14 II Ionica C-tin Lungu Amelin CNT M Constantinescu Robert Şc.2 III Coada Carmen Nichiţelea Teodor Şc.6 M Guran Maria Mihaela Şc.2 III Coada Carmen Diaconeasa Lucian Şc.2 M Toader Simona Şc.2 III Coada Carmen Pacioagă Florentina Şc.Sergescu M Grosu Vlad CNT III Cainiceanu Gh. Grecu Anda Teodora Ţiţeica M Asproniu Robert CNT M Mucică Alexandru Şc.2 M Drăghia Beatrice Şc.2 M Croitoru Ioana Andra Şc.2 M Zugravu Rozalia CNT M Mavlea Ionona Şc.Vj.Mare M Buţă Alexandra Şc.Strehaia M Rădulescu Silvia Şc.6 M Nişulescu Maria Şc.14 M Andreescu Mădălina CNT M Odor Cristina Şc.2 M Andrei Lavinia Lic.Odobleja M Roşca Ioana CNT M Belcea Manuela CNT M Caragea Flaviu Lic.Paulian M Clain Andrei CNT M Drăghici Răzvan CNT M Drăguşin Costinel Şc.5 M Crac Mihai CNT M Lupiţu Gabriela CNT M Mitroi Roxana CNT M Sârbu Adrian Mihai Ţiţeica M Prunescu Flavius CNT M Ţintoş Mădălina CNT M

Page 70: RMM 6.pdf

Concursuri

- 63 -

H SSM

Clasa a VII-a Numele şi prenumele Scoala Pr. Profesor Numele şi prenumele Scoala Pr.

Teşilă Bianca Ţiţeica I Zaman Irina Papa Florin CNT M Radu Cornel Ţiţeica II Zaman Irina Soare Răzvan Şc.Orşova M Arcuş Ana Maria Şc.14 III Palasca V. Stănişoară Sarah CNT M Mzi Adelina Ţiţeica III Ungureanu O. Ştefan Radu Ţiţeica M Gugulici Alexandra Şc.2 III Coada Carmen Tuce Ovidiu Ţiţeica M Andreca Mihai CNT M Popescu Edward CNT M Caplea Luminiţa CNT M Amza Loredana Ţiţeica M Cepeşi Cristian C.Decebal M Andriuc Adina CNT M Chereşdi Ioan Ţiţeica M Borontiş Elena Şc.2 M Doroiman Costinela Lic.Odobleja M Cilibiu Andrei Şc.11 M Ţuculanu Andreea CNT M Crăc Elena Gr.Şc.Dl.Tudor M Ganţolea Adina Ţiţeica M Minescu Mădălina CNT M Poenaru Alexandra Şc.14 M Pais Amalia Şc.2 M Popa Bogdan Şc.14 M Papa Andreea Mădălina CNT M Boţoacă Mădălina Şc.2 M Păcală Răzvan Ţiţeica M Budu Anca Şc.6 M Tudosie Claudia Ţiţeica M Copăceanu Adela Şc.2 M Zorocliu Andra CNT M Milici Alina Simona Şc.5 M Clasa a VIII-a

Numele şi prenumele Scoala Pr. Profesor Numele şi prenumele Scoala Pr. Stretcu Otilia Ţiţeica I Stretcu Dan Marin Andreca Cristina Ţiţeica M Carapencea Constantin CNT II Paponiu Dana Huza Alexandru CNT M Croitoru Răzvan CNT II Paponiu Dana Nef Nicoleta Şc.2 M Piţ Andrei CNT III Paponiu Dana Rădoi Ana Maria Ţiţeica M Calinovici Paul CNT M Răduţ Marian Şc.2 M Ecobici Gabriela CNT M Brăgaru Evelina Ţiţeica M Nicoară Călin CNT M Ciuciu Lucian Florin Ţiţeica M Duţă Adrian CNT M Crîng Larisa Lic.Odobleja M Orbu Alexandru Ţiţeica M Durac Raluca Şc.2 M Şeitan Mihaela CNT M Miloşevici Larisa Lic.Odobleja M Iorga Cristian Şc.2 M Nuţă Lucian Bogdan Ţiţeica M Mureşan Bogdan Ţiţeica M Torsin Ligia Şc.Orşova M Chelu Adina Şc.2 M Clasa a IX-a Numele şi prenumele Scoala Premiul Profesor Numele şi prenumele Scoala Premiul Putan Cătălin Ţiţeica I Stretcu Daniel Tuce Andreea Ţiţeica M Nicolescu Alexandra Cnt II Gimoiu Iuliana Bechir Adriana Cnt M Dunăreanu Lidia Ţiţeica III Stretcu Daniel Trancotă Ioan Andrei Vînju-Mare M Nistor Ovidiu Cnt III Cainiceanu Gheorghe Nica Flavius Cnt M Ştefănoiu Anca Cnt III Cainiceanu Gheorghe Oprea Radu Cnt M Popescu Alina Cnt M Răducu Alexandru Cnt M Mutu Marie-Jeanne Cnt M Robu Alexandru Cnt M Purcaru Alin Ţiţeica M Ciouca Eugen Cnt M Butaru Nicu Cnt M Creţu Adina Ţiţeica M Echim Ioana Cnt M Lincan Dan Cnt M Picu Vulpaşin Bianca Cnt M Clasa a X-a Numele şi prenumele Scoala Premiul Profesor Numele şi prenumele Scoala Premiul Tigora Andrei CNT I Cainiceanu Gheorghe Răveanu Ioana CNT M Coandă Oana CNT II Cainiceanu Gheorghe Suselea Robert CNT M Tuta Leontin CNT II Cainiceanu Gheorghe Gal Raluca CNT M Ciocea Marina CNT II Giugiuc Leonard Belbu Loredana CNT M Gogoloiu Gabriela Ţiţeica M Mariescu Radu CNT M Porojean Otilia CNT M Prundeanu Andreea CNT M Rachieru Adrian CNT M Sandu Oana CNT M Rădoi Alexandra Ţiţeica M Jiplea Bogdan CNT M

Page 71: RMM 6.pdf

Concursuri

- 64 -

Clasa a XI-a Numele şi prenumele Scoala Premiul Profesor Numele şi prenumele Scoala Premiul Rapcea Mihai CNT I Paponiu Dana Ivanovici Daniela Ţiţeica M Roşu Ştefan CNT I Paponiu Dana Sbîrcea Răzvan CNT M Bobiţi Ruxandra Ţiţeica II Popescu Rodica Viaşu Mihai Odobleja M Dăgădiţă Monica CNT M Pleşu Georgiana Strehaia M Mareş Alexandra CNT M Plotogea Cătălin CNT M Gintaru Silviu Vînju-Mare M Ungureanu Ionuţ Ţiţeica M Strezariu Iulia Ţiţeica M Mănescu Teodora CNT M Bîzdoacă Mihai CNT M Popescu Cristina CNT M Voichiţa Viorel Strehaia M Turturea Roxana CNT M Clasa a XII-a Numele şi prenumele Scoala Premiul Profesor Numele şi prenumele Scoala Premiul Ungureanu Andrei CNT I Prajea Manuela Chilom George CNT M Băzăvan Eduard Ţiţeica I Stretcu Daniel Cretescu Oana CNT M Paşov Iulia CNT II Popescu Eleodor Luca Alexandru CNT M Mirea Theodor CNT M Mareş Flavia CNT M Tirtea Gabriela CNT M Stoica George CNT M Janca Andrei Ţiţeica M

Rezultate obtinute de elevii mehedinteni la concursuri de matematica in anul 2006

Olimpiada Internaţională de Matematică – Liubliana Slovenia Ungureanu Andrei Bogdan medalie de aur prof. Prajea Manuela Bazavan Eduard medalie de aur prof. Stretcu Daniel Olimpiada Balcanică de Matematică - Cipru Bazavan Eduard medalie de aur prof. Stretcu Daniel Olimpiada Nationala Ungureanu Andrei Bogdan premiul I prof Prajea Manuela medalie de aur SSM Bazavan Eduard premiul al III-lea prof. Stretcu Daniel medalie de aur SSM Nicolescu Alexandra mentiune prof. Gimoiu Iuliana medalie de argint Tesila Bianca medalie de bronz SSM prof. Zaman Irina Carapencea Constantin medalie de bronz SSM prof. Paponiu Dana Tigora Andrei medalie de bronz SSM prof. Cainiceanu Gheorghe Pasov Iulia medalie de bronz SSM prof.Popescu Eleodor

CONCURSUL NATIONAL AL. MYLLER, Iasi

Croitoru Razvan premiul al III-lea prof. Paponiu Dana Carapencea Constantin mentiune prof. Paponiu Dana Pit-Rada Andrei mentiune prof. Paponiu Dana

CONCURSUL INTERJUDETEAN GH.TITEICA, Craiova

Pit-Rada Andrei mentiune prof.Paponiu Dana Tigora Andrei mentiune prof.Cainiceanu Gheorghe

Page 72: RMM 6.pdf

Concursuri

- 65 -

H SSM

CONCURSUL INTERJUDETEAN ION CIOLAC , Craiova Rapcea Mihai premiul I prof.Paponiu Dana

CONCURSUL NATIONAL SCOALA CU CEAS, Rm. Vilcea

Proba individuala Sbarcea Alexandra premiul al III-lea prof. Prajea Manuela Gimoiu Ruxandra premiul al III-lea prof. Gimoiu Iuliana Andreescu Madalina premiul al III-lea prof. Antonie Rodica Mitroi Roxana mentiune prof. Cainiceanu Gheorghe Asproniu Robert mentiune prof. Cainiceanu Gheorghe Nicoara Calin mentiune prof. Paponiu Dana Seitan Mihaela mentiune prof. Paponiu Dana Duta Adrian mentiune prof. Paponiu Dana

Proba La ceas Voicu Razvan premiul al III-lea prof. Prajea Manuela Grosu Vlad premiul al II-lea prof. Cainiceanu Gheorghe Papa Florin premiul al III-lea prof. Paponiu Dana Nicoara Calin premiul al II-lea prof. Paponiu Dana

Concursul „Kangourou” CALIFICATI LA BARAJUL NATIONAL CANGURUL (clasele VII-XII)

Cepesi Cristian Decebal VII Bobiti Ruxandra Titeica XI Carapencea Constantin CNT VIII Rosu Stefan CNT XI Croitoru Razvan CNT VIII Bazavan Eduard Titeica XII Sosu Cristian CNT VIII Ungureanu Andrei CNT XII Stretcu Otilia Titeica VIII Enescu Flavius CNT XII Cretu Andrei CNT X Chilom George CNT XII Coanda Oana CNT X Priboi Cristian CNT XII

BARAJUL NATIONAL CANGURUL (clasele VII-XII), Bucuresti Numele şi prenumele Scoala Profesor Cl. punctaj Premiul

Cepesi Cristian CNT PupazaEcaterina 7 115.5 Premiul II - Tabara Poiana Pinului Sosu Cristian CNT Paponiu Dana 8 118 Premiul II - Tabara Poiana Pinului Carapencea C-tin CNT Paponiu Dana 8 110 Premiul II - Tabara Poiana Pinului Croitoru Razvan CNT Paponiu Dana 8 102.75 Premiul III Rosu Stefan Lucian CNT Paponiu Dana 11 75 Premiul III Bazavan Eduard Ţiţeica Stretcu Daniel 12 122.5 Premiul I - Excursie Franta

BARAJUL JUDETEAN CANGURUL (clasele V-VI) Clasa a II-a Clasa a V-a Popescu Maria Sc.3 Doana Cristina Popescu Cristiana Lic.Paulian Rapcea Lucian Vinju Mare Vitus Eduard Sc.13 Baluta Adrian C.N.Traian Popescu Simona Sc. 2 Stanciu Alexandru Gen.6 Clasa a III-a Arbanasi Emil Gh.Titeica Prencea Cassian Lic.Paulian Nuta Flavius C.N.Traian Marghescu Gabriel Nicola Silviu C.N.Traian Dirpes Iulian Sc. 2 Borcean Diana Severinesti Marculescu Cristina Sc. 13 Ciontea Stefan Gh.Titeica Clasa a IV-a Stefan Andrei C.N.Traian Dragomir Andrei Lic.Paulian Clasa a VI-a Mema Serban Lic.Paulian Grosu Vlad C.N.Traian Floarea Ionut Strehaia Clain Andrei Catalin C.N.Traian Mihutescu Diana Strehaia Rosca Ioana C.N.Traian Stoichita Andreea Strehaia Pirvulescu Eleodor C.N.Traian Ciuta Cora Lic.Paulian Ciocoteala Alexandru C.N.Traian Gevelica Simon C.N.Traian Tudosie Dorin

Page 73: RMM 6.pdf

Concursuri

- 66 -

Elevi care au primit diplome din partea American Mathematics Competitions

AMC 12,AIME

Ungureanu Andrei Bogdan 239p CNT prof. Prajea Manuela Carapencea Constantin 168p CNT prof. Paponiu Dana Pasov Iulia 159,5p CNT prof.Popescu Eleodor Mateisescu Alexandru 159p CNT prof. Giugiuc Leonard Meilescu Simona 159p CNT prof. Paponiu Dana Sfetcu Adina 155,5p CNT prof. Paponiu Dana Rapcea Mihai 155p CNT prof. Paponiu Dana Chilom George 152,5p CNT prof.Popescu Eleodor Turturea Roxana 150,5p CNT prof. Gimoiu Iuliana Rosu Stefan 150p CNT prof. Paponiu Dana Croitoru Razvan 148p CNT prof. Paponiu Dana Golenteanu Magdalena 141p CNT prof. Prajea Manuela Achimescu Andreea 140p CNT prof. Prajea Manuela Togoe Bogdan 138p CNT prof. Prajea Manuela Bazavan Cristina 135,5p CNT prof. Prajea Manuela Bocse Bogdan 134,5p CNT prof. Paponiu Dana Bejinaru Amalia 122p CNT prof. Paponiu Dana Dagadita Monica 122p CNT prof. Paponiu Dana

AMC 10, AIME Tigora Andrei 210p CNT prof. Cainiceanu G.

AMC 8 Carapencea Constantin 21p CNT prof. Paponiu Dana Andreca Mihai 17p CNT prof. Paponiu Dana Pit-Rada Andrei 17p CNT prof. Paponiu Dana Capraru Alexandru 16p CNT prof. Paponiu Dana Duta Adrian 16p CNT prof. Paponiu Dana Zorocliu Andra 16p CNT prof. Paponiu Dana Agape Mihai 15p CNT prof. Paponiu Dana Grosu Vlad 13p CNT prof. Cainiceanu G. Asproniu Robert 10p CNT prof. Cainiceanu G. Vasiu Mihnea 10p CNT prof. Antonie Rodica Prunescu Flavius 10p CNT prof. Cainiceanu G. Boescu Anca 9p CNT prof. Cainiceanu G. Lungu Amelin 9p CNT prof. Cainiceanu G.

Elevi calificati la Concursul anual al rezolvitorilor Gazetei Matematice, Pitesti , august 2006

Pit-Rada Andrei VIII CNT Raveanu Ioana X CNT

Page 74: RMM 6.pdf

REZOLVITORI

H SSM

- 67 -

Colegiul National Traian Prof. Gh.Cainiceanu - clasa a VI-a Pavel Cristian [8], Mituca Anda [8], Pisleaga Anemona [8], Toma Octavia [8], Oprita Geanina [8],Capitanescu Madalina [8], Mitroi Roxana [8], Boescu Anca [8] - clasa a IX-a Tanasescu Vlad [10] - clasa a X-a Susnea George [10], Stavaru Daniel [10], Cretu Andrei [10], Bojinovici Sergiu [10], Badea Sabin [10], Ghiga Octavian [10], Prundeanu Andreea [10], Pasat Cristina [10], Vladu Margareta [10], Tuta Leontin [10], Modalca Denisa [10], Georgescu Alin [10], Ionica Adina [10], Porojan Otilia [10], Gamala Andreea [10], Mariescu Radu [10], Gociu Andrei [10], Rachieru Adrian [10], Cotet Alexandra [10], Abagiu Aurel [10], Cucu Irina [10], Cristea Valentina [10], Lupu Olivia [10], Trasca Mihai [10], Negrea Nicolae [10], Saftoiu Mihai [10], Buncianu Ilie [10], Marin Daniela [10], Stoica Raluca [10], Hanes Anca [10], Pitulicu Lorena [10], Dogaru Oana [10], Ganda Irina [10], Cainiceanu Andrei [10], Urse Ruxandra [10],Hinoveanu Catalin [10], Belbu Loredana [10], Mituca Atena [10], Tomescu Liana [10], Nitulescu Maria [10].

Liceul Teoretic “Gh. “Ţiţeica” Prof. Irina Zăman – clasa a V-a Doană Cristina [17], Fonea Dan [5], Fonea Alex [5], Florescu Alex [6], Săbieşan Roxana [5], Arbănaşi Emil [7], Ghiţu Alex [11], Mituleţu Dana [4], Safta Roxana [5] – clasa a VII-a Teşilă Bianca [9], Mură Ion [5], Chereşdi Ion [6], Ştefan Radu [6], Botoşanu Cornel [6], Troncuţă Paul [6].

Scoala generală Nr.11 Prof. Victor Saceanu - clasa a-V-a Bica Claudia; Cafadaru Emanuel; Camenita Daniel; Matinescu Ionut; Scarlat Liliana; Stanciulescu Sidonica; Ticana Georgiana; Mihaescu Catalin; Matu Diana - clasa a-VI-a Covresvu Laurentiu; Cilihiu Claudiu; Gavrila Costinel; Iancu Nicusor; Dumitrascu Ramona; Popescu Laurentiu - clasa a-VII-a Buzatu Damian; Carjoi Valentin; Cucu Bianca; Ghita Gabriela; Mladin Elvira; Munteanu Stefan; Sarbu Valentina - clasa a-VIII-a Avramescu Alexandra; Bacioiu Loredana; Cimpoieru Diana; Mosneanu Angela; Roman Georgiana; Zaharescu Victor.

Page 75: RMM 6.pdf

COLABORATORI

- 68 -

prof. Daniel Sitaru prof. Manuela Opriţa prof. Alexandru Szoros prof. Manuela Prajea prof. Angela Niţoiu prof. Marica Piţ-Rada prof. Antonie Rodica prof. Nedeianu Dan prof. Băloi Valeria prof. Ovidiu Ticusi prof. Calafeteanu Gheorghe prof. Paponiu Dana prof. Dan Daniel prof. Popescu Octavian Tudor prof. Dan Nedeianu prof. Stefan Marica prof. Daniel Sitaru, prof. Stoican Victor prof. Draga Tătucu Mariana prof. Stretcu Daniel prof. Draga Tătucu Porfirel prof. Victor Saceanu prof. Eleodor Popescu inst. Maria Ungureanu prof. Florentina Roman inst. Mariana Cornea prof. Gheorghe Cainiceanu inst. Niculina Opriţa prof. Giugiuc Constantin drd. Laviniu Bejenaru prof. Gorun Sanda elev Noël Negrea prof. Ion Chilea elev Aida Giurcan prof. Ionel-Vasile Piţ-Rada elev Otilia Stretcu prof. Iuliana Gimoiu elev Rapcea Mihai prof. Leonard Giugiuc elev Bocşe Bogdan prof. Lucian Dragomir drd. Iuliana Radu

Page 76: RMM 6.pdf

REZOLVITORI

H SSM

- 69 -

Colectivul de redacţie: Gheorghe Căiniceanu

Dan Daniel Manuela Prajea Dana Paponiu Nănuţi Dan Octavian Ungureanu Dan Nedeianu Popescu Eleodor Sitaru Dan Săceanu Victor Chilea Ion

Decembrie 2006